PATHOLOGY QUESTIONS AND ANSWERS
Prof. Dr. Murat Alper
www.adscientificindex.com
SECTION 1: CELLULAR INJURY
1. What is the first cellular damage caused by hypoxia?
A) Plasma membrane injury
B) Release of intracellular lysosomes
C) Impairment of aerobic respiration
D) Intracellular calcium accumulation
E) Detachment of ribosomes from the rough endoplasmic reticulum
(Answer: C)
2. Which of the following is the earliest cellular change observed in ischemic and hypoxic injury?
A) Fatty change
B) Cytolysis
C) Hemosiderin accumulation
D) Apoptosis
E) Cellular swelling
(Answer: E)
3. Which of the following is the organelle first affected and the substance accumulated during cellular
swelling?
A) Rough endoplasmic reticulum - protein
B) Smooth endoplasmic reticulum - water
C) Ribosome - protein
D) Mitochondria - water
E) Mitochondria - protein
(Answer: D)
4. At which stage of cellular injury does the damage become irreversible?
A) Cellular swelling
1 Pathology Questions and Answers, Prof.Dr. Murat ALPER
B) Membrane damage
C) Endoplasmic reticulum swelling
D) Nuclear chromatin clumping
E) Ribosomal disintegration
(Answer: B)
5. Which of the following acts as a mediator in the structural changes during cell death?
A) Hydrogen
B) Oxygen
C) Nitrogen
D) Magnesium
E) Calcium
The initial event in hypoxia is the inhibition of the cell's aerobic respiration, particularly in the
mitochondria, leading to decreased oxidative phosphorylation and ATP production. ATP loss reduces
the activity of the plasma membrane's Na+/K+ ATPase, resulting in sodium retention and potassium
loss, accompanied by isosmotic water accumulation, leading to acute cellular swelling. The decrease
in ATP is associated with an increase in AMP, stimulating phosphofructokinase and accelerating
anaerobic glycolysis to produce ATP from glycogen, which leads to the accumulation of lactic acid and
phosphate, lowering intracellular pH. As hypoxia persists, ribosomes detach from the rough
endoplasmic reticulum, and polysomes dissociate into monosomes. Continued hypoxia leads to
mitochondrial swelling due to water uptake, endoplasmic reticulum dilation, and overall cell
enlargement. All these stages are reversible if oxygen is restored. The light microscopic features of
reversible cell injury include cellular swelling and fatty change. Prolonged ischemia results in
irreversible changes, such as excessive mitochondrial vacuolization, plasma membrane damage,
lysosomal rupture, and autolysis, ultimately leading to cell death mediated by calcium accumulation
in the mitochondria. Calcium activates four enzymes: phospholipase (which breaks down
phospholipids), ATPase (which decreases ATP levels), endonuclease (which damages nuclear
chromatin), and protease (which degrades membrane and cytoskeletal proteins). Glycine protects
hypoxic cells from irreversible membrane damage. (Answer: E)
6. Which of the following is an early event directly associated with ATP depletion in hypoxic cell
injury?
A) Lipid peroxidation
B) Nuclear pyknosis
C) Cellular swelling
D) Free radical formation
2 Pathology Questions and Answers, Prof.Dr. Murat ALPER
E) Membrane damage
ATP-dependent sodium pump failure leads to water influx and cellular swelling, an early and
reversible sign of hypoxic injury. Lipid peroxidation primarily affects membranes surrounding cells and
organelles due to free radical damage. Nuclear pyknosis is calcium-mediated. Free radical formation
in hypoxic injury is seen in damaged mitochondria. Membrane damage is a late event associated with
free radical formation and calcium-dependent activation of membrane phospholipases. (Answer: C)
7. Which of the following tissues is most susceptible to hypoxia when exposed to the same duration?
A) Myocardial cells
B) Fibroblasts
C) Proximal tubular epithelium
D) Hepatocytes
E) Neurons
Neurons in the nervous system suffer irreversible damage within 3-4 minutes of ischemia. Myocardial
cells, although more resilient than neurons, are still highly sensitive to anoxia. Fibroblasts are
relatively resistant. The proximal tubular epithelium of the nephron is more sensitive to hypoxia than
other segments. (Answer: E)
8. Which intracellular change is characteristic of severe ischemic injury?
A) Decreased calcium
B) Decreased pH
C) Decreased sodium
D) Increased ATP
E) Increased Na+/K+ pump activity
Severe ischemic injury leads to a decrease in intracellular pH due to mitochondrial damage and
reduced oxidative phosphorylation, resulting in decreased ATP synthesis. Reduced ATP levels trigger
glycolysis, lactic acid production, and further pH decline. ATP depletion also decreases the activity of
the membrane-associated Na+/K+ pump, leading to water and sodium influx. The final stage of
severe ischemic injury involves a large influx of extracellular calcium, resulting in cell death.
Reperfusion injury occurs when blood flow is restored to a reversibly injured cell, potentially
worsening the damage by causing high calcium influx, leading to tissue damage via activation of
phospholipases, endonucleases, ATP, and proteases. Additionally, incoming inflammatory cells
increase free radical production, causing further membrane damage. (Answer: B)
3 Pathology Questions and Answers, Prof.Dr. Murat ALPER
9. The mechanism of cellular swelling in tissue hypoxia is most closely related to which of the
following?
A) Free radical toxicity
B) Decreased apolipoprotein synthesis
C) Calcium influx into mitochondria
D) Irreversible cell membrane damage
E) Decreased ATP concentration
In tissue hypoxia, the absence of oxygen impairs oxidative phosphorylation in mitochondria, leading
to ATP deficiency, which is the primary problem. ATP is crucial for the continuous operation of the
sodium-potassium ATPase pump, and its absence causes sodium and water influx, leading to cell
swelling. ATP deficiency also inhibits protein synthesis, including apolipoproteins, leading to fatty
change in the liver. The other options represent irreversible changes. (Answer: E)
10. Which of the following changes is reversible?
A) Fat necrosis
B) Amyloid accumulation
C) Hydropic change
D) Liquefactive necrosis
E) Apoptosis
(Answer: C)
11. What is the term for reversible cell swelling observed in renal tubules during prolonged open
surgery due to renal hypoperfusion?
A) Fibrinoid necrosis
B) Atrophy
C) Hydropic (vacuolar) change
D) Karyorrhexis
E) Coagulative necrosis
Prolonged surgeries often result in insufficient oxygenated blood supply, leading to reversible hypoxic
cell injury (hydropic change), characterized by sodium and water influx. If blood flow is restored,
4 Pathology Questions and Answers, Prof.Dr. Murat ALPER
swelling resolves, and the cell returns to its normal shape and function. Hydropic change is a
reversible condition. (Answer: C)
12. What is the primary function of the mitochondrion?
A) Detoxification
B) Energy production
C) Protein synthesis
D) Digestion of foreign particles
E) Enzyme production
The mitochondrion is the organelle responsible for generating the energy (ATP) required by the cell.
Mitochondria contain their own DNA, RNA, ribosomes, and proteins, and are the only organelles in
the cell capable of self-replication. (Answer: B)
13. Which organelle is primarily involved in autolysis?
A) Nucleus
B) Smooth endoplasmic reticulum
C) Nucleolus
D) Golgi complex
E) Lysosome
Lysosomes are membrane-bound organelles containing acid hydrolases responsible for breaking
down foreign particles and cellular debris, leading to autolysis. (Answer: E)
14. In which organelle does detoxification occur?
A) Lysosome
B) Ribosome
C) Golgi complex
D) Smooth endoplasmic reticulum
E) Rough endoplasmic reticulum
The smooth endoplasmic reticulum is the site of steroid synthesis and drug detoxification. The rough
endoplasmic reticulum, which has ribosomes attached to its outer surface, is responsible for the
5 Pathology Questions and Answers, Prof.Dr. Murat ALPER
synthesis of proteins and lysosomal enzymes for export from the cell. Ribosomes are small, non-
membranous organelles responsible for protein synthesis. Mitochondria are the site of ATP
production and are the only organelles capable of self-replication. Phosphorylation enzymes and the
electron transport system are located on the cristae of the mitochondrial inner membrane. (Answer:
D)
15. Which of the following organelles undergoes hypertrophy in the liver due to barbiturate
administration?
A) Mitochondria
B) Lysosome
C) Ribosome
D) Rough endoplasmic reticulum
E) Smooth endoplasmic reticulum
Barbiturates cause hypertrophy of the smooth endoplasmic reticulum in liver cells. This is related to
the phenomenon of tolerance, in which barbiturates are detoxified by oxidative demethylation in the
P-450 mixed-function oxidase system of the smooth endoplasmic reticulum. (Answer: E)
16. Which of the following is not an antioxidant?
A) Superoxide dismutase
B) Coagulase
C) Glutathione peroxidase
D) Catalase
E) Sulfhydryls
(Answer: B)
17. Which of the following increases in cells of the salivary gland, thyroid, parathyroid, and kidney,
leading to a characteristic eosinophilic appearance?
A) Endoplasmic reticulum
B) Mitochondria
C) Cytosol
D) Peroxisome
E) Golgi apparatus
6 Pathology Questions and Answers, Prof.Dr. Murat ALPER
In some benign tumors, mitochondrial hyperplasia results in an "oncocytic" eosinophilic appearance,
as seen in Hurthle cells in Hashimoto's thyroiditis. (Answer: B)
18. Which of the
following does not protect cells against free radicals?
A) Vitamin E
B) Proline
C) Ceruloplasmin
D) Cysteine
E) Glutathione
Free radicals are highly reactive and unstable chemical derivatives that contain an unpaired electron
in their outer orbit. They are generated in cells through the absorption of radiant energy, such as
ultraviolet or X-rays, by the Fenton reaction, normal physiological oxidation-reduction processes, or
enzymatic metabolism of exogenous chemicals. Free radicals can react with organic and inorganic
chemicals, leading to membrane lipid peroxidation, mitochondrial damage, DNA lesions, and protein
cross-linking, ultimately resulting in cellular injury. The organism neutralizes free radicals with
enzymes such as superoxide dismutase, glutathione peroxidase, and catalase, as well as antioxidants
like vitamin E and C, cysteine, glutathione, and ceruloplasmin. (Answer: B)
19. Which of the following does not neutralize free radicals?
A) Vitamin E
B) Vitamin A
C) Vitamin C
D) Beta-carotene
E) Pantothenic acid
All the options except pantothenic acid neutralize free radicals. Superoxide dismutase, vitamin C,
glutathione peroxidase, ferritin, and ceruloplasmin in the cytosol, and superoxide dismutase and
glutathione peroxidase in the mitochondria, as well as catalase in peroxisomes, contribute to free
radical neutralization. In all membranes, vitamin A and E, along with beta-carotene, neutralize free
radicals. (Answer: E)
20. Which of the following factors does not involve an increase in free radical formation in the
pathogenesis of tissue damage?
7 Pathology Questions and Answers, Prof.Dr. Murat ALPER
A) Oral ingestion of carbon tetrachloride (CCl4)
B) Ionizing radiation
C) Metastatic calcification
D) Oxygen toxicity
E) Reperfusion after ischemia
Metastatic calcification occurs when calcium deposits in normal or damaged tissues and organs
during hypercalcemia. CCl4 damage is a classic example of membrane damage due to free radical
formation, where CCl4 is converted to the active free radical CCl3- in the smooth endoplasmic
reticulum by the P-450 system. Ionizing radiation, oxygen toxicity, and reperfusion after ischemia are
examples of tissue damage due to free radicals. (Answer: C)
21. Free radical injury is primarily associated with which of the following?
A) Acetaminophen hepatotoxicity
B) Necrosis in immune vasculitis
C) Fatty change in the liver of an alcoholic patient
D) Granuloma formation in a patient with tuberculosis
E) Dystrophic calcification in acute pancreatitis
Acetaminophen is converted to a free radical by the hepatocyte cytochrome P-450 system,
responsible for widespread liver damage. Immune vasculitis shows fibrinoid necrosis, with
neutrophil-related free radical damage playing a minor role. Fatty change in alcoholic liver disease is
due to increased synthesis and decreased secretion of triglycerides. Granuloma formation in
tuberculosis is a cellular immune response related to the release of cytokines from macrophages and
helper T cells. Dystrophic calcification in acute pancreatitis occurs due to calcium deposition in
damaged tissues. (Answer: A)
22. The mechanism of mercury poisoning involves which of the following?
A) Interaction with DNA metabolism
B) Blockade of heme synthesis
C) Inhibition of oxygen release from hemoglobin
D) Inhibition of glucose metabolism
E) Inhibition of oxidative phosphorylation
8 Pathology Questions and Answers, Prof.Dr. Murat ALPER
Some chemicals act by directly binding to critical molecular components and organelles. In mercury
chloride poisoning, mercury binds to the sulfhydryl groups of cell membranes, increasing membrane
permeability and inhibiting ATPase-dependent transport. (Answer: E)
23. Which of the following causes retrolental fibroplasia in newborns?
A) Glutathione
B) Aspirin
C) Oxygen toxicity
D) CCl4
E) Superoxide dismutase
Oxygen toxicity leads to free radical formation, causing retrolental fibroplasia in newborns requiring
high-dose oxygen therapy in certain conditions. (Answer: C)
24. Which of the following causes acute toxic damage to the liver and severe cell injury?
A) Phenobarbital
B) Iron
C) Magnesium
D) CCl4
E) Carbon dioxide
Carbon tetrachloride (CCl4), used in the dry-cleaning industry, is converted to toxic CCl3 in the
smooth endoplasmic reticulum of the liver by P-450 mixed-function oxidases, leading to rapid and
severe liver cell injury, including fatty liver. (Answer: D)
25. Which of the following is the most common type of necrosis?
A) Liquefactive necrosis
B) Caseous necrosis
C) Coagulative necrosis
D) Enzymatic fat necrosis
E) Fibrinoid necrosis
(Answer: C)
9 Pathology Questions and Answers, Prof.Dr. Murat ALPER
26. Which type of necrosis is observed in acute myocardial infarction?
A) Coagulative necrosis
B) Caseous necrosis
C) Fibrinoid necrosis
D) Liquefactive necrosis
E) Fat necrosis
(Answer: A)
27. Which of the following organ-necrosis type pairings is incorrect?
A) Pancreas - Enzymatic fat necrosis
B) Heart - Coagulative necrosis
C) Spleen - Liquefactive necrosis
D) Brain - Liquefactive necrosis
E) Tuberculosis - Caseous necrosis
Lethal cell injury results in necrosis through progressive autolysis or protein denaturation. Coagulative
necrosis, the most common type, occurs through protein denaturation and maintains the basic cell
architecture for at least several days. Coagulative necrosis is characteristic of hypoxic cell death,
except in the brain. Myocardial, hepatic, renal, and splenic tissues exhibit coagulative necrosis, except
the brain. Liquefactive necrosis results from autolysis and heterolysis, commonly seen in focal
bacterial infections and unexplained brain necrosis. Caseous necrosis is associated with tuberculosis.
Enzymatic fat necrosis occurs in the pancreas, often associated with calcium deposition over time.
(Answer: C)
28. What is the most common cause of coagulative necrosis?
A) Autolytic digestion of intracellular enzymes
B) Protein deposition in arteriole walls
C) Granulomatous inflammation
D) Heterolytic digestion by invading bacteria
E) Ischemia due to the interruption of blood supply
Coagulative necrosis is initially identified by the preservation of tissue architecture and is most
commonly caused by ischemia due to the interruption of blood supply. (Answer: E)
10 Pathology Questions and Answers, Prof.Dr. Murat ALPER
29. Which of the following microscopic changes is seen in dry gangrene?
A) Coagulative necrosis
B) Fat necrosis
C) Caseous necrosis
D) Liquefactive necrosis
E) Apoptosis
(Answer: A)
30. Gas gangrene is associated with which type of necrosis?
A) Empyema
B) Tuberculosis
C) General body trauma
D) Air embolism
E) Clostridium infection
(Answer: E)
31. Which of the following is an example of coagulative necrosis?
A) Acute pancreatitis
B) Wet gangrene of a diabetic patient's toe
C) Henoch-Schönlein vasculitis
D) Ischemic acute tubular necrosis
E) Atherosclerotic brain infarction
In ischemic acute tubular necrosis, renal tubular cells undergo coagulative necrosis, sloughing off into
the lumen and contributing to oliguria. Acute pancreatitis results in enzymatic fat necrosis. Dry
gangrene involves coagulative necrosis, while wet gangrene with bacterial infection leads to
liquefactive necrosis. Henoch-Schönlein vasculitis is an example of fibrinoid necrosis, while brain
infarction represents liquefactive necrosis. (Answer: D)
32. Which vascular lesion is seen in PAN and SLE?
A) Fat necrosis
B) Fibrinoid necrosis
11 Pathology Questions and Answers, Prof.Dr. Murat ALPER
C) Coagulative necrosis
D) Caseous necrosis
E) Liquefactive necrosis
Fibrinoid necrosis occurs in the vascular walls of autoimmune diseases like polyarteritis nodosa (PAN)
and systemic lupus erythematosus (SLE), as well as in leukocytoclastic vasculitis, such as Henoch-
Schönlein purpura. (Answer: B)
33. What term describes localized soft areas in the brain due to middle cerebral artery occlusion?
A) Caseous necrosis
B) Liquefactive necrosis
C) Coagulative necrosis
D) Metastatic calcification
E) Fat necrosis
Brain infarctions typically present as soft areas. Liquefactive necrosis is common in the central
nervous system and frequently occurs in the area supplied by the middle cerebral artery. The term
"encephalomalacia" refers to softening of the brain, synonymous with brain infarction. (Answer: B)
34. Which pathology is most likely present in a patient with Mycobacterium tuberculosis infection in
the lymph node, displaying yellowish-white, cheese-like material?
A) Liquefactive necrosis
B) Caseous necrosis
C) Coagulative necrosis
D) Fat necrosis
E) Fibrinoid necrosis
Mycobacterium tuberculosis infection leads to caseous necrosis, characterized by yellowish-white,
cheese-like material in the lymph node. Similar lesions may also be seen with Histoplasma fungal
infections. Sarcoidosis does not involve caseous necrosis. (Answer: B)
35. Which type of
necrosis is observed in focal bacterial infections?
12 Pathology Questions and Answers, Prof.Dr. Murat ALPER
A) Enzymatic fat necrosis
B) Coagulative necrosis
C) Liquefactive necrosis
D) Fibrinoid necrosis
E) Caseous necrosis
(Answer: C)
36. Which type of necrosis is observed in abscesses?
A) Coagulative necrosis
B) Fat necrosis
C) Caseous necrosis
D) Fibrinoid necrosis
E) Liquefactive necrosis
An abscess is a localized accumulation of purulent inflammatory cells. Lytic enzymes released from
polymorphonuclear leukocytes cause liquefactive necrosis and cell disintegration. (Answer: E)
37. Which of the following are apoptotic cell fragments?
A) Councilman bodies
B) Mallory bodies
C) Lipofuscin
D) Russell bodies
E) Verocay bodies
(Answer: A)
38. Which intracellular hyaline degeneration example is an immunoglobulin?
A) Mallory bodies
B) Russell bodies
C) Councilman bodies
D) Crooke hyaline change
E) Hemosiderin
13 Pathology Questions and Answers, Prof.Dr. Murat ALPER
Russell bodies represent intracellular hyalinosis in plasma cells, consisting of immunoglobulins.
Councilman bodies are apoptotic cell fragments in hepatocytes. Civatte bodies are eosinophilic bodies
seen in the basal layer of the epidermis in lichen planus and are examples of apoptosis. Mallory
bodies are cytoplasmic inclusions rich in keratin, seen in alcoholic liver disease. Crooke hyaline change
occurs in basophilic cells of the pituitary gland in Cushing's syndrome. Verocay bodies are seen in
schwannomas. (Answer: B)
39. Which of the following is not an actin regulatory protein involved in actin-myosin interaction and
contraction?
A) Filamin
B) Gelsolin
C) Profilin
D) Calmodulin
E) Interferon-gamma
(Answer: E)
40. Which intermediate filament is abundant in connective tissue cells?
A) Keratin
B) Neurofilament
C) Desmin
D) Vimentin
E) Glial filaments
(Answer: D)
41. Which cytoplasmic bodies result from cytoskeletal filament condensation in hepatocytes of
chronic alcoholics?
A) Hemosiderin
B) Karyorrhexis
C) Apoptosis
D) Mallory bodies
E) Acidophilic bodies
14 Pathology Questions and Answers, Prof.Dr. Murat ALPER
Intermediate filaments are divided into five groups: keratin filaments (in epithelial cells),
neurofilaments (in neurons), desmin filaments (in muscle cells), vimentin filaments (in connective
tissue cells), and glial filaments (in astrocytes). Cells also contain thin filaments (actin and myosin) and
microtubules, all of which play roles in cell movement, migration, and leukocyte function. Some drugs
target actin filaments, such as cytochalasin B, which inhibits actin filament polymerization, and
phalloidin, a toxin from Amanita phalloides, which binds to actin filaments. In Wiskott-Aldrich
syndrome, defects in cytoskeleton-related lymphocyte antigen receptor binding are observed. In
chronic alcoholic patients, Mallory bodies or alcoholic hyalin represent cytoplasmic inclusions rich in
keratin intermediate filaments. Neurofibrillary tangles in Alzheimer's disease contain microtubule-
associated proteins and neurofilaments. (Answer: D)
42. Immotile cilia syndrome is expected to lead to which of the following conditions?
A) Heart failure
B) Bronchiectasis
C) Hepatitis
D) Cirrhosis
E) Pancreatitis
Immotile cilia syndrome results from dynein deficiency, leading to defective microtubule organization
in cells, causing infertility due to inhibited sperm motility and bronchiectasis due to defective
respiratory cilia. Microtubules, like microfilaments, are essential in various stages of leukocyte
migration and phagocytosis and are required for cell division. Colchicine and vinca alkaloids
negatively affect microtubules. If immotile cilia syndrome is accompanied by dextrocardia, it is called
Kartagener syndrome. (Answer: B)
43. What is the most common change observed in the thymus following corticosteroid therapy?
A) Atrophy
B) Hypoplasia
C) Metaplasia
D) Apoptosis
E) Necrosis
(Answer: D)
44. In which of the following events is apoptosis more likely than necrosis?
A) Thymic involution
B) Abnormal mitochondrial structure
15 Pathology Questions and Answers, Prof.Dr. Murat ALPER
C) Widespread tissue necrosis
D) Inflammatory infiltration
E) Pale cytoplasmic staining
Apoptosis involves the death of individual cells without inflammatory infiltration. Organelles remain
normal, with nuclear pyknosis and eosinophilic cytoplasmic staining observed. (Answer: A)
45. Which of the following is a characteristic of apoptosis that differs from necrosis?
A) Nuclear fragmentation
B) RNA reduction
C) Absence of inflammation
D) Membrane changes
E) Enzyme activation
(Answer: C)
46. Which of the following is associated with apoptosis?
A) Increased inflammatory cells in the tissue
B) Karyolysis
C) Decreased cytosolic calcium
D) Activation of endonucleases
E) Increased DNA synthesis
Apoptosis is characterized by the activation of endogenous endonucleases, leading to DNA
fragmentation. In apoptosis, chromatin aggregates within the nucleus, but this is not karyolysis, but
rather pyknosis. The nucleus may fragment. Apoptosis does not enhance the arrival of inflammatory
cells to the injured tissue and does not trigger an inflammatory response. It is associated with the
entry of calcium into the cytosol. Bcl-2 prevents apoptosis by inhibiting mitochondrial permeability
and stabilizing proteins like Apaf-1. Apoptotic bodies are phagocytosed by macrophages. Within the
Bcl-2 family, Bcl-XL inhibits apoptosis, while BAX and BAD accelerate it. Phosphatidylserine on the
surface of apoptotic cells is recognized by macrophages, promoting the phagocytosis of apoptotic
cells. (Answer: D)
47. The round, nucleus-free, acidophilic bodies formed in the liver of a patient infected with hepatitis
B virus are an example of which pathology?
A) Apoptosis
16 Pathology Questions and Answers, Prof.Dr. Murat ALPER
B) Atrophy
C) Pyknosis
D) Karyorrhexis
E) Karyolysis
In hepatitis B-infected liver, round, nucleus-free, acidophilic bodies, known as Councilman bodies, are
formed as a result of virus-induced apoptosis. (Answer: A)
48. In which of the following organs is hyperplasia not observed?
A) Liver
B) Spleen
C) Lung
D) Heart
E) Prostate
(Answer: D)
49. Which of the following is not an adaptation mechanism?
A) Atrophy
B) Hypertrophy
C) Metaplasia
D) Hypoplasia
E) Hyperplasia
(Answer: D)
50. Which of the following is irreversible?
A) Atrophy
B) Metaplasia
C) Mild dysplasia
D) Necrosis
E) Hypertrophy
17 Pathology Questions and Answers, Prof.Dr. Murat ALPER
Atrophy, hypertrophy, and metaplasia are adaptive mechanisms and are reversible. Hypoplasia,
aplasia, and agenesis are developmental defects, not adaptive mechanisms. Necrosis is irreversible.
(Answer: D)
51. Which of the following is not a cause of atrophy?
A) Decreased workload
B) Nerve transection
C) Ischemia
D) Aging
E) Obesity
(Answer: E)
52. What change occurs in the remaining kidney after unilateral nephrectomy?
A) Hypertrophy
B) Atrophy
C) Hormonal hyperplasia
D) Compensatory hyperplasia
E) Pathological hyperplasia
(Answer: D)
53. The wart on the skin caused by papillomavirus infection is an example of which condition?
A) Atrophy
B) Metaplasia
C) Hypoplasia
D) Hyperplasia
E) Hypertrophy
(Answer: D)
54. Which cellular adaptation is primarily responsible for the significant growth of the uterus during
pregnancy?
A) Hyperplasia
B) Hypertrophy
18 Pathology Questions and Answers, Prof.Dr. Murat ALPER
C) Atrophy
D) Metaplasia
E) Apoptosis
(Answer: B)
55. Which of the following changes in the uterus can be precancerous?
A) Atrophy
B) Necrosis
C) Hyperplasia
D) Hypertrophy
E) Apoptosis
(Answer: C)
56. The changes observed in the heart muscle of a patient with systemic hypertension are described
by which of the following?
A) Physiological hyperplasia
B) Pathological atrophy
C) Pathological hypertrophy
D) Pathological hyperplasia
E) Physiological atrophy
(Answer: C)
57. Which change is observed in the prostate epithelium following castration?
A) Atrophy
B) Metaplasia
C) Hypoplasia
D) Hyperplasia
E) Hypertrophy
(Answer: A)
58. What is the term for the transformation of one cell type into another, more resilient, mature cell
type?
19 Pathology Questions and Answers, Prof.Dr. Murat ALPER
A) Hyperplasia
B) Metaplasia
C) Dysplasia
D) Heterotopia
E) Hypoplasia
(Answer: B)
59. Which of the following is most commonly associated with metaplasia?
A) Developmental defect
) Immunological reaction
C) Viral infection
D) Chronic irritation
E) Somatic mutation
Loss of substance due to cellular shrinkage is known as atrophy. It can result from decreased
workload, reduced endocrine stimulation, nerve transection, malnutrition (caused by excessive TNF
production, which leads to decreased appetite and lipid loss), ischemia, pressure, and aging.
Hypertrophy is an increase in cell size, which can occur in both physiological and pathological
conditions. Hypertrophy is observed in striated muscle and cardiac muscle, while hyperplasia does
not occur. Hyperplasia is an increase in the number of cells within a tissue or organ. Growth factors
such as TGF-alpha, EGF-alpha, TNF-alpha, and IL-6 promote hyperplasia. Hypertrophy is stimulated by
mechanical factors like increased workload, growth factors (TGF-beta, fibroblast growth factor,
insulin-like growth factor-1), and vasoactive amines (alpha-adrenergic agonists, endothelin-1, and
angiotensin II). Hyperplasia can be physiological (hormonal or compensatory) or pathological, such as
endometrial hyperplasia in the uterus. Hormonal hyperplasia is seen in the mammary glands during
pregnancy and puberty and in the uterus during pregnancy. Compensatory hyperplasia is observed
after partial hepatectomy or unilateral nephrectomy, with changes in the remaining kidney.
Hyperplasia is also an important response in wound healing, involving fibroblast and vascular
proliferation. In HPV infections, hyperplasia occurs through growth factors, leading to wart formation.
Endometrial hyperplasia can be precancerous. Metaplasia is the reversible transformation of one
mature cell type into another, more resistant cell type, commonly occurring in epithelial tissues due
to chronic irritation. (Answer: D)
60. Which of the following is characterized by the presence of lipofuscin granules?
A) Atrophy
20 Pathology Questions and Answers, Prof.Dr. Murat ALPER
B) Hypertrophy
C) Hyperplasia
D) Metaplasia
E) Dysplasia
(Answer: A)
61. What is the cause of the small, brown heart observed in an elderly person?
A) Atrophy
B) Apoptosis
C) Glycogen accumulation
D) Hemosiderin accumulation
E) Fat necrosis
In the absence of cardiac pathology, the heart of an elderly person may shrink and turn brown, an
example of senile atrophy. The brown color is due to the accumulation of lipofuscin pigment in the
myocardium. (Answer: A)
62. Which change is observed in osteoporosis?
A) Metaplasia
B) Atrophy
C) Hypertrophy
D) Hyperplasia
E) Metastatic calcification
Osteoporosis reflects atrophy of trabecular and cortical bone. It is particularly common in
postmenopausal women, where bones become brittle and prone to fractures. (Answer: B)
63. Which of the following is primarily an example of hypoplasia rather than atrophy?
A) Small left ventricle in a newborn with heart failure
B) Loss of skeletal muscle mass in a limb immobilized in a cast
C) Loss of renal mass due to renal artery atherosclerosis
D) Muscle mass loss in a 90-year-old patient
21 Pathology Questions and Answers, Prof.Dr. Murat ALPER
E) Thinning of the adrenal cortex in hypopituitarism
Hypoplasia refers to incomplete or partial development of an organ or tissue. In this case, hypoplastic
left heart syndrome is being described. Atrophy, on the other hand, occurs in a normal organ that
subsequently shrinks. Loss of skeletal muscle mass, as seen in an immobilized limb, or thinning of the
adrenal cortex in hypopituitarism are well-known examples of atrophy. (Answer: A)
64. Which of the following does not increase in hypertrophic cardiac muscle?
A) EGR1
B) Phagosomes
C) β-myosin heavy chain
D) Embryonic contractile proteins
E) C-Fos
A hypertrophic cardiac cell typically forms as an adaptive response to conditions such as heart failure
or hypertension. It is larger than a normal cardiac cell, aiming for stronger contractions. The cell
contains a larger cytoplasm and a larger nucleus, with increased DNA and mRNA content.
Myofilaments and mitochondria are also increased, but phagosomes are not. The ANF gene,
expressed in the embryonic heart, disappears at birth but is reinduced during cardiac hypertrophy.
Beta-myosin heavy chain contracts more slowly and with less energy. The weight of the hypertrophic
heart varies depending on the etiology. In pulmonary hypertension and ischemic heart disease, it
ranges from 350-600 grams; in systemic hypertension, aortic stenosis, mitral regurgitation, or dilated
cardiomyopathy, it ranges from 400-800 grams; and in aortic regurgitation or hypertrophic
cardiomyopathy, it ranges from 600-1000 grams. (Answer: B)
65. What is the term for the changes observed in the muscles of a limb after excessive exercise in an
athlete?
A) Hyperplasia
B) Hypertrophy
C) Metaplasia
D) Glycogen accumulation
E) Pseudohypertrophy
Increased workload is the most important stimulus for hypertrophy in skeletal muscles. Striated
muscles are considered permanent cells. They respond to increased stress by hypertrophy.
Hyperplasia is a response seen in cells with the ability to divide. Pseudohypertrophy occurs in some
muscle diseases. (Answer: B)
22 Pathology Questions and Answers, Prof.Dr. Murat ALPER
66. In which of the following cells can both hyperplasia and hypertrophy occur?
A) Skeletal muscle
B) Smooth muscle
C) Cardiac muscle
D) Lower motor neuron
E) Lens cell
Smooth muscle is a stable cell type, normally in the G0 phase of the cell cycle, but it can be
stimulated to reenter the cell cycle, as seen in pregnancy with estrogen stimulation. Since stable cells
can reenter the cell cycle, they can undergo both hypertrophy and hyperplasia. Skeletal and cardiac
muscle respond to increased workload by hypertrophy. Lower motor neurons and lens cells are
examples of permanent cells. (Answer: B)
67. Which of the following is an example of hyperplasia rather than metaplasia?
A) Benign squamous epithelium in the true vocal cord
B) Bone formation in the area of previous muscle injury
C) Increased goblet cells in the stomach mucosa
D) Squamous epithelium in the bladder in schistosomiasis
E) Increased goblet cells in the bronchi of a smoker
Goblet cells are normally present in the bronchi, so their increase represents hyperplasia. Metaplasia
is the replacement of one mature cell type with another. Benign squamous epithelium in the true
vocal cord is an example of squamous metaplasia because the true vocal cord is normally lined with
pseudostratified ciliated columnar epithelium. The presence of bone in the area of muscle injury is
osseous metaplasia (myositis ossificans). The presence of goblet cells in the stomach mucosa is an
example of intestinal metaplasia. Schistosomiasis and bladder stones cause squamous metaplasia in
the bladder. (Answer: E)
68. Which of the following describes the pathology in myositis ossificans?
A) Hyperplasia
B) Hypertrophy
C) Metaplasia
D) Fat necrosis
E) Metastatic calcification
23 Pathology Questions and Answers, Prof.Dr. Murat ALPER
Myositis ossificans is characterized by the formation of bone trabeculae within skeletal muscle,
usually following trauma. It is an example of osseous metaplasia, which should not be confused with
osteogenic sarcoma. Unlike epithelial metaplasia, osseous metaplasia is irreversible. (Answer: C)
69. Which of the following is not a cause of systemic hemosiderosis?
A) Increased iron absorption from the diet
B) Impaired iron utilization
C) Hemolytic anemias
D) Transfusions
E) Ceruloplasmin deficiency
Ceruloplasmin is a copper-carrying protein associated with Wilson's disease. It is not related to iron
metabolism. It is also an acute phase reactant. (Answer: E)
70. Which of the following is a hemoglobin derivative pigment that does not contain iron?
A) Bilirubin
B) Hemosiderin
C) Melanin
D) Ferritin
E) Lipofuscin
(Answer: A)
71. Which of the following findings in aging is associated with free radical damage?
A) Cerebral atrophy
B) Prostatic hyperplasia
C) Stromal fat deposition in parenchymal organs
D) Solar elastosis in the dermis
E) Lipofuscin accumulation in parenchymal cells
(Answer: E)
72. Which of the following is not among endogenous or exogenous pigments?
24 Pathology Questions and Answers, Prof.Dr. Murat ALPER
A) Carbon
B) Lysine
C) Melanin
D) Hemosiderin
E) Lipofuscin
The most common exogenous pigment is carbon or coal dust. Endogenous pigments include
lipofuscin, melanin, and hemosiderin. Lipofuscin is a product of lipid peroxidation that accumulates
progressively in atrophy and aging. Melanin is an endogenous, non-hemoglobin-derived brown-black
pigment formed in melanocytes through the oxidation of tyrosine by tyrosinase to
dihydroxyphenylalanine. Melanin is the only endogenous brown-black pigment. Hemosiderin is an
intracellular storage form of iron, a hemoglobin-derived yellow-brown pigment. Hemosiderin pigment
represents aggregates of ferritin micelles and is best known in localized hemosiderosis, such as a
bruise.
Hematin is a brown-black pigment and a hemoglobin derivative. Despite containing ferric iron, it
does not stain with Prussian blue. Bilirubin is a hemoglobin derivative but does not contain iron. In
alkaptonuria, black pigment accumulates in tissues due to the deposition of homogentisic acid,
leading to ochronosis. (Answer: B)
73. In which of the following diseases is a black, non-melanin pigment found?
A) Addison's disease
B) Nevus cell nevus
C) Black lung disease
D) Malignant melanoma
E) Adrenogenital syndrome
Black lungs develop due to the accumulation of coal dust, or anthracotic pigment, in alveolar
macrophages, lung tissue, and hilar lymph nodes. In Addison's disease and adrenogenital syndrome,
there is a decrease in cortisol synthesis. The loss of cortisol's negative feedback on ACTH increases
ACTH production and, with it, MSH, leading to increased skin pigmentation. Nevus cell nevi and
malignant melanoma are benign and malignant disorders of melanocytes, respectively. (Answer: C)
74. Lipofuscin pigment is formed as a result of which of the following processes?
A) Melanin metabolism
B) Heterophagy
25 Pathology Questions and Answers, Prof.Dr. Murat ALPER
C) Iron oxidation
D) Lipid peroxidation
E) Glycogenolysis
(Answer: D)
75. In which of the following conditions is a non-hemoglobin-derived pigment found?
A) "Heart failure" cells in sputum
B) Brown atrophy of the heart in an elderly patient
C) Jaundice in a patient with hepatitis
D) Kernicterus in a newborn
E) Pink-colored urine in a patient with hemolysis
Brown atrophy is due to the accumulation of lipofuscin, indigestible lipid residues, in atrophic cells.
Heart failure cells are alveolar macrophages containing hemosiderin derived from the breakdown of
erythrocytes in congested lungs. Kernicterus results from the accumulation of bilirubin in the brain.
Pink-colored urine in a hemolytic patient is due to hemoglobinuria. (Answer: B)
76. In which of the following conditions is dystrophic calcification observed?
A) Sarcoidosis
B) Papillary thyroid carcinoma
C) Vitamin D intoxication
D) Renal osteodystrophy
E) Multiple myeloma
(Answer: B)
77. Which of the following is not an example of dystrophic calcification?
A) Atheromatous plaque
B) Calcification of a heart valve
C) Sarcoidosis
D) Enzymatic fat necrosis
26 Pathology Questions and Answers, Prof.Dr. Murat ALPER
Dystrophic calcification occurs in damaged tissues with normal calcium metabolism and normal
serum calcium levels, such as in coagulative, caseous, liquefactive, and fat necrosis, atheromatous
plaques, and damaged heart valves. (Answer: C)
78. Which of the following conditions does not lead to metastatic calcification?
A) Sarcoidosis
B) Milk-alkali syndrome
C) Atheromatous plaque
D) Hyperparathyroidism
E) Vitamin D intoxication
Metastatic calcification occurs when calcium deposits in normal or damaged tissues and organs in
hypercalcemic conditions. It is associated with increased parathyroid hormone secretion, bone tissue
breakdown, vitamin D-related conditions, and renal failure. Hyperparathyroidism, vitamin D
intoxication, systemic sarcoidosis, multiple myeloma, and milk-alkali syndrome are associated with
metastatic calcification. (Answer: C)
79. Which of the following is more important in aging and carcinogenesis?
A) Telomerase
B) Phospholipase
C) Cyclin-dependent kinases
D) RAS
E) Endonuclease
In aging, 1) the Hayflick phenomenon refers to the cessation of cell division after approximately 50
divisions. 2) The genetic clock concept is important. Superoxide radical production also plays a role.
Ultimately, aging is a process influenced by both programmed events and environmental factors. Due
to the mechanism of DNA replication, each normal cell division results in a slightly shortened copy of
each chromosome. Telomeres are short, multiply repeated, unreplicated DNA sequences at the ends
of chromosomes. As somatic cells divide, a small portion of each telomere sequence is not copied,
leading to progressive shortening. Eventually, after many cell divisions, the severely shortened
telomeres are thought to signal cellular aging. Germ cells have telomerase activity, which repairs the
telomere shortening after each division, and this telomerase activity is also present in cancer cells,
preserving telomere length and enabling unlimited division. Thus, telomere length maintenance is
crucial in carcinogenesis. (Answer: A)
80. Which disease causes progeria?
27 Pathology Questions and Answers, Prof.Dr. Murat ALPER
A) Werner syndrome
B) Thyrotoxicosis
C) Hypothyroidism
D) Osteosarcoma
E) Retinoblastoma
Aging is influenced by genetic factors, including defects in DNA repair, as well as environmental
factors like free radical damage. Progeria refers to premature aging. Werner syndrome patients
experience accelerated aging due to a defect in helicase, a protein necessary for DNA replication and
repair. DNA repair defects are also seen in Cockayne syndrome and ataxia-telangiectasia, where aging
is accelerated. (Answer: A)
---
REVIEW
(Recall and write down your knowledge on the following topics)
- Reversible events in hypoxia
- Irreversible events in hypoxia
- Necrosis
- Apoptosis
- Coagulative necrosis
- Liquefactive necrosis
- Enzymatic fat necrosis
- Fibrinoid necrosis
- Free radicals
- Antioxidants
- Mercury poisoning
- Retrolental fibroplasia
- Adaptation mechanisms
- Metaplasia
- Atrophy
28 Pathology Questions and Answers, Prof.Dr. Murat ALPER
- Hypertrophy
- Hyperplasia
- Intermediate filaments
- Metastatic calcification
- Dystrophic calcification
- Russell body
- Civatte body
- Councilman body
- Mallory body
- Endogenous pigments
- Exogenous pigments
- Aging
- Telomerase
- Premature aging
29 Pathology Questions and Answers, Prof.Dr. Murat ALPER
SECTION 2: INFLAMMATION
1. Which cell responds earliest to chemotactic agents in an inflammatory response?
A) T lymphocyte
B) B lymphocyte
C) Histiocyte
D) Neutrophil
E) Plasma cell
(Answer: D)
2. Which of the following is not a feature of chronic inflammation?
A) Increased vascularization
B) Lymphocytes
C) Polymorphonuclear leukocytes
D) Histiocytes
E) Tissue destruction
Inflammation is the host's response to injurious agents and damaged tissue. The main components
of inflammation are vascular reactions and cellular responses. Chronic inflammation is characterized
by the presence of lymphocytes, histiocytes, plasma cells, increased vascularization, and fibrosis.
Polymorphonuclear leukocytes (PMNs) are typically associated with acute inflammation but can
persist in chronic infections like osteomyelitis and chronic lung disease caused by smoking.
(Answer: C)
3. Which of the following more strongly stimulates acute inflammation?
A) Autolysis
B) Dysplasia
C) Necrosis
D) Apoptosis
E) Metaplasia
(Answer: C)
30 Pathology Questions and Answers, Prof.Dr. Murat ALPER
4. Which of the following does not occur in response to local trauma?
A) Vasodilation
B) Increased permeability
C) Inhibition of chemotaxis
D) Leukocyte migration
E) Accumulation of exudate at the injury site
(Answer: C)
5. When leukocytes are mobilized during acute inflammation, what is the first intravascular event?
A) Margination
B) Emigration
C) Diapedesis
D) Phagocytosis
E) Opsonization
(Answer: A)
6. What term is used to describe the peripheral positioning of leukocytes along the vascular
endothelium?
A) Stasis
B) Transmigration
C) Exudation
D) Adhesion
E) Margination
(Answer: E)
7. Which of the following best defines the process by which polymorphonuclear neutrophils are
directed to the site of injury during acute inflammation?
A) Active hyperemia
B) Karyorrhexis
C) Passive hyperemia
D) Chemotaxis
E) Karyolysis
31 Pathology Questions and Answers, Prof.Dr. Murat ALPER
(Answer: D)
8. Which of the following describes the active process in which arterioles dilate and blood flow
increases in skeletal muscle during exercise?
A) Congestion
B) Cyanosis
C) Edema
D) Hemorrhage
E) Hyperemia
(Answer: E)
9. Which of the following occurs first in acute inflammation?
A) Leukocyte margination
B) Transmigration
C) Vasodilation
D) Vasoconstriction
E) Exudation
Acute inflammation begins with transient vasoconstriction, followed by prolonged vasodilation,
which leads to increased blood flow and the characteristic heat and redness of inflammation. This is
followed by increased vascular permeability, leading to exudation of protein-rich fluid into the tissues.
(Answer: D)
10. Which of the following is not a circulating cell?
A) Neutrophil
B) Monocyte
C) Lymphocyte
D) Basophil
E) Mast cell
Circulating cells include neutrophils, monocytes, eosinophils, lymphocytes, basophils, and platelets.
Mast cells reside in connective tissue and are located near blood vessels.
32 Pathology Questions and Answers, Prof.Dr. Murat ALPER
(Answer: E)
11. Which of the following does not have a chemotactic effect on leukocytes?
A) Soluble bacterial products
B) Complement C5a
C) Leukotriene B4
D) Complement C3b
E) Interleukin-8
Chemotaxis is the movement of leukocytes toward a chemical signal. C5a and leukotriene B4 are
potent chemotactic factors for neutrophils. C3b acts as an opsonin but does not directly attract
leukocytes.
(Answer: D)
12. Which of the following is an inflammatory mediator derived from plasma proteins and not
synthesized by cells?
A) Histamine
B) Serotonin
C) C3a
D) Prostaglandins
E) Lysosomal enzymes
Mediators of inflammation can be derived from plasma proteins or synthesized by cells. C3a is a
plasma protein-derived mediator, while histamine, serotonin, and prostaglandins are cell-derived.
(Answer: C)
13. Which of the following chemical mediators contributes most to the major signs of acute
inflammation (rubor, calor, and tumor)?
A) NO
B) Bradykinin
C) C5a
D) Prostaglandins
E) Histamine
33 Pathology Questions and Answers, Prof.Dr. Murat ALPER
Histamine is a key mediator in acute inflammation, causing vasodilation and increased vascular
permeability, which lead to redness (rubor), heat (calor), and swelling (tumor).
(Answer: E)
14. Which of the following is most responsible for the redness observed in acute inflammation?
A) Edema
B) Leukocyte margination
C) Thrombosis
D) Blood vessel dilation
E) Fibrin deposition
(Answer: D)
15. Which of the following primarily triggers the rapid transient increase in vascular permeability
observed in tissue injury?
A) Serum albumin
B) Histamine
C) Complement
D) Hageman factor
E) Anaphylatoxin
(Answer: B)
16. Endothelial cells located in which of the following vessels are most responsive to inflammatory
mediators?
A) Veins
B) Venules
C) Arterioles
D) Muscular arteries
E) Elastic arteries
Venules are the most responsive to inflammatory mediators, particularly histamine, which causes
increased permeability by inducing endothelial cell contraction.
34 Pathology Questions and Answers, Prof.Dr. Murat ALPER
(Answer: B)
17. Which of the following mediators causes arteriolar dilation and is also found in basophils and
platelets?
A) Lipoxin
B) Histamine
C) Bradykinin
D) Cytokine
E) Leukotriene
Histamine, primarily found in mast cells, basophils, and platelets, causes arteriolar dilation and
increases venular permeability.
(Answer: B)
18. Which of the following mediators is involved in the transmission of pain signals, regulation of
blood pressure, stimulation of secretion from endocrine cells, and increased vascular permeability?
A) Substance P
B) Nitric oxide
C) Histamine
D) Platelet-activating factor
E) Bradykinin
Substance P, a neuropeptide, is involved in various processes, including pain transmission, blood
pressure regulation, and increasing vascular permeability.
(Answer: A)
19. Which of the following mediators causes arteriolar dilation and increased permeability of venules,
similar to histamine?
A) Prostaglandin
B) C3b
C) Leukotriene
D) Bradykinin
E) Platelet-activating factor
35 Pathology Questions and Answers, Prof.Dr. Murat ALPER
Bradykinin causes vasodilation and increases vascular permeability, similarly to histamine, but its
effects are short-lived.
(Answer: D)
20. Which of the following conditions is most likely associated with "toxic granulation and a left shift"
in a peripheral blood smear?
A) Allergic rhinitis
B) Acute appendicitis
C) Whooping cough
D) Rheumatoid arthritis
E) Pinworm infestation
Toxic granulation and a left shift (increased immature neutrophils) typically indicate a bacterial
infection, such as acute appendicitis.
(Answer: B)
21. Which of the following mediators directly causes tissue damage during acute inflammation?
A) Leukotriene B4
B) Tumor necrosis factor-alpha
C) Nitric oxide
D) Platelet-activating factor
E) Interleukin 3
Nitric oxide (NO) is a potent mediator that can cause direct tissue damage during inflammation by
generating free radicals.
(Answer: C)
22. Which of the following is a low-molecular-weight vasodilator produced by endothelial cells that
inhibits platelet aggregation?
A) Cytokine
B) Defensin
C) Nitric oxide
36 Pathology Questions and Answers, Prof.Dr. Murat ALPER
D) Selectin
E) Tumor necrosis factor
Nitric oxide (NO) causes relaxation of vascular smooth muscle
, leading to vasodilation, and inhibits platelet aggregation.
(Answer: C)
23. Which of the following lesions does not contain cholesterol or cholesterol esters?
A) Atherosclerosis
B) Xanthoma
C) Cholesterolosis
D) Niemann-Pick disease type C
E) Albinism
Cholesterol is a component of cell membranes, and it accumulates in conditions such as
atherosclerosis and xanthomas. Albinism does not involve cholesterol accumulation.
(Answer: E)
24. Which of the following lesions does not contain dense histiocytes?
A) Cholesterolosis
B) Whipple's disease
C) Xanthoma
D) Malakoplakia
E) Waldenström macroglobulinemia
Waldenström macroglobulinemia is a plasma cell dyscrasia, not a histiocytic lesion.
(Answer: E)
25. Which cell type predominates in the inflammatory response to viral infections?
A) Mast cell
37 Pathology Questions and Answers, Prof.Dr. Murat ALPER
B) Lymphocyte
C) Plasma cell
D) Eosinophil
E) Neutrophil
(Answer: B)
26. Which cell type predominates in the inflammatory response to protozoal parasites?
A) Mast cell
B) Lymphocyte
C) Plasma cell
D) Eosinophil
E) Neutrophil
(Answer: D)
27. Which of the following peripheral blood findings is most likely in patients with typhoid fever and
certain rickettsial diseases?
A) Leukocytosis
B) Leukopenia
C) Eosinophilia
D) Lymphocytosis
E) Eosinopenia
Leukopenia is commonly observed in typhoid fever and some rickettsial diseases.
(Answer: B)
28. In which of the following conditions is an increase in eosinophils in peripheral blood least
expected?
A) Strongyloidiasis
B) Hookworm infestation
C) Drug allergy
D) Contact dermatitis
E) Addison's disease
38 Pathology Questions and Answers, Prof.Dr. Murat ALPER
Contact dermatitis is a type IV hypersensitivity reaction and is not typically associated with
eosinophilia.
(Answer: D)
29. Which condition is characterized by absolute lymphocytosis but without atypical lymphocytes?
A) Pertussis
B) Cytomegalovirus infection
C) Epstein-Barr virus infection
D) Phenytoin use
E) Toxoplasmosis
Pertussis causes a significant lymphocytosis, but the lymphocytes are not atypical, unlike in
infections such as EBV or CMV.
(Answer: A)
30. A woman treated for erysipelas develops a fluctuating swelling in the subcutaneous tissue. The
tissue is likely to exhibit which type of necrosis?
A) Coagulation necrosis
B) Caseation necrosis
C) Fat necrosis
D) Liquefaction necrosis
E) Gangrenous necrosis
(Answer: D)
31. Which of the following is a localized purulent infection within the parenchyma of an organ?
A) Sinus
B) Fistula
C) Phlegmon
D) Empyema
E) Abscess
39 Pathology Questions and Answers, Prof.Dr. Murat ALPER
An abscess is a localized collection of pus within a tissue or organ, characterized by liquefactive
necrosis.
(Answer: E)
32. What is the term for the accumulation of pus in the pleural cavity, commonly seen as a
complication of staphylococcal pneumonia?
A) Septicemia
B) Leukocytosis
C) Abscess
D) Empyema
E) Wet gangrene
Empyema refers to pus accumulation in the pleural cavity, often due to an extension of a lung
abscess or pneumonia.
(Answer: D)
33. Which of the following substances is not directly involved in the killing of microbes?
A) Oxygen
B) Complement C3b
C) Lactoferrin
D) Defensin
E) Bacterial permeability-increasing protein
Complement C3b acts as an opsonin, aiding in phagocytosis but not directly killing microbes.
(Answer: B)
34. Which of the following is not a morphological pattern of acute inflammation?
A) Serous inflammation
B) Fibrinous inflammation
C) Suppurative inflammation
D) Ulcers
E) Granulomatous inflammation
40 Pathology Questions and Answers, Prof.Dr. Murat ALPER
Granulomatous inflammation is a pattern of chronic, not acute, inflammation.
(Answer: E)
35. Which type of inflammation is characteristic of diphtheria?
A) Fibrinous
B) Serous
C) Pseudomembranous
D) Catarrhal
E) Purulent
Pseudomembranous inflammation is characteristic of diphtheria, where a false membrane forms
over mucosal surfaces.
(Answer: C)
36. Which of the following is the most characteristic feature of granulomatous inflammation?
A) Epithelioid histiocytes
B) Necrosis
C) Lymphocytes
D) Eosinophils
E) Multinucleated giant cells
Epithelioid histiocytes are a hallmark of granulomatous inflammation, forming the core of the
granuloma.
(Answer: A)
37. The epithelioid cells found in granulomas originate from which of the following?
A) Mononuclear phagocytic cells
B) Neutrophils
C) Plasma cells
D) T lymphocytes
E) Mast cells
41 Pathology Questions and Answers, Prof.Dr. Murat ALPER
(Answer: A)
38. In which of the following diseases are Schaumann bodies and asteroid bodies found?
A) Berylliosis
B) Silicosis
C) Cat-scratch disease
D) Sarcoidosis
E) Schistosomiasis
(Answer: D)
39. Which of the following diseases is characterized by caseating granulomas?
A) Sarcoidosis
B) Berylliosis
C) Cat-scratch disease
D) Tuberculosis
E) Asbestosis
(Answer: D)
40. Which of the following diseases does not produce granulomatous inflammation?
A) Actinomycosis
B) Leprosy
C) Schistosomiasis
D) Cryptococcal infection
E) Granulosa cell tumor
Granulosa cell tumors are ovarian tumors and do not produce granulomatous inflammation.
(Answer: E)
41. In sarcoidosis, which of the following cells accumulates in the alveolar and interstitial spaces,
playing a crucial role in the immune response?
A) CD8+ cytotoxic T lymphocytes
42 Pathology Questions and Answers, Prof.Dr. Murat ALPER
B) CD4+ helper T lymphocytes
C) B lymphocytes
D) Natural killer (NK) cells
E) Plasma cells
(Answer: B)
42. Which type of giant cell is observed in tuberculosis?
A) Reed-Sternberg cell
B) Foreign body giant cell
C) Touton giant cell
D) Langhans giant cell
E) Langerhans giant cell
Langhans giant cells are characteristic of tuberculosis and are formed by the fusion of epithelioid
histiocytes.
(Answer: D)
43. Which of the following complements initiates the classical pathway by binding to an antigen-
antibody complex?
A) C1
B) C2
C) C3a
D) C3b
E) C5a
(Answer: A)
44. Which complement component is involved in the initiation of the alternative pathway?
A) C1
B) C2
C) C3b
D) C4
E) C5a
43 Pathology Questions and Answers, Prof.Dr. Murat ALPER
(Answer: C)
45. At which step do the classical and alternative pathways of the complement system converge?
A) C1
B) C2
C) C3
D) C4
E) C5
(Answer: C)
46. Which of the following is the most abundant complement component?
A) C1
B) C2
C) C3
D) C4
E) C5
(Answer: C)
47. Which of the following complement components is involved in opsonization?
A) C3a
B) C3b
C) C5a
D) C6
E) C7
C3b is an important opsonin that enhances the phagocytosis of pathogens.
(Answer: B)
48. Which of the following does not have chemotactic activity?
44 Pathology Questions and Answers, Prof.Dr. Murat ALPER
A) Leukotriene B4
B) PGE2
C) Bacterial products
D) Neutrophil cationic proteins
E) C5a
PGE2 is involved in vasodilation and fever but does not have chemotactic properties.
(Answer: B)
49. In which of the following processes does the complement system play a role?
A) Antibody-dependent cellular cytotoxicity
B) Mast cell degranulation
C) Delayed hypersensitivity reaction
D) Paroxysmal nocturnal hemoglobinuria (PNH)
E) Cytolysis by CD8 T cells
PNH is associated with a defect in complement regulation, leading to increased susceptibility to
complement-mediated lysis of red blood cells.
(Answer: D)
50. Which of the following complement components is incorrectly paired with its function?
A) C3a and C5a - Mast cell degranulation
B) C3b - Opsonization
C) C4a - Interleukin-1 release
D) C5a - Chemotaxis
E) C5b-9 - Cytolysis
C4a does not induce the release of interleukin-1. The other functions are correctly matched.
(Answer: C)
51. Which of the following arachidonic acid metabolites enhances chemotaxis?
45 Pathology Questions and Answers, Prof.Dr. Murat ALPER
A) Thromboxane A2
B) Prostacyclin I2
C) Leukotriene B4
D) Lipoxin
E) Prostaglandin E2
Leukotriene B4 is a potent chemotactic agent that attracts neutrophils to the site of inflammation.
(Answer: C)
52. Which of the following inflammatory mediators is involved in the development of fever?
A) Leukotriene B4
B) Bradykinin
C) Histamine
D) Interleukin 1
E) Interleukin 8
Interleukin 1 (IL-1) is a cytokine that plays a key role in fever production by acting on the
hypothalamus.
(Answer: D)
53. Which of the following inflammatory mediators is associated with the sensation of pain?
A) Bradykinin
B) C5a
C) Tumor necrosis factor
D) PGF2alpha
E) LTC4
Bradykinin and prostaglandins, particularly PGE2, are involved in pain signaling during
inflammation.
(Answer: A)
54. Which of the following acts as an opsonin?
46 Pathology Questions and Answers, Prof.Dr. Murat ALPER
A) Complement C3a
B) Complement C5a
C) IgM Fc fragment
D) IgG Fc fragment
E) PGE2
The Fc fragment of IgG acts as an opsonin by binding to pathogens and enhancing their recognition
and phagocytosis by immune cells.
(Answer: D)
55. Which of the following is involved in the resolution of acute inflammation?
A) Lipoxins
B) Platelet-activating factor
C) Kinin system
D) Tumor necrosis factor
E) TGF-alpha
Lipoxins are anti-inflammatory molecules derived from arachidonic acid that promote the
resolution of inflammation.
(Answer: A)
56. Which of the following cells contains high levels of myeloperoxidase?
A) Macrophage
B) Basophil
C) Eosinophil
D) Neutrophil
E) Epithelioid histiocyte
Myeloperoxidase is an enzyme found in the azurophilic granules of neutrophils and is involved in
the production of hypochlorous acid, a potent antimicrobial agent.
(Answer: D)
47 Pathology Questions and Answers, Prof.Dr. Murat ALPER
Here is the complete version of Section 2, including questions 57-70 and the review section:
---
SECTION 2: INFLAMMATION AND IMMUNE RESPONSE
57. Which of the following is found only in primary granules and not in other granules?
A) Histaminase
B) Lysozyme
C) Acid hydrolase
D) Alkaline phosphatase
E) Cationic proteins
Neutrophils and monocytes contain lysosomal granules. Neutrophils have primary (azurophilic) and
secondary granules. The primary granules, or azurophils, contain lysosomal enzymes such as acid
hydrolases, cationic proteins, phospholipase A2, myeloperoxidase, defensin, and lysozyme. Secondary
granules also contain lysozyme and defensin, but they do not contain acid hydrolases.
(Answer: C)
58. Which of the following cationic bactericidal proteins is released from the primary granules of
polymorphonuclear leukocytes?
A) Bradykinin
B) Cytokine
C) Selectin
D) Lipocortin
E) Defensin
Defensin is a cationic protein released from the primary granules (lysosomes) of polymorphonuclear
leukocytes (neutrophils). It increases vascular permeability and acts as a chemotactic agent.
(Answer: E)
59. Which of the following is not an effect of anaphylatoxins?
A) Vasoconstriction
48 Pathology Questions and Answers, Prof.Dr. Murat ALPER
B) Chemotaxis
C) Bronchoconstriction
D) Fibrinolysis
E) Increased vascular permeability
Anaphylatoxins like C3a and C5a are involved in bronchoconstriction, increased vascular permeability,
chemotaxis, fibrinolysis, and vasodilation, but not in vasoconstriction.
(Answer: A)
60. Which of the following decreases during inflammation and is useful as an indicator of protein
status?
A) C3 complement
B) CRP
C) Haptoglobin
D) Transferrin
E) Alpha1-antitrypsin
Transferrin decreases during inflammation because the liver shifts to producing other acute-phase
reactants. Transferrin has a half-life of about 10 days and is a useful indicator of protein status. The
other proteins listed generally increase during inflammation.
(Answer: D)
61. Which of the following is not part of the mononuclear phagocytic system?
A) Kupffer cell of the liver
B) Sinus histiocytes
C) Alveolar macrophages
D) Microglia
E) Plasma cells
The mononuclear phagocytic system (formerly known as the reticuloendothelial system) includes
cells derived from bone marrow stem cells, such as monocytes and macrophages in tissues like the
liver, lungs, spleen, and brain. Plasma cells are not part of this system.
(Answer: E)
49 Pathology Questions and Answers, Prof.Dr. Murat ALPER
62. Which of the following cells contain major basic protein and are protective against parasites?
A) Neutrophil
B) Lymphocyte
C) Basophil
D) Eosinophil
E) Plasma cell
Eosinophils are involved in immune reactions, particularly against parasites, and contain major basic
protein (MBP), which is toxic to parasites and can cause epithelial damage.
(Answer: D)
63. The coexistence of pulmonary emphysema and liver cirrhosis is most likely due to a deficiency in
which of the following?
A) Alpha-1-antitrypsin
B) Galactokinase
C) Glucose-6-phosphatase
D) Glucocerebrosidase
E) Phenylalanine hydroxylase
Alpha-1-antitrypsin deficiency can lead to both pulmonary emphysema and liver cirrhosis. This
condition results in the unchecked activity of neutrophil elastase, leading to tissue damage in the
lungs and liver.
(Answer: A)
64. Which of the following conditions primarily presents with leukocyte adhesion defects?
A) Hemodialysis
B) Thermal injury
C) Malignancy
D) Sepsis
E) Immunodeficiency
50 Pathology Questions and Answers, Prof.Dr. Murat ALPER
Hemodialysis and diabetes mellitus can lead to leukocyte adhesion defects, while thermal injury,
sepsis, and malignancy often present with chemotaxis defects.
(Answer: A)
65. NADPH oxidase deficiency is seen in which of the following diseases?
A) Chediak-Higashi syndrome
B) Bruton's disease
C) Wiskott-Aldrich syndrome
D) Chronic granulomatous disease
E) Agammaglobulinemia
Chronic granulomatous disease (CGD) is caused by a deficiency in NADPH oxidase, which leads to an
inability to produce reactive oxygen species needed to kill certain bacteria and fungi.
(Answer: D)
66. In a disease where oxidative metabolism during phagocytosis is defective, which pathogen is most
likely to cause frequent infections?
A) Mycobacterium tuberculosis
B) Mycoplasma pneumoniae
C) Pneumococcus
D) Rotavirus
E) Staphylococcus
Staphylococcus aureus, a catalase-positive organism, frequently infects patients with chronic
granulomatous disease (CGD), where oxidative metabolism is defective. These patients are more
susceptible to catalase-positive organisms.
(Answer: E)
67. Which syndrome involves delayed or reduced fusion of phagosomes with lysosomes, impairing
bacterial phagocytosis?
A) Chronic granulomatous disease
B) Job’s syndrome
C) Nezeloff syndrome
51 Pathology Questions and Answers, Prof.Dr. Murat ALPER
D) Chediak-Higashi syndrome
E) Leukocyte adhesion deficiency (LAD) type I
Chediak-Higashi syndrome is a rare autosomal recessive disorder characterized by defective
phagosome-lysosome fusion, resulting in impaired intracellular killing of bacteria. It is associated with
giant granules in neutrophils.
(Answer: D)
68. Recurrent bacterial infections and poor wound healing are most characteristic of which of the
following?
A) Nezeloff syndrome
B) Isolated IgA deficiency
C) Chronic granulomatous disease
D) Chediak-Higashi syndrome
E) Leukocyte adhesion deficiency (LAD) type I
Leukocyte adhesion deficiency (LAD) type I is characterized by recurrent bacterial infections and
impaired wound healing due to a defect in the integrin CD18, which is critical for leukocyte adhesion
and migration.
(Answer: E)
69. Recurrent cold staphylococcal abscesses in fair-skinned, red-haired girls are characteristic of
which of the following conditions?
A) Nezeloff syndrome
B) Isolated IgA deficiency
C) Chronic granulomatous disease
D) Chediak-Higashi syndrome
E) Job’s syndrome
Job's syndrome, also known as Hyper-IgE syndrome, is characterized by recurrent cold staphylococcal
abscesses, particularly in fair-skinned, red-haired individuals.
(Answer: E)
52 Pathology Questions and Answers, Prof.Dr. Murat ALPER
70. A 5-year-old boy has recurrent Staphylococcus aureus infections. Peripheral blood leukocytes
appear normal, but the Nitroblue Tetrazolium (NBT) dye test is abnormal. Which of the following best
explains the pathogenesis of his disease?
A) NADPH oxidase deficiency
B) Defective fusion of phagosomes and lysosomes
C) C1-INH deficiency
D) Myeloperoxidase deficiency
E) Immunoglobulin deficiency
The abnormal NBT test indicates a diagnosis of chronic granulomatous disease (CGD), which is caused
by NADPH oxidase deficiency, leading to defective respiratory burst and inability to kill ingested
pathogens effectively.
(Answer: A)
---
REVIEW TOPICS
- Opsonins: IgG, C3b
- Chemotactic factors: C5a, IL-8, Leukotriene B4
- Factors increasing vascular permeability: Histamine, Bradykinin, Leukotrienes C4, D4, E4
- Pain-inducing substances: Bradykinin, Prostaglandins (especially PGE2)
- Fever-inducing substances: IL-1, IL-6, TNF, Prostaglandins
- Effects of bradykinin: Vasodilation, increased vascular permeability, pain
- Components of the mononuclear phagocytic system: Macrophages, Kupffer cells, Microglia,
Langerhans cells
- Key cells in acute inflammation: Neutrophils
- Key cells in chronic inflammation: Lymphocytes, macrophages, plasma cells
- Abscess: Localized collection of pus, typically due to bacterial infection
- Predominant cells in viral infections: Lymphocytes
- Predominant cells in parasitic infections: Eosinophils
- Pus: Accumulation of dead neutrophils, bacteria, and tissue debris
- Empyema: Accumulation of pus in a body cavity, particularly the pleural space
53 Pathology Questions and Answers, Prof.Dr. Murat ALPER
- Gangrene: Death of body tissue due to lack of blood flow or a bacterial infection
Gangrene: Death of body tissue due to lack of blood flow or a bacterial infection
Effects of histamine: Vasodilation, increased vascular permeability, bronchoconstriction
Effects of C3a and C5a: Anaphylatoxins causing vasodilation, increased vascular permeability,
chemotaxis
Role of myeloperoxidase: Enzyme in neutrophils responsible for producing hypochlorous acid
during the respiratory burst
Emigration: Movement of leukocytes from the bloodstream to the site of inflammation
Adhesion: The binding of leukocytes to the endothelium during inflammation, primarily mediated
by integrins
Margination: The process by which leukocytes move to the periphery of the blood vessel, ready for
adhesion
Role of selectins: Mediate the initial rolling of leukocytes along the endothelium during
inflammation
Causes of granulomatous inflammation: Tuberculosis, sarcoidosis, leprosy, Crohn's disease, foreign
bodies
Lesions with dense histiocyte involvement: Xanthoma, Whipple's disease, Malakoplakia
Chronic granulomatous disease: Defect in NADPH oxidase, leading to impaired microbial killing
Disease with defective oxidative burst during phagocytosis: Chronic granulomatous disease
Nezeloff syndrome: A rare immunodeficiency disorder resembling DiGeorge syndrome
Isolated IgA deficiency: The most common primary immunodeficiency, often asymptomatic but can
present with recurrent infections
Chronic granulomatous disease: A condition where phagocytes cannot produce reactive oxygen
species
Chediak-Higashi syndrome: A disorder with impaired lysosomal trafficking leading to giant granules
in leukocytes
Job’s syndrome: Characterized by recurrent cold staphylococcal abscesses, eczema, and elevated
IgE levels
Leukocyte adhesion deficiency (LAD) type I: A disorder characterized by recurrent bacterial
infections due to defective leukocyte adhesion
54 Pathology Questions and Answers, Prof.Dr. Murat ALPER
SECTION 4: TISSUE REPAIR AND WOUND
HEALING
1. Which of the following organs has a higher rate of scar tissue formation in
response to inflammation?
A) Spleen
B) Skeletal muscle
C) Uterus
D) Liver
E) Lung
Skeletal muscle has a higher tendency to form scar tissue (fibrosis) in response
to inflammation because it is composed of permanent cells that do not readily
regenerate, leading to fibrosis rather than restoration of normal tissue architecture.
(Answer: B)
2. Which of the following cells is not a labile cell?
A) Kidney
B) Erythrocyte
C) Salivary gland duct epithelium
D) Skin
E) Pancreatic duct epithelium
Kidney cells are classified as stable cells, which do not continuously divide but
have the capacity to proliferate if stimulated. Labile cells are continuously dividing
cells, such as those in the skin, salivary glands, and pancreatic ducts.
(Answer: A)
3. Which of the following cells is classified as a stable but not a permanent
cell?
A) Hepatocyte
B) Kidney cell
55 Pathology Questions and Answers, Prof.Dr. Murat ALPER
C) Fibroblast
D) Smooth muscle
E) Skeletal muscle
Skeletal muscle cells are considered permanent cells because they do not
readily divide and regenerate. Stable cells, like hepatocytes and kidney cells, have a
low rate of turnover but can proliferate if necessary.
(Answer: E)
4. Which of the following is a permanent cell?
A) Endothelial cell
B) Hepatocyte
C) Ganglion cell
D) Smooth muscle cell
E) Kidney tubular cell
Ganglion cells (neurons) are considered permanent cells because they do not
divide and are not capable of regeneration. Damage to these cells typically results in
permanent loss of function.
(Answer: C)
5. What is the process of repairing damaged tissue with similar parenchymal
cells called?
A) Resolution
B) Granulation
C) Regeneration
D) Organization
E) Reparative
Regeneration refers to the process of replacing damaged tissue with the same
type of parenchymal cells, thereby restoring normal function.
56 Pathology Questions and Answers, Prof.Dr. Murat ALPER
(Answer: C)
6. Which of the following cells regenerates rapidly?
A) Labile cells
B) Stable cells
C) Permanent cells
D) Neurons
E) Hepatocytes
Labile cells, such as those in the skin, gastrointestinal tract, and bone marrow,
constantly divide and regenerate quickly.
(Answer: A)
7. Which of the following cells originates from the mesoderm?
A) Bone marrow cells
B) Liver epithelial cells
C) Gastrointestinal epithelium
D) Oligodendrocytes
E) Lung epithelial cells
Bone marrow cells originate from mesodermal progenitor cells, while liver
epithelial cells, gastrointestinal epithelium, and lung epithelial cells are derived from
the endoderm. Oligodendrocytes are derived from the ectoderm.
(Answer: A)
8. Which of the following cells is most sensitive to ionizing radiation?
A) Erythrocyte
B) Melanocyte
C) Cartilage cell
D) Intestinal crypt cells
57 Pathology Questions and Answers, Prof.Dr. Murat ALPER
E) Glial cells
Cells with a high turnover rate, such as intestinal crypt cells, are more sensitive
to ionizing radiation due to their rapid division and replication.
(Answer: D)
9. Which of the following is the most abundant adhesive glycoprotein in the
basement membrane?
A) Collagen type IV
B) Laminin
C) Heparan sulfate
D) Fibronectin
E) Dermatan sulfate
Laminin is the most abundant adhesive glycoprotein in the basement
membrane, playing a crucial role in the structure and function of the extracellular
matrix.
(Answer: B)
10. Which of the following is not found in the basement membrane?
A) Collagen type IV
B) Fibrin
C) Heparan sulfate
D) Laminin
E) Fibronectin
Fibrin is not normally found in the basement membrane; it is involved in blood
clotting and is deposited in tissues during pathological processes like inflammation.
(Answer: B)
58 Pathology Questions and Answers, Prof.Dr. Murat ALPER
11. Excessive vascular permeability in newly formed inflammatory granulation
tissue primarily leads to which of the following?
A) Edema
B) Ulcer
C) Fibrosis
D) Abscess formation
E) Necrosis
Excessive vascular permeability in granulation tissue leads to the accumulation
of fluid in the extracellular space, resulting in edema.
(Answer: A)
12. When do macrophages replace neutrophils during wound healing?
A) Within the first 24 hours
B) 24-48 hours
C) On the third day
D) On the fifth day
E) In the second week
Macrophages replace neutrophils around the third day of wound healing,
playing a crucial role in clearing debris and orchestrating the repair process.
(Answer: C)
13. Which factor promotes keratinocyte migration, angiogenesis, wound
contraction, and matrix deposition during wound healing?
A) Epidermal growth factor (EGF)
B) Fibroblast growth factor (FGF)
C) Vascular endothelial growth factor (VEGF)
D) Tumor necrosis factor (TNF)
E) Transforming growth factor-beta (TGF-β)
59 Pathology Questions and Answers, Prof.Dr. Murat ALPER
Fibroblast growth factor (FGF) is a potent angiogenic factor that also promotes
keratinocyte migration, wound contraction, and matrix deposition, all crucial steps in
wound healing.
(Answer: B)
14. When is the granulation tissue gap typically filled with granulation tissue
during wound healing?
A) Day 2
B) Day 3
C) Day 5
D) Day 7
E) Day 14
By the fifth day of wound healing, the incision gap is usually filled with
granulation tissue, characterized by new capillaries, fibroblast proliferation, and the
beginning of collagen deposition.
(Answer: C)
15. What is the most characteristic feature of granulation tissue?
A) Exudate
B) Histiocytes
C) Granuloma-like appearance
D) Fibroblast and vascular proliferation
E) Giant cells
Granulation tissue is characterized by the proliferation of fibroblasts and the
formation of new blood vessels, which contribute to tissue repair.
(Answer: D)
16. Which of the following conditions is associated with granulation tissue
formation?
A) Cat-scratch disease
60 Pathology Questions and Answers, Prof.Dr. Murat ALPER
B) Sarcoidosis
C) Histoplasmosis
D) Tuberculosis
E) Wound healing
Granulation tissue forms as part of the wound healing process, consisting of
new capillaries and fibroblasts that help restore damaged tissue. Granulomatous
inflammation, seen in conditions like sarcoidosis and tuberculosis, is a different
process.
(Answer: E)
17. Which of the following acts as a growth inhibitor for leukocytes and
epithelial cells, stimulates fibroblast and smooth muscle cell proliferation, is a potent
fibrogenic agent, and has strong anti-inflammatory effects?
A) VEGF
B) bFGF
C) EGF
D) PDGF
E) TGF-β
Transforming growth factor-beta (TGF-β) has multiple roles, including growth
inhibition for certain cells, stimulation of fibroblast proliferation, and strong anti-
inflammatory effects.
(Answer: E)
18. Which type of collagen is first deposited in granulation tissue?
A) Type I
B) Type III
C) Type IV
D) Type VI
E) Type VII
61 Pathology Questions and Answers, Prof.Dr. Murat ALPER
Type III collagen, which is more commonly found in embryonic tissues, is
initially deposited in granulation tissue. It is later replaced by the stronger Type I
collagen during tissue remodeling.
(Answer: B)
19. When does collagen synthesis by fibroblasts begin during wound healing?
A) Within the first 24 hours
B) 24-48 hours
C) 3-5 days
D) 1 week
E) 15 days
Collagen synthesis typically begins around the third to fifth day of wound
healing, helping to strengthen the tissue and promote proper healing.
(Answer: C)
20. What is the granulation tissue containing newly formed bone in fracture
healing called?
A) Callus
B) Fibrocartilage
C) Involucrum
D) Periosteal new bone
E) Fibromatosis
The term "callus" refers to the granulation tissue that forms during bone
healing, eventually leading to the formation of new bone at the fracture site.
(Answer: A)
21. The development of tensile strength during wound healing primarily
depends on which of the following?
A) Collagen content
62 Pathology Questions and Answers, Prof.Dr. Murat ALPER
B) Ascorbic acid content
C) Keloidogenic factor level
D) Scar contraction
E) Capillary vessel quantity
The tensile strength of a wound is mainly determined by the amount and
quality of collagen deposited in the tissue during healing.
(Answer: A)
22. Which of the following is a hypertrophic reaction with raised scar tissue
and dense collagen formation after injury?
A) Granuloma
B) Fibrosis
C) Desmoid
D) Keloid
E) Exuberant granulation
Keloids are raised, thickened scars that result from excessive collagen
deposition during wound healing, more common in individuals with darker skin tones.
(Answer: D)
23. Which of the following best describes the tumor-like scar tissue
development after ear piercing in a young woman?
A) Benign fibrous histiocytoma
B) Dermatofibrosarcoma protuberans
C) Fibroepithelial polyp
D) Keloid
E) Xanthoma
63 Pathology Questions and Answers, Prof.Dr. Murat ALPER
Keloids are excessive scar tissue formations that appear tumor-like and can
develop after procedures like ear piercing. They are more prevalent in individuals with
darker skin.
(Answer: D)
24. Which of the following correctly matches the tissue with its repair
process?
A) Liver: Hepatocytes stimulate fibrogenesis
B) Kidney: Renal tubular epithelium undergoes hyperplasia
C) Lung: Type I pneumocytes are responsible for repair
D) Brain: Fibroblasts are responsible for gliosis
E) Peripheral nervous system: Schwann cells are crucial for nerve regeneration
Schwann cells are essential for the regeneration of peripheral nerves, guiding
the growth of new axons after injury.
(Answer: E)
25. Which of the following factors correctly identifies conditions that lead to
abnormal collagen formation during wound healing?
A) Copper deficiency and scurvy
B) Zinc deficiency and tissue hypoxia
C) Infection and obesity
D) Marfan syndrome and anemia
E) Ehlers-Danlos syndrome and hypoxemia
Copper is necessary for collagen cross-linking, and scurvy is caused by a
vitamin C deficiency, which impairs collagen synthesis. Both conditions lead to
defective collagen formation.
(Answer: A)
26. Which of the following diseases is not caused by protein misfolding?
A) Cystic fibrosis
64 Pathology Questions and Answers, Prof.Dr. Murat ALPER
B) Familial hypercholesterolemia
C) Retinitis pigmentosa
D) Amyloidosis
E) Alpha-1 antitrypsin deficiency
Amyloidosis is characterized by the extracellular deposition of misfolded
proteins, but it is not directly caused by a single protein misfolding event leading to a
specific disease. The other conditions listed are all linked to specific misfolded
proteins.
(Answer: D)
Review Topics:
- Cell cycle: G1 (presynthetic), S (synthesis), G2 (premitotic), M (mitosis), G0
(resting phase)
- Labile cells: Cells that continuously divide, such as epithelial cells,
hematopoietic cells, and gastrointestinal cells.
- Stable cells: Cells that do not usually divide but can proliferate in response to
injury, such as hepatocytes, renal tubular cells, and fibroblasts.
- Permanent cells: Cells that do not divide and have limited regeneration
capacity, such as neurons and cardiac myocytes.
- Basement membrane components: Laminin, Collagen type IV, Heparan
sulfate, and Fibronectin.
- Laminin: A major adhesive glycoprotein in the basement membrane.
- Collagen types: Type I (most common in adults), Type III (first in granulation
tissue), Type IV (in the basement membrane).
- Emigration: The movement of leukocytes from the bloodstream to the site of
inflammation.
- Wound healing:
- Primary intention: Wound edges are brought together, typically seen in
surgical incisions.
- Secondary intention: Wound heals from the base up, typically seen in larger
wounds.
65 Pathology Questions and Answers, Prof.Dr. Murat ALPER
- Events on the 3rd day of wound healing: Macrophages replace neutrophils,
and granulation tissue begins to form.
- Granulation tissue: A vascular and cellular tissue that fills the wound during
the healing process.
- Keloid: An overgrowth of scar tissue that extends beyond the original wound
boundaries.
- Hypertrophic scar: A raised scar that stays within the boundaries of the
wound.
- Desmoid tumor: A type of aggressive fibromatosis, a fibrous tissue
overgrowth that can be locally invasive.
- Fibromatoses: Benign soft tissue growths, such as Dupuytren’s contracture or
plantar fibromatosis.
66 Pathology Questions and Answers, Prof.Dr. Murat ALPER
SECTION 4: INFLAMMATION
Here are the questions and explanations for the latest section:
---
SECTION 3: HEMODYNAMIC DISORDERS, THROMBOSIS, AND SHOCK
1. What is the term for a mass that forms as a result of hemorrhage into soft
tissue?
A) Ecchymosis
B) Purpura
C) Petechiae
D) Thrombus
E) Hematoma
A hematoma is a localized collection of blood outside the blood vessels,
usually in liquid form within the tissue. Hematomas are typically caused by injury,
which causes blood to leak from the damaged blood vessels. Ecchymosis is a
subcutaneous hematoma greater than 1-2 cm in diameter, typically seen as a bruise.
(Answer: E)
2. What is the term used when new capillaries, fibroblasts, and collagen form
around the periphery of a hematoma?
A) Recanalization
B) Thrombosis
C) Embolization
D) Clot dissolution
E) Organization
Organization refers to the process in which granulation tissue forms at the
periphery of a hematoma, leading to the formation of scar tissue. If this process
67 Pathology Questions and Answers, Prof.Dr. Murat ALPER
occurs within a blood vessel, it may result in partial recanalization, which restores
blood flow through the thrombus.
(Answer: E)
3. Which compartment contains the highest proportion of body fluid?
A) Intracellular
B) Interstitial
C) Extracellular
D) Plasma
E) Blood
The intracellular compartment contains the majority of the body's fluid,
making up approximately 2/3 of the total body water.
(Answer: A)
4. Rank the following compartments by the volume of fluid they contain, from
most to least:
A) Interstitial / Intracellular / Plasma
B) Plasma / Intracellular / Interstitial
C) Intracellular / Interstitial / Plasma
D) Intracellular / Plasma / Interstitial
E) Interstitial / Plasma / Intracellular
The correct order, from most to least fluid volume, is intracellular > interstitial
> plasma. The intracellular compartment has the largest volume of fluid, followed by
the interstitial compartment, and then the plasma.
(Answer: C)
5. Which of the following is incorrect regarding transudate?
A) It is low in protein.
B) It is due to hemodynamic imbalances.
68 Pathology Questions and Answers, Prof.Dr. Murat ALPER
C) It has a specific gravity less than 1012.
D) It is associated with inflammation.
E) It is protein-rich.
Transudate is typically associated with hemodynamic imbalances such as
increased hydrostatic pressure or decreased oncotic pressure, and it is low in protein.
It is not associated with inflammation, which is instead linked to exudate, a fluid high
in protein and cells.
(Answer: E)
6. Which of the following mechanisms is responsible for edema formation in
liver failure due to cirrhosis?
A) Hypocalcemia
B) Hypoproteinemia
C) Anemia
D) Hypertension
E) Hyponatremia
In cirrhosis, the liver's ability to produce albumin is impaired, leading to
hypoproteinemia and a decrease in plasma oncotic pressure, which results in edema.
(Answer: B)
7. Which of the following conditions causes edema through a mechanism
different from the others?
A) Congestive heart failure
B) Malnutrition
C) Nephrotic syndrome
D) Liver failure
E) Hypoalbuminemia
69 Pathology Questions and Answers, Prof.Dr. Murat ALPER
Congestive heart failure leads to edema primarily through increased
hydrostatic pressure, while the other conditions cause edema mainly due to
decreased oncotic pressure (hypoproteinemia).
(Answer: A)
8. Which of the following conditions leads to edema due to increased
hydrostatic pressure?
A) Right heart failure / Left heart failure
B) Left heart failure / Celiac disease
C) Pleural effusion in pneumonia / Ascites in cirrhosis
D) Cerebral edema in encephalitis / Cerebral edema in hyponatremia
E) Tissue swelling in bee sting / Pulmonary edema in ARDS
Both right and left heart failure lead to increased venous pressure and,
consequently, increased hydrostatic pressure, resulting in edema. Right heart failure
typically causes peripheral edema, while left heart failure causes pulmonary edema.
(Answer: A)
9. In which of the following situations would exudate be expected?
A) Pleural effusion in a patient with fever and lung consolidation
B) Inspiratory crackles at the lung base on the second day after acute
myocardial infarction
C) Ascites in a malnourished child
D) Pleural effusion (hydrothorax) in left heart failure
E) Presacral edema in congestive heart failure
Exudate is typically associated with inflammatory conditions, such as pleural
effusion secondary to pneumonia. The other conditions are associated with
transudate, which is low in protein and cells.
(Answer: A)
70 Pathology Questions and Answers, Prof.Dr. Murat ALPER
10. Which factor is most critical in the adhesion of platelets to subendothelial
collagen?
A) Hageman Factor
B) Von Willebrand factor
C) Platelet Factor 4
D) Ionized calcium
E) Fibronectin
Von Willebrand factor (VWF) is essential for platelet adhesion to
subendothelial collagen following vascular injury. Deficiencies in VWF lead to
impaired platelet adhesion and increased bleeding tendencies.
(Answer: B)
11. After platelets adhere to subendothelial collagen, which of the following is
critical for platelet aggregation?
A) ADP
B) ATP
C) Histamine
D) PGI2
E) P-selectin
ADP is released from activated platelets and stimulates the recruitment and
aggregation of additional platelets to the site of injury, promoting the formation of a
platelet plug.
(Answer: A)
12. Which of the following is not an anticoagulant?
A) Antithrombin III
B) Protein C
C) Kininogen
D) Streptokinase
71 Pathology Questions and Answers, Prof.Dr. Murat ALPER
E) Tissue plasminogen activator (tPA)
Kininogen is not an anticoagulant. The other options are either natural
anticoagulants (Antithrombin III, Protein C) or fibrinolytic agents (Streptokinase, tPA)
that help to dissolve clots.
(Answer: C)
13. In which bleeding disorder is the prothrombin time (PT) prolonged?
A) Platelet disorders
B) Hemophilia A
C) Vitamin K deficiency
D) Vascular changes
E) Thrombocytopenia
Vitamin K deficiency affects the synthesis of several clotting factors, including
Factors II, VII, IX, and X. This leads to a prolonged PT.
(Answer: C)
14. Which of the following conditions is associated with a high risk of
thrombosis?
A) Cardiomyopathy
B) Nephrotic syndrome
C) Oral contraceptive use
D) Sickle cell anemia
E) Atrial fibrillation
Atrial fibrillation is associated with stasis of blood in the atria, which increases
the risk of thrombosis and embolization. Other conditions like oral contraceptive use
and sickle cell anemia are associated with a lower risk of thrombosis.
(Answer: E)
72 Pathology Questions and Answers, Prof.Dr. Murat ALPER
15. In a patient under 50 with no predisposing conditions for
hypercoagulability, which genetic cause of thrombosis should be investigated?
A) Factor V Leiden mutation
B) Methylenetetrahydrofolate reductase (MTHFR) mutation
C) Antithrombin III deficiency
D) Protein C deficiency
E) Protein S deficiency
Factor V Leiden mutation is the most common genetic cause of thrombosis,
particularly in younger patients without other risk factors. It increases the risk of
recurrent venous thromboembolism.
(Answer: A)
16. Venous thrombi most commonly originate from which veins?
A) Femoral vein
B) Iliac vein
C) Popliteal vein
D) Deep calf veins
E) Superficial calf veins
Deep venous thrombi most often originate in the deep veins of the calf, which
can propagate proximally to involve the femoral and iliac veins.
(Answer: D)
17. Which of the following is more characteristic of arterial thrombi than
venous thrombi?
A) They develop in areas of stasis.
B) They consist primarily of fibrin, erythrocytes, and platelets.
C) They form red thrombi that adhere to the site of injury.
D) Warfarin is more protective than aspirin.
E) They occur at sites of turbulence near bifurcations.
73 Pathology Questions and Answers, Prof.Dr. Murat ALPER
Arterial thrombi typically form at sites of turbulence, such as at arterial
bifurcations or areas with atherosclerotic plaques, making them different from
venous thrombi, which form in areas of stasis.
(Answer: E)
18. Arterial emboli most commonly lodge in which of the following locations?
A) Lower extrem
ities
B) Brain
C) Intestines
D) Kidneys
E) Spleen
Arterial emboli most frequently travel to the lower extremities, causing
ischemia or infarction.
(Answer: A)
19. In which of the following is the presence of Zahn lines most likely?
A) Intra-arterial thrombus
B) Right ventricular thrombus
C) Fresh gastrointestinal bleeding
D) Deep venous thrombosis
E) Intra-abdominal blood clots
Zahn lines are characteristic of thrombi that form in high-flow areas like
arteries or the heart, where alternating layers of platelets and fibrin can form.
(Answer: A)
74 Pathology Questions and Answers, Prof.Dr. Murat ALPER
20. What is the term for tissue necrosis resulting from blocked arterial or
venous flow?
A) Ischemia
B) Embolism
C) Thrombus
D) Infarction
E) Hyperemia
Infarction is the term for tissue necrosis caused by the blockage of blood flow,
typically due to a thrombus or embolus.
(Answer: D)
21. In which organ is a red (hemorrhagic) infarct most commonly seen?
A) Lungs
B) Heart
C) Spleen
D) Kidneys
E) Liver
Red infarcts occur in organs with dual blood supplies, such as the lungs,
where blood can flow into the infarcted tissue from the surrounding vessels.
(Answer: A)
22. Which organ is least likely to develop a hemorrhagic infarct?
A) Spleen
B) Lungs
C) Testes
D) Ovaries
E) Brain
75 Pathology Questions and Answers, Prof.Dr. Murat ALPER
The spleen, being a solid organ with a single blood supply, is more likely to
develop a pale infarct rather than a hemorrhagic one.
(Answer: A)
23. In which organ would arterial occlusion lead to a different macroscopic
appearance compared to the others?
A) Heart
B) Lungs
C) Kidneys
D) Spleen
E) Liver
The lungs are unique in that they can develop hemorrhagic infarcts due to
their dual blood supply, leading to a different appearance compared to infarcts in
solid organs like the heart or kidneys.
(Answer: B)
24. What is the most common cause of fat embolism?
A) Soft tissue trauma
B) Burns
C) Immobilization
D) Long bone fractures
E) Enzymatic fat necrosis
Fat embolism most commonly occurs after long bone fractures, where fat
globules from the bone marrow enter the bloodstream and can lead to embolism.
(Answer: D)
25. A 65-year-old man dies suddenly after developing tachypnea, dyspnea, and
tachycardia on the third day after a femoral head fracture. What is the most likely
finding at autopsy?
A) Squamous cells and hair structures in pulmonary circulation
76 Pathology Questions and Answers, Prof.Dr. Murat ALPER
B) Intracytoplasmic fat accumulation in hepatocytes
C) Liquefactive necrosis in the brain
D) Bone marrow tissue and fat globules in pulmonary circulation
E) Fatty infiltration of the myocardium
The findings are consistent with a fat embolism syndrome, which can occur
after fractures of long bones like the femur. This is confirmed by the presence of bone
marrow elements and fat globules in the pulmonary circulation at autopsy.
(Answer: D)
26. A construction worker suffers multiple fractures of the pelvis and femur
after a fall. On the second day in the hospital, he develops confusion, petechial rash,
severe dyspnea, and hypoxemia. What is the most likely diagnosis?
A) Disseminated intravascular coagulation
B) Fat embolism
C) Endotoxic shock
D) Thrombotic thrombocytopenic purpura
E) Amniotic fluid embolism
The patient's history of fractures and subsequent symptoms are characteristic
of fat embolism syndrome, which can present with confusion, petechiae, and
respiratory distress.
(Answer: B)
27. What causes Caisson disease?
A) Hydrogen
B) Carbon monoxide
C) Carbon dioxide
D) Oxygen
E) Nitrogen
77 Pathology Questions and Answers, Prof.Dr. Murat ALPER
Caisson disease, or decompression sickness, is caused by the formation of
nitrogen gas bubbles in the bloodstream during rapid decompression. These bubbles
can obstruct blood flow and cause ischemia in various tissues.
(Answer: E)
Review Topics:
- Thrombosis
- Venous thrombosis
- Fat embolism
- Decompression sickness
- Transudate
- Exudate
- Plasma colloidal osmotic pressure
- Nephrotic syndrome
- Cirrhosis
- Protein malnutrition
- Edema
- Pulmonary edema
- Hyperemia
- Congestion
- Acute pulmonary congestion
- Chronic pulmonary congestion
- Acute hepatic congestion
- Chronic hepatic congestion
- Hematoma
- Ecchymosis
- Recanalization
- Thrombosis
- Embolization
- Clot dissolution
- Organization
78 Pathology Questions and Answers, Prof.Dr. Murat ALPER
SECTION 5: HISTOCHEMISTRY AND
IMMUNOHISTOCHEMISTRY
1. Which histochemical stain is used to differentiate muscle, connective tissue, and nerve in
tissues?
A) Masson-Trichrome
B) Silver
C) Hematoxylin-Eosin
D) Prussian blue
E) PTAH
Masson-Trichrome is a histological stain that differentiates between muscle, collagen
(connective tissue), and nerve fibers by staining them in different colors.
(Answer: A)
2. Which histochemical method is used to detect iron deposits in tissues?
A) Prussian blue
B) Trichrome
C) Silver staining
D) Congo red
E) Fontana
Prussian blue stain is specifically used to detect ferric iron deposits in tissues, commonly
used to diagnose conditions like hemochromatosis.
(Answer: A)
3. Which histochemical stain is used to identify mast cells?
A) Masson-Trichrome
B) Methylene blue
79 Pathology Questions and Answers, Prof.Dr. Murat ALPER
C) Toluidine blue
D) Alcian blue
E) Periodic acid-Schiff
Toluidine blue is a metachromatic stain that stains mast cells and basophils, highlighting their
granules.
(Answer: C)
4. Which histochemical stain is used to demonstrate fungi and basement membranes in
tissues?
A) Masson-Trichrome
B) Methylene blue
C) Toluidine blue
D) Alcian blue
E) Periodic acid-Schiff (PAS)
PAS stain is used to detect glycogen, mucins, fungi, and basement membranes in tissue
sections, making it a versatile stain for various pathological conditions.
(Answer: E)
5. Which tumor is expected to show birefringence (green birefringence) under polarized light
when stained with Congo red?
A) Papillary thyroid carcinoma
B) Ovarian serous papillary carcinoma
C) Medullary thyroid carcinoma
D) Medullary carcinoma of the breast
E) Lobular carcinoma of the breast
Congo red stain, under polarized light, exhibits apple-green birefringence in tissues
containing amyloid deposits. Medullary thyroid carcinoma is known to have amyloid
deposits.
80 Pathology Questions and Answers, Prof.Dr. Murat ALPER
(Answer: C)
6. Which histochemical stain is used to detect amyloid in tissues?
A) Prussian blue
B) Sudan III
C) Congo red
D) Hematoxylin-Eosin
E) Masson-Trichrome
Congo red is the standard stain used for the detection of amyloid deposits in tissues, which
show characteristic birefringence under polarized light.
(Answer: C)
7. Which histochemical stain is used to differentiate chromophobe renal cell carcinoma from
oncocytoma by staining mucin?
A) Prussian blue
B) Sudan III
C) Congo red
D) Hale's colloidal iron
E) Masson-Trichrome
Hale's colloidal iron stain is used to identify mucin and can help differentiate chromophobe
renal cell carcinoma (positive) from oncocytoma (negative).
(Answer: D)
8. What solution should tissues be sent in for routine pathological examination?
A) 10% alcohol
B) 10% xylene
C) 10% formalin
D) 30% formalin
81 Pathology Questions and Answers, Prof.Dr. Murat ALPER
E) Normal saline
Tissues for routine pathological examination should be fixed in 10% formalin to preserve
cellular details for histopathological analysis.
(Answer: C)
9. How should tissues be sent to the pathology laboratory for frozen section analysis?
A) In 1% glutaraldehyde
B) In 10% glutaraldehyde
C) In 10% formalin
D) In picric acid
E) Unfixed
For frozen section analysis, tissues should be sent unfixed to allow for rapid freezing and
immediate microscopic examination during surgery.
(Answer: E)
10. Which fixative is used for tissue preparation for electron microscopy?
A) 10% formalin
B) Bouin solution
C) Buffered formalin
D) Stieve solution
E) 1% glutaraldehyde
1% glutaraldehyde is the preferred fixative for tissue preparation for electron microscopy due
to its ability to preserve ultrastructural details.
(Answer: E)
11. Which immunohistochemical marker is indicative of carcinoma rather than lymphoma?
A) Cytokeratin
82 Pathology Questions and Answers, Prof.Dr. Murat ALPER
B) Vimentin
C) Desmosome
D) Calcitonin
E) Neurofilament
Cytokeratin is a marker used to identify epithelial cells and is typically positive in carcinomas,
helping to differentiate them from lymphomas.
(Answer: A)
12. Desmin is an intermediate filament that stains which type of tumor cells using
immunohistochemistry?
A) Neuronal tumors
B) Gliomatous tumors
C) Cartilage tumors
D) Muscle tumors
E) Carcinomas
Desmin is a marker for muscle tissue and is used in immunohistochemistry to identify muscle
tumors, including leiomyomas and rhabdomyosarcomas.
(Answer: D)
13. Which of the following is a pan-T cell marker in immunohistochemistry?
A) CD19
B) CD20
C) CD2
D) CD11
E) CD45
CD2 is one of the pan-T cell markers, along with CD3, CD5, and CD7, commonly used to
identify T cells in immunohistochemistry.
(Answer: C)
83 Pathology Questions and Answers, Prof.Dr. Murat ALPER
---
Review Topics:
- Frozen section analysis
- Formalin as a fixative
- Electron microscopy preparation
- PAS (Periodic acid-Schiff) stain
- Congo red stain
- Prussian blue stain
- Masson-Trichrome stain
- Sudan III stain
- Giemsa stain
- Toluidine blue stain
- Immunohistochemistry
- Immunofluorescence
84 Pathology Questions and Answers, Prof.Dr. Murat ALPER
SECTION 6: IMMUNOPATHOLOGY
1. Which of the following is incorrect regarding T lymphocytes?
A) They make up 60% of peripheral lymphocytes.
B) They are located in the paracortical region of lymph nodes.
C) They are found in the periarteriolar region of the spleen.
D) The CD8/CD4 ratio is 2.
E) They are activated by CD28.
Immunity is mediated in two ways: 1- Humoral immunity, which involves
neutralization of extracellular pathogens like bacteria by B lymphocytes, and includes
the actions of soluble antibodies secreted by plasma cells, complement-mediated
lysis, and phagocytosis by neutrophils and macrophages. 2- Cellular immunity, which
involves the processing of intracellular pathogens like viruses by antigen-presenting
cells, the presentation of these antigens to CD4 T-helper cells, and subsequent lysis by
cytotoxic T cells, NK cells, and macrophages. The mediators of cellular immunity, T
lymphocytes, are found in the periarteriolar region of the spleen and the
interfollicular region of lymph nodes. T lymphocytes develop from precursors in the
thymus and constitute 60-70% of peripheral lymphocytes, of which 60% are CD4
helpers and 30% are CD8 suppressors (cytotoxic). Protein-energy malnutrition is the
most common factor limiting lymphocyte production.
During antigen recognition, CD4 molecules bind to Class II MHC molecules on
antigen-presenting cells (APCs), while CD8 binds to Class I MHC. Full activation of T
cells requires two signals: Signal 1 is the appropriate MHC-antigen combination, and
Signal 2 is the stimulation of CD28 on T lymphocytes by CD80 and CD86 on APCs.
Without CD28 stimulation, the cells undergo apoptosis and become anergic. The
costimulation by CD28 is critical in anergy. CD4 (helper) cells secrete cytokines, while
CD8 cells also secrete cytokines but primarily kill virus-infected cells and tumor cells.
B lymphocytes make up 10-20% of circulating lymphocytes and are found in
lymphoid tissues (nodes, spleen, tonsils, gastrointestinal tract, bone marrow,
lymphoid follicles, and white pulp of the spleen). Upon antigenic stimulation, they
differentiate into plasma cells that secrete immunoglobulins IgG, A, M, D, and E. The
interaction of CD40 on T cells with CD40 on B cells is critical for the secretion of IgG,
85 Pathology Questions and Answers, Prof.Dr. Murat ALPER
A, and E. Patients with CD40 mutation secrete IgM only, leading to hyper-IgM
syndrome.
Natural Killer (NK) cells constitute 10-15% of lymphocytes and can lyse tumor
cells, virus-infected cells, and normal cells without prior sensitization. They are CD3
(-), CD16, and CD56 (+). CD16 serves as an Fc receptor for IgG, equipping NK cells with
the ability to lyse IgG-opsonized cells, a process known as antibody-dependent
cellular cytotoxicity (ADCC). Class I MHC molecules on target cells interact with the
Killer Inhibitory Receptors (KIRs) on NK cells, protecting the cells from lysis.
Class I MHC molecules are present on all nucleated cells and interact with CD8
cytotoxic cells, enabling them to detect and lyse virus-infected cells. Class II MHC
molecules interact with CD4 T cells and are found on APCs and B lymphocytes. Class
III MHC molecules include complement components and encode TNF and TNF-Beta
but are not histocompatibility antigens.
Ankylosing spondylitis is associated with HLA B-27, Rheumatoid arthritis with
HLA DR 4, and Diabetes mellitus with HLA DR3 – DR-4.
(Answer: D)
2. Which of the following cells processes antigen and presents it to
immunocompetent T cells?
A) Plasma cell
B) Basophil
C) Natural killer cell
D) Dendritic cells
E) Eosinophil
Dendritic cells are crucial in processing antigens and presenting them to
immunocompetent T lymphocytes, playing an essential role in immunity and wound
healing. Dendritic cells in the epidermis are called Langerhans cells.
(Answer: D)
3. Which of the following is most protective against viruses?
86 Pathology Questions and Answers, Prof.Dr. Murat ALPER
A) Interleukin 1
B) Interleukin 8
C) Interferon
D) Tumor necrosis factor
E) Colony-stimulating factor
Among cytokines, interferon is highly effective against viral infections, whereas
interleukin 1, 8, and TNF primarily initiate nonspecific inflammatory responses.
(Answer: C)
4. Which of the following is most effective against tumor cells?
A) B lymphocyte
B) T lymphocyte
C) Macrophage
D) Toll-like receptors
E) Natural Killer
NK cells, comprising 10-15% of peripheral blood lymphocytes, lack T cell
receptors or surface immunoglobulins. They may be larger than normal lymphocytes
and contain azurophilic granules, giving them the name large granular lymphocytes.
NK cells are negative for T cell receptors (TCR) and CD3 but positive for CD16 and
CD56. CD16 is found on all NK cells and granulocytes and serves as an Fc receptor for
IgG, allowing NK cells to mediate antibody-dependent cellular cytotoxicity (ADCC). NK
cells can kill tumor cells, virus-infected cells, and some normal cells without prior
sensitization. Toll-like receptors (TLR) recognize microbial molecules; plasma
membrane TLRs recognize bacterial products like lipopolysaccharides, while
endosomal TLRs recognize bacterial and viral RNA and DNA.
(Answer: E)
5. Which of the following is associated with KIR (Killer Inhibitory Receptor)?
A) Plasma cell
B) Colony-stimulating factor
C) Natural killer cell
87 Pathology Questions and Answers, Prof.Dr. Murat ALPER
D) Macrophage
E) Interleukin
(Answer: C)
6. Inhibiting which of the following would reduce transplant rejection?
A) CD2
B) CD5
C) CD20
D) CD28
E) CD56
Autoimmunity: Represents a loss of self-tolerance. Self-tolerance is the
immune system's ability to recognize and not respond to the body's own antigens.
Tolerance is divided into central and peripheral tolerance. Central tolerance
involves the deletion of self-reactive T and B cells during their maturation in central
lymphoid organs (T → thymus, B → bone marrow) through apoptosis.
Peripheral tolerance: Self-reactive cells that escape central tolerance are
managed peripherally through:
1) Anergy (absence of CD28-B7 interaction)
2) Activation-induced cell death via Fas-Fas ligand-mediated apoptosis
3) Peripheral suppression by T cells.
Autoimmune diseases arise from the breakdown of these mechanisms. The
breakdown of tolerance can result from immunologic, genetic, and infectious factors.
Breakdown of tolerance involves:
1) Loss of anergy
2) Failure of activation-induced cell death
3) Defective peripheral suppression by T cells
88 Pathology Questions and Answers, Prof.Dr. Murat ALPER
4) Molecular mimicry
5) Release of sequestered antigens “uveitis and orchitis”
Preventing the interaction between CD28 receptors on host T cells and donor
dendritic cells halts the second signal required for T cell activation, leading to
apoptosis or T cell anergy.
(Answer: D)
7. Which immunoglobulin has the greatest capacity to bind antigens?
A) Ig A
B) Ig D
C) Ig E
D) Ig G
E) Ig M
(Answer: E)
8. Which immunoglobulin is found in mucous membranes and helps prevent
the entry of infectious agents?
A) Ig A
B) Ig E
C) Ig G
D) Ig G
E) Ig M
(Answer: A)
9. Which immunoglobulin is normally found in the least quantity in serum?
A) Ig A
B) Ig D
C) Ig E
89 Pathology Questions and Answers, Prof.Dr. Murat ALPER
D) Ig G
E) Ig M
IgG is the most abundant immunoglobulin in serum, with a concentration of
1200 mg/100 ml, constituting 75% of gamma globulins. IgG crosses the placenta and
protects the newborn until the immune system matures. Ig A is 250 mg/100 ml, Ig M
is 120 mg/100 ml, Ig D is 3 mg/100 ml, while Ig E is the least abundant, at 0.03
mg/100 ml. Ig E levels increase in parasitic infections and allergic reactions. Ig A is
found in mucosal barriers and prevents the entry of infectious agents. Ig M does not
cross the placenta and has the highest antigen-binding capacity. Both IgG and IgM
activate the classical complement pathway, aiding in phagocytosis and the elimination
of microbes.
(Answer: C)
10. Which cells contain Birbeck granules?
A) Macrophages
B) Eosinophils
C) Natural killer cells
D) CD8 + T cells
E) Skin Langerhans cells
Skin Langerhans cells and lymphoid tissue dendritic cells are antigen-
presenting cells similar to macrophages. Skin Langerhans cells contain racket-shaped
Birbeck granules. Langerhans cells and their tumors stain positively with CD1a
antigen. Natural killer cells directly kill tumor cells, fungi, and viruses through
cytotoxic effects.
(Answer: E)
11. Which chromosome is associated with histocompatibility antigens?
A) 3rd chromosome
B) 4th chromosome
C) 5th chromosome
D) 6th chromosome
90 Pathology Questions and Answers, Prof.Dr. Murat ALPER
E) 10th chromosome
The HLA system plays a role in both humoral and
cellular immunity. Its major physiological function is to present antigens to T
lymphocytes. The genes related to HLA are located on the 6th chromosome. In organ
transplantation, rejection occurs if the HLA system is not compatible.
Class I antigens are present on all nucleated cells and platelets and are
polymorphic heavy chain glycoproteins. HLA-A, HLA-B, HLA-C, and beta2-
microglobulin are Class I. Class I MHC molecules bind to KIRs (Killer Inhibitory
Receptors) on NK cells to protect normal cells from NK cell-mediated lysis. When this
protection is lost, autoimmune diseases can develop via CD8 T lymphocytes.
Class II is encoded by genes in the HLA-D region and includes DR, DQ, and DP,
found on antigen-presenting cells (dendritic cells, macrophages), B cells, and
activated T cells.
Class III includes complement components like C2, C4, and Bf, as well as TNF-
alpha and TNF-beta, and is not related to tissue compatibility.
(Answer: D)
12. In which of the following conditions is HLA B-27 positivity most commonly
seen?
A) Rheumatoid arthritis
B) Ankylosing spondylitis
C) Hemochromatosis
D) Diabetes mellitus
E) 21-hydroxylase deficiency
(Answer: B)
13. Which is most frequently positive in rheumatoid arthritis?
A) HLA-B-27
91 Pathology Questions and Answers, Prof.Dr. Murat ALPER
B) HLA-DR-4
C) HLA DR 3
D) HLA A3
E) HLA BW 47
Ankylosing spondylitis, post-gonococcal arthritis, and acute anterior uveitis are
associated with HLA B 27, Rheumatoid arthritis with HLA DR 4, active hepatitis, and
Sjögren’s syndrome with HLA DR 3, and 21-Hydroxylase deficiency with HLA BW 47.
Celiac disease is associated with HLA B-8, juvenile diabetes with HLA B8, DR-3, and
B3. Hemochromatosis is associated with HLA-A3, while Addison's disease is
associated with HLA-A1 and B8.
(Answer: B)
14. Mast cells and basophils are involved in which type of reaction?
A) Anaphylaxis
B) Cytotoxic type
C) Immune complex type
D) Delayed-type hypersensitivity
E) Antibody-dependent cellular cytotoxicity
Type I anaphylactic reactions occur when allergens combine with IgE, leading
to the activation of basophils and mast cells and the release of mediators like
histamine. Conditions like anaphylaxis, skin allergies, food allergies, hay fever, certain
types of asthma, and atopy fall under this category.
(Answer: A)
15. Which of the following is significantly involved in the development of
anaphylaxis?
A) Tuberculin reaction
B) Glomerulonephritis
C) Anaphylaxis
D) Transplant rejection
92 Pathology Questions and Answers, Prof.Dr. Murat ALPER
E) Erythroblastosis fetalis
(Answer: C)
16. Goodpasture’s disease is associated with which type of reaction?
A) Immune complex
B) Complement and Fc-mediated inflammation
C) ADCC
D) Delayed-type hypersensitivity
E) Antibody-mediated cellular dysfunction
(Answer: B)
17. Which of the following is an example of a type II hypersensitivity reaction
involving antibody-mediated cellular dysfunction?
A) Erythroblastosis fetalis
B) Autoimmune hemolytic anemia
C) Goodpasture syndrome
D) Transfusion reaction
E) Myasthenia gravis
(Answer: E)
18. Which of the following is not a complement-dependent type II
hypersensitivity reaction?
A) Erythroblastosis fetalis
B) Autoimmune hemolytic anemia
C) Goodpasture syndrome
D) Transfusion reaction
E) Myasthenia gravis
(Answer: E)
93 Pathology Questions and Answers, Prof.Dr. Murat ALPER
19. In which condition are autoantibodies against acetylcholine receptors
observed?
A) Myasthenia gravis
B) Dermatomyositis
C) Scleroderma
D) Pernicious anemia
E) Rheumatoid arthritis
In type II antibody-dependent reactions, IgG, IgM, and complement activation
lead to target cell lysis. There are three subtypes of type II reactions: a) opsonization
and complement and Fc receptor-mediated phagocytosis-dependent (e.g.,
autoimmune hemolytic anemia, ITP), b) complement and Fc receptor-mediated
inflammation (e.g., Goodpasture syndrome, vascular rejection, transfusion reactions,
and certain drug reactions), and antibody-mediated cellular dysfunction (e.g.,
myasthenia gravis, pemphigus vulgaris, and Graves’ disease). Pernicious anemia and
acute rheumatic fever are also examples of type II hypersensitivity reactions.
(Answer: A)
20. Systemic Lupus Erythematosus is associated with which type of reaction?
A) Antibody-dependent cellular cytotoxicity
B) Anaphylaxis
C) Antibody-mediated cellular dysfunction
D) Immune complex type
E) Delayed-type hypersensitivity
(Answer: D)
21. In which of the following diseases is the vasculitis observed not due to
immune complex formation?
A) Serum sickness
B) Wegener's granulomatosis
C) Systemic lupus erythematosus
94 Pathology Questions and Answers, Prof.Dr. Murat ALPER
D) Chronic hepatitis B
E) Henoch-Schönlein purpura
(Answer: B)
22. Which of the following does not involve immune complex (Type III)
formation?
A) Arthus reaction
B) Serum sickness
C) SLE
D) Post-streptococcal glomerulonephritis
E) Autoimmune hemolytic anemia
Type III immune complex disorders arise from antigen-antibody complex
formation and complement activation, leading to neutrophil recruitment and the
release of lysosomal and other toxic substances. Examples include Arthus reaction,
serum sickness, systemic lupus erythematosus, polyarteritis nodosa, and post-
streptococcal glomerulonephritis. The pathological lesion is necrotizing vasculitis,
characterized by inflammation and fibrinoid necrosis. Autoimmune hemolytic anemia
is a type II hypersensitivity reaction involving opsonization and phagocytosis of
erythrocytes.
(Answer: E)
23. A patient with chronic renal failure undergoing hemodialysis receives a
cadaveric kidney transplant. Following vascular anastomosis, the graft appears
cyanotic, mottled, and soft. The kidney is removed, and histopathological examination
reveals vasculitis in the walls of arteries and arterioles. What is the most likely cause
of this condition?
A) Cellular acute rejection
B) Humoral acute rejection
C) Recurrence of primary disease
D) Chronic rejection
E) Hyperacute rejection
95 Pathology Questions and Answers, Prof.Dr. Murat ALPER
(Answer: E)
24. Contact dermatitis is associated with which mechanism?
A) Anaphylaxis
B) Opsonization and complement-mediated phagocytosis
C) Immune complex type
D) Delayed-type hypersensitivity
E) Antibody-dependent cellular cytotoxicity
Type IV delayed-type hypersensitivity involves CD4+ T lymphocyte
sensitization. Examples include the tuberculin test, granuloma formation, contact
dermatitis, and transplant rejection. A subtype of Type IV involves direct cytotoxicity
by T lymphocytes through perforin, granzyme, and Fas-Ligand. Granuloma formation
is initiated by macrophages secreting IL-12, which stimulates interferon-gamma
secretion from CD4 lymphocytes and NK cells, transforming macrophages into
epithelioid histiocytes. Transplant rejection: Hyperacute rejection occurs due to
preformed antibodies against antigens on graft endothelial cells. While cross-match
testing is routinely performed for all donors and recipients, hyperacute rejection is
rare, but the affected kidney becomes cyanotic and anuric quickly, with fibrinoid
necrosis in arterioles. The affected kidney is non-functional and is removed. Acute
rejection occurs due to T cell and antibody activation by alloantigens in the graft. It
has two subtypes: acute cellular and acute humoral rejection. Recognizing cellular
rejection is essential, especially in cases not accompanied by humoral rejection, as it
responds well to immunosuppression.
(Answer: D)
25. Which of the following is most commonly associated with T-cell-mediated
hypersensitivity?
A) Contact dermatitis
B) Graves’ disease
C) Polyarteritis nodosa
D) Anti-glomerular basement membrane disease
E) Hemolytic disease of the newborn
96 Pathology Questions and Answers, Prof.Dr. Murat ALPER
Contact dermatitis is caused by T-cell-mediated (Type IV) hypersensitivity or
direct chemical damage to the skin. Conditions like Graves’ disease, hemolytic disease
of the newborn, polyarteritis nodosa, and anti-glomerular basement membrane
disease are antibody-mediated (Types I, II, and III) hypersensitivity reactions.
(Answer: A)
26. Which of the following is not an example of T-cell-mediated (Type IV)
hypersensitivity?
A) Type I diabetes mellitus
B) Multiple sclerosis
C) Rheumatoid arthritis
D) Peripheral neuropathy
E) Myasthenia gravis
Type I diabetes mellitus involves a T-cell response against pancreatic islet cells.
Similarly, multiple sclerosis involves a T-cell response against myelin, rheumatoid
arthritis involves a T-cell response against joint surfaces, and peripheral neuropathy
(e.g., Guillain-Barré syndrome?) involves a T-cell response against peripheral nerve
myelin. Myasthenia gravis is a Type II hypersensitivity reaction.
(Answer: E)
27. Which immunoglobulin is most effective against parasites?
A) Ig A
B) Ig D
C) Ig E
D) Ig G
E) Ig M
Ig E and ADCC are effective against parasites.
(Answer: C)
97 Pathology Questions and Answers, Prof.Dr. Murat ALPER
28. Which of the following pairs of diseases fall under the same
hypersensitivity reaction classification?
A) Allergic rhinitis and poison ivy
B) ABO incompatibility and bee sting
C) Hyperacute transplant rejection and serum sickness
D) Nickel contact dermatitis and atopic dermatitis
E) Graves' disease and Goodpasture syndrome
Graves' disease and Goodpasture syndrome are examples of Type II
hypersensitivity. The former involves thyroid-stimulating antibodies against TSH
receptors, while the latter involves autoantibodies against the glomerular and
pulmonary capillary basement membranes. Allergic rhinitis, atopic dermatitis, and
bee stings cause Type I hypersensitivity reactions, primarily mediated by histamine.
Poison ivy and nickel dermatitis are examples of Type IV hypersensitivity (contact
dermatitis). ABO incompatibility and hyperacute transplant rejection are cytotoxic
antibody-mediated reactions (Type II hypersensitivity), whereas serum sickness is an
immune complex-mediated Type III hypersensitivity reaction.
(Answer: E)
29. Which of the following diseases involves complement activation and
neutrophil-mediated tissue damage due to chemotactic factors?
A) Myasthenia gravis
B) Tuberculous granuloma
C) Post-streptococcal glomerulonephritis
D) Destruction of invasive helminths
E) Acute rejection of a kidney transplant
Post-streptococcal glomerulonephritis is an immune complex-mediated
disease (Type III hypersensitivity). Its pathogenesis involves complement activation
and the production of C5a, a chemotactic agent that attracts neutrophils to the site of
immune complex deposition. Tuberculous granuloma is a delayed-type
hypersensitivity reaction (Type IV) and is not related to complement or neutrophils.
Myasthenia gravis involves autoantibodies against acetylcholine receptors (Type II
hypersensitivity). The destruction of invasive helminths is mediated by a variant of
98 Pathology Questions and Answers, Prof.Dr. Murat ALPER
antibody-dependent cellular immunity, involving Ig E. The rejection of a kidney
transplant is primarily a Type IV hypersensitivity reaction involving cytotoxic CD8 and
helper CD4 T cells.
(Answer: C)
30. Which of the following is least involved in the acute rejection of a kidney
transplant from a living donor?
A) Donor CD4 T helper cells
B) Host CD8 cytotoxic T cells
C) Host NK cells
D) Donor macrophages
E) Interleukin 1 and 2
In the acute rejection of a kidney transplant from a living donor, donor
macrophages interact with CD8 cytotoxic T cells and CD4 T helper cells. IL-2, secreted
by CD4 T cells, stimulates antibody production and the proliferation of additional CD8
cytotoxic T cells. Host macrophages and NK cells contribute to graft destruction. IL-1
is secreted by both donor and host macrophages. Cytotoxic T cells also contain
granzymes and perforin. Immune-privileged tissues, such as the cornea, lens, and
CNS, are usually protected from immune rejection. In rejection, humoral rejection
involves vasculitis (arteriolitis), and cellular rejection involves interstitial lymphocyte
infiltration and edema.
(Answer: A)
31. A patient with aplastic anemia who recently underwent a bone marrow
transplant develops widespread maculopapular rash, jaundice, and diarrhea. Which
of the following is the most likely pathogenic mechanism?
A) Tissue damage by lymphokine-activated NK cells
B) Tissue damage by CD8 cytotoxic T cells
C) Vascular thrombosis secondary to anti-HLA antibody damage
D) Interaction between CD4 T helper cells and macrophages
E) Complement system activation following immune complex deposition.
99 Pathology Questions and Answers, Prof.Dr. Murat ALPER
In hyperacute rejection, the process begins with an antigen-antibody reaction
on vascular endothelium. Damage caused by anti-HLA antibodies leads to vascular
thrombosis, responsible for hyperacute transplant rejection. However, it is essential
to understand that different cell types and reaction types often overlap in rejection.
Patients who undergo bone marrow transplantation are at risk for Graft-versus-host
(GVH) disease. IL-2 release by immunocompetent donor T helper cells stimulates NK
cells, which are responsible for the maculopapular rash in the skin, epithelial necrosis
in the bile ducts (jaundice), and epithelial necrosis in the intestines (diarrhea). In
chronic GVHD, skin lesions resemble systemic sclerosis. Tissue damage mediated by
CD8 cytotoxic T cells occurs in acute transplant rejection. To reduce the risk of GVH
disease, careful HLA matching between donor and recipient should be performed
using DNA sequencing methods.
(Answer: A)
32. Which of the following features is not observed in SLE?
A) Wire-loop lesion
B) Onion skin appearance
C) Libman-Sacks endocarditis
D) Granular IgA deposition in unaffected skin
E) Type I reaction
(Answer: E)
33. In Systemic Lupus Erythematosus, which organ is most frequently affected
clinically and pathologically?
A) Eye
B) Kidney
C) Skin
D) Hematologic conditions
E) Serous membranes
(Answer: D)
34. In SLE, which organ’s involvement most significantly affects morbidity and
mortality?
100 Pathology Questions and Answers, Prof.Dr. Murat ALPER
A) Heart
B) Liver
C) Kidney
D) Central Nervous System
E) Joint
(Answer: C)
35. Non-bacterial verrucous endocarditis is most commonly associated with
which of the following?
A) Infective endocarditis
B) SLE
C) EBV infection
D) CMV infection
E) Chagas disease
(Answer: B)
36. Libman-Sacks endocarditis is seen in which of the following conditions?
A) Rheumatic fever
B) Multiple myeloma
C) SLE
D) Amyloidosis
E) Chronic kidney disease
(Answer: C)
37. Which of the following is diagnostic of Lupus?
A) Positive lupus cell test
B) Anti-nuclear antibody
C) TPI
D) Double-stranded DNA antibody
E) Anti-neutrophil cytoplasmic antibody
101 Pathology Questions and Answers, Prof.Dr. Murat ALPER
SLE, a multisystem disease caused by a Type III hypersensitivity reaction,
predominantly affects young women. The central issue is a defect in maintaining self-
tolerance. Symptoms include butterfly-shaped rash, photosensitivity, arthritis,
pericarditis, kidney involvement, and neurological manifestations. Clinically and
pathologically, hematologic findings are most common, followed by joint
involvement. The kidneys are clinically affected in 50% of cases, but nearly 100% of
cases show kidney involvement on detailed microscopic examination. Certain drugs
like procainamide, D-penicillamine, and hydralazine can cause similar clinical
symptoms. Estrogen worsens SLE, while androgens may have a protective effect. B
cell hyperactivity and CD4 T lymphocytes play a significant role in pathogenesis. In
SLE, tissue damage occurs primarily through Type III hypersensitivity reactions, while
most hematologic damage is due to Type II reactions. Lupus cells (hematoxylin
bodies) are present in 70% of cases. ANA is positive in a high percentage but is not
specific. Anti-double-stranded DNA and Anti-Sm antibodies are diagnostic. The
deposition of granular immunoglobulins in unaffected skin is crucial for differential
diagnosis from other skin diseases. Non-bacterial verrucous endocarditis (Libman-
Sacks endocarditis) and glomerulonephritis are common. Long-standing disease,
especially in young patients on corticosteroid therapy, is often associated with
coronary artery disease. The wire-loop appearance of glomeruli is characteristic.
Onion-skinning is seen in the spleen, in central and penicillary arteries.
Neuropsychiatric symptoms may occur. The distinguishing feature of joint
involvement is non-deforming arthritis (non-erosive synovitis). The leading causes of
death are kidney failure and infections.
In Class I, there are no significant light microscopic changes. Mesangial Lupus
GN (Class II) is the mildest form of lupus nephritis. Class III is characterized by focal
proliferative glomerulonephritis. Class IV (diffuse proliferative GN) is the most severe
and most common form of kidney involvement in SLE. Membranous
glomerulonephritis (Class V) is seen in 10-15% of cases. The wire-loop lesion,
indicating a poor prognosis, is most common in Class IV but can occasionally be seen
in Classes III and V as well.
(Answer: D)
38. Felty syndrome includes which of the following?
A) Polyarthritis, leukocytosis, splenomegaly
B) Polyarthritis, leukopenia, hepatomegaly
C) Polyarthritis, splenomegaly, leukopenia
D) Polyarthritis, leukocytosis, hepatomegaly
102 Pathology Questions and Answers, Prof.Dr. Murat ALPER
E) Lymphadenomegaly, leukocytosis, monoarthritis
Felty syndrome is characterized by a triad of polyarthritis, splenomegaly, and
leukopenia.
(Answer: C)
39. Which of the following is incorrect regarding rheumatoid arthritis?
A) Ig M rheumatoid factor positivity
B) Symmetric polyarthropathy
C) HLA DR 4 positivity
D) Predominantly affects distal interphalangeal joints
E) Subcutaneous nodules
(Answer: D)
40. Rheumatoid factors are which type of immunoglobulin?
A) Ig A
B) Ig G
C) Ig M
D) Ig D
E) Ig E
(Answer: C)
41. Which of the following is incorrect regarding rheumatoid arthritis?
A) Morning joint stiffness
B) Inflammatory cells in syn
ovial fluid
C) Reactive amyloidosis
D) Raynaud's phenomenon
103 Pathology Questions and Answers, Prof.Dr. Murat ALPER
E) Mucosal erosions in the gastrointestinal tract
Rheumatoid arthritis is a sterile, low-viscosity synovitis with neutrophils that
causes joint destruction, typically seen in the 3rd-4th decade and is 3-5 times more
common in women. It typically involves symmetric arthritis of small joints, with
autoimmunity against Type II collagen. 80% of patients with RA have autoantibodies
against the Fc portion of autologous Ig G, most of which are of the IgM type. Proximal
interphalangeal and metacarpophalangeal joints are typically involved, while distal
interphalangeal joints are usually spared. The classic appearance involves
inflammatory cells, granulation tissue, connective tissue, and proliferative synovial
cells forming a pannus. The pannus fills the joint space, leading to fibrosis,
calcification, and permanent ankylosis, ultimately resulting in deformities like swan-
neck deformity and boutonniere deformity. Subcutaneous rheumatoid nodules, the
most common skin lesion, are found on the extensor surface of the forearm and can
affect the lungs, spleen, pericardium, and heart valves. Interstitial fibrosis in the lungs,
uveitis, and keratoconjunctivitis are potential ocular manifestations. Fibrinous
pleuritis, pericarditis, Raynaud’s phenomenon, chronic leg ulcers, gastrointestinal
mucosal ulcers, and infarctions in the brain, heart, and intestines are possible
complications. Reactive amyloidosis (AA type) occurs in 5-10% of cases.
(Answer: B)
42. Which of the following is incorrect regarding Still's disease?
A) Leukopenia
B) Juvenile idiopathic arthritis
C) Rheumatoid factor is usually negative.
D) Affects knees, hands, and ankles
E) Monoarthritis is present in 1/3 of cases.
Juvenile idiopathic arthritis (Still’s disease) is a polyarthritis (1/3 of cases
present with monoarthritis) affecting larger joints than RA. Leukocytosis (15-20k),
hepatosplenomegaly, and rash are common. Rheumatoid factor and subcutaneous
nodules are rare.
(Answer: A)
43. Which of the following is not a feature of Reiter's syndrome?
A) Sacroiliitis
104 Pathology Questions and Answers, Prof.Dr. Murat ALPER
B) Urethritis
C) Uveitis
D) Conjunctivitis
E) Rheumatoid factor positivity
(Answer: E)
44. Which of the following is incorrect regarding seronegative
spondyloarthropathies?
A) Frequent involvement of the sacroiliac joint
B) HLA B-27 positivity
C) Rheumatoid factor negativity
D) Involvement of ligament-bone attachment sites
E) Affects only joints.
(Answer: E)
45. Which of the following is not part of the seronegative
spondyloarthropathies group?
A) Scleroderma
B) Reiter’s syndrome
C) Ankylosing spondylitis
D) Psoriatic arthropathy
E) Spondylitis associated with inflammatory bowel disease
Reiter’s syndrome, ankylosing spondylitis, psoriatic arthropathy, spondylitis
associated with inflammatory bowel disease, and reactive arthropathy fall under the
group of seronegative spondyloarthropathies. Characterized by HLA B-27 positivity,
rheumatoid factor negativity, frequent sacroiliac joint involvement, and ligamentous
attachment site involvement, this group also exhibits extra-articular features such as
urethritis, uveitis, and conjunctivitis. Nongonococcal cervicitis or urethritis, uveitis,
and conjunctivitis are especially seen in Reiter’s syndrome and reactive arthritis.
Psoriatic arthropathy occurs in 5% of psoriasis patients, typically affecting the small
joints of the hands and feet. Enteropathic arthritis occurs in 10-20% of inflammatory
bowel disease patients, presenting as migratory oligoarthritis in large joints and the
105 Pathology Questions and Answers, Prof.Dr. Murat ALPER
spine. Scleroderma is a collagen vascular disease characterized by excessive fibroblast
proliferation, not a seronegative spondyloarthropathy.
(Answer: A)
46. A 40-year-old woman diagnosed with systemic lupus erythematosus four
years ago presents with complaints of dry mouth. Laboratory tests reveal the
presence of rheumatoid factor and antinuclear antibodies against ribonucleoprotein
SS-A(Ro). Biopsy of minor salivary glands shows periductal lymphocytic infiltration
and ductal epithelial hyperplasia. What is the most likely diagnosis for this patient?
A) Ehlers-Danlos syndrome
B) Sarcoidosis
C) Sjögren’s syndrome
D) Lichen planus
E) Reiter’s syndrome
(Answer: C)
47. Which of the following is not a characteristic feature of Sjögren’s
syndrome?
A) Dry mouth
B) Dry eyes
C) Lymphoplasmacytic infiltration of the parotid gland
D) Hypogammaglobulinemia
E) Association with connective tissue diseases
Sjögren’s syndrome is characterized by dry mouth (xerostomia), dry eyes
(xerophthalmia), and is commonly associated with connective tissue diseases like
rheumatoid arthritis. Lymphoplasmacytic infiltration of the parotid gland causes
parotid gland enlargement, and polyclonal hypergammaglobulinemia is seen.
Sjögren’s syndrome also carries a significantly increased risk of pseudolymphoma and
B-cell malignant lymphoma. Conditions with excessive antigenic stimulation, like
Hashimoto’s thyroiditis and H. pylori infection, also carry an increased risk of MALT
lymphoma.
(Answer: D)
106 Pathology Questions and Answers, Prof.Dr. Murat ALPER
48. In which of the following immune system diseases is the presence of
periductal T lymphocytes in a minor salivary gland biopsy from the lip important for
diagnosis?
A) Rheumatoid arthritis
B) Systemic lupus erythematosus
C) Inflammatory myopathy
D) Sjögren’s syndrome
E) Scleroderma
(Answer: D)
49. Which of the following conditions is most commonly associated with
Sjögren’s syndrome?
A) Reiter's syndrome
B) SLE
C) Ankylosing spondylitis
D) Rheumatoid arthritis
E) Scleroderma
(Answer: D)
50. In Sjögren’s syndrome, the presence of which of the following is associated
with an increased risk of extraglandular involvement?
A) Rheumatoid factor
B) Lupus cell
C) Anti-SS-A
D) Hypergammaglobulinemia
E) There is no extraglandular involvement.
(Answer: C)
51. Which of the following is true regarding Sjögren’s syndrome?
A) Leukemia may develop
107 Pathology Questions and Answers, Prof.Dr. Murat ALPER
B) Hodgkin lymphoma may develop.
C) Mucoepidermoid carcinoma may develop
D) Non-Hodgkin lymphoma may develop
E) Malignancy does not develop
Sjögren’s syndrome is an autoimmune disorder characterized by immune-
mediated destruction of the lacrimal and salivary glands, leading to dry eyes
(keratoconjunctivitis sicca) and dry mouth (xerostomia). The disease can also affect
the nasopharynx, upper respiratory tract, and even the vagina. Lip biopsy is required
to examine minor salivary glands for Sjögren’s syndrome diagnosis. The earliest
finding is periductal and perivascular T lymphocyte infiltration in the salivary gland. It
predominantly affects women aged 50-60 and is associated with HLA DR3. It is most
commonly associated with rheumatoid arthritis among other autoimmune diseases.
CD4+ T cell activation plays a significant role in pathogenesis.
Hypergammaglobulinemia is always present, with 75% of patients testing positive for
rheumatoid factor, 50-80% positive for ANA, and 25% positive for lupus cells. Anti-SS-
A and anti-SS-B antibodies are present, though these antibodies are also found in
small amounts in SLE. The presence of anti-SS-A antibodies is associated with an
earlier onset of the disease. Extraglandular manifestations affecting the central
nervous system, kidneys (tubulointerstitial nephritis), and muscles are more common
in those with anti-SS-A antibodies. Non-Hodgkin B-cell lymphoma (Marginal zone
lymphoma = MALT lymphoma) and pseudolymphomatous proliferation are 40 times
more frequent in Sjögren’s syndrome.
(Answer: D)
52. Which of the following diseases is characterized by diffuse fibrosis of the
skin and internal organs, along with symptoms such as calcinosis, mask-like facies,
and digital autoamputation?
A) Wegener's granulomatosis
B) Rheumatoid arthritis
C) Systemic sclerosis
D) Dermatomyositis
E) Sjögren’s syndrome
(Answer: C)
108 Pathology Questions and Answers, Prof.Dr. Murat ALPER
53. Which organ is most commonly affected by systemic sclerosis in the
gastrointestinal tract?
A) Esophagus
B) Stomach
C) Duodenum
D) Colon
E) Rectum
(Answer: A)
54. Which of the following is not seen in limited scleroderma?
A) Calcinosis
B) Dysphagia
C) Raynaud's phenomenon
D) Telangiectasia
E) Sacroiliitis
Systemic sclerosis (scleroderma) most commonly affects individuals in the 5th-
6th decade of life, with a 3:1 female predominance. There is increased collagen
biosynthesis, with the skin involved in 95% of cases. Collagen accumulation leads to
the loss of
skin appendages and thinning of the epidermis. It also affects the
gastrointestinal tract, kidneys, lungs, heart, and skeletal muscles. Skin changes
(fibrosis) progress from distal to proximal areas. Subcutaneous calcifications, vascular
occlusion leading to digital autoamputation, and mask-like facies may occur. The
gastrointestinal tract is affected in more than half of patients, with the esophagus
being the most frequently involved organ, leading to fibrosis and dysphagia. Lung
involvement results in a honeycomb appearance, pulmonary changes lead to right
heart hypertrophy, and Raynaud's phenomenon is almost universally present. Limited
scleroderma (morphea) or CREST syndrome is characterized by (C: calcinosis, R:
Raynaud’s phenomenon, E: esophageal dysmotility, S: sclerodactyly, T: telangiectasia).
Renal failure accounts for 50% of deaths in systemic sclerosis.
(Answer: E)
109 Pathology Questions and Answers, Prof.Dr. Murat ALPER
55. Which of the following is most specific for systemic sclerosis?
A) Anti-DNA topoisomerase I
B) Hypergammaglobulinemia
C) Antinuclear antibody
D) Rheumatoid factor
E) Anticentromere antibody
Hypergammaglobulinemia is present in 50% of systemic sclerosis cases,
antinuclear antibodies are present in 70-90%, and rheumatoid factor is present in
25%, but these findings are nonspecific. Anti-DNA topoisomerase I (Anti-Scl 70) is
seen in 70% of cases and is highly specific. The presence of this antibody should raise
concern for the development of pulmonary fibrosis and vascular pathology. The
anticentromere antibody is also specific but is seen in 90% of patients with limited
scleroderma (e.g., CREST syndrome) and indicates a relatively better prognosis.
(Answer: A)
56. In which of the following diseases can malabsorption be the most
significant clinical feature?
A) Granulomatosis with polyangiitis
B) Systemic Lupus Erythematosus
C) Rheumatoid arthritis
D) Progressive systemic sclerosis
E) Polymyositis
Progressive systemic sclerosis causes diffuse sclerotic atrophy of the skin,
beginning in the fingers and spreading proximally to the upper extremities. Intimal
fibrosis is seen in blood vessels. Progressive fibrosis in the muscular layer affects the
gastrointestinal tract, with the esophagus being most severely affected, leading to
villus atrophy in the small intestine. The kidneys are most commonly affected, causing
hypertension. The lungs exhibit diffuse interstitial and alveolar fibrosis, and the heart
shows myocardial fibrosis.
(Answer: D)
110 Pathology Questions and Answers, Prof.Dr. Murat ALPER
57. Which of the following is not a common feature of mixed connective tissue
disease compared to other diseases?
A) Arthralgia
B) Raynaud's phenomenon
C) Renal failure
D) Myositis
E) Esophageal hypomotility
Mixed connective tissue disease (MCTD) combines features of systemic lupus
erythematosus, scleroderma, and polymyositis, but renal involvement is not a
common feature of MCTD, and the condition generally responds well to steroids. High
levels of antibodies against ribonucleoprotein particles (U1RNP) are characteristic of
MCTD.
(Answer: C)
58. Which disease is most commonly associated with malignancy?
A) SLE
B) Scleroderma
C) Dermatomyositis
D) Polyarteritis nodosa
E) Granulomatosis with polyangiitis
Dermatomyositis, a form of inflammatory myopathy, affects skeletal muscles
and skin. It is the most common inflammatory myopathy in children, while inclusion
body myositis is the most common in those over 60. Morphologically, “rimmed
vacuoles” are observed. Muscle biopsy shows perifascicular atrophy and
inflammation, with capillaries as the primary target. Dermatomyositis presents with
heliotrope (purplish) discoloration and periorbital edema on the upper eyelid,
Gottron’s lesions on the knees, muscle weakness (primarily affecting the proximal
muscles), soft tissue calcifications, and gastrointestinal ulcerations and dysphagia.
Visceral cancers (lung, ovary, stomach) are more common in patients with
dermatomyositis. Serum creatine kinase levels are elevated in inflammatory
myopathies. Anti-Jo-1 antibodies against t-RNA synthetase are specific for this group
of disorders. Inclusion body myositis begins distally, unlike the other two
111 Pathology Questions and Answers, Prof.Dr. Murat ALPER
inflammatory myopathies. Polymyositis resembles dermatomyositis but lacks skin
involvement.
(Answer: C)
59. Which disease is characterized by purple skin lesions on the eyelids,
periorbital edema, and proximal muscle weakness, and may be associated with
internal organ cancers?
A) Dermatitis herpetiformis
B) Actinic keratosis
C) Seborrheic dermatitis
D) Dermatofibroma
E) Dermatomyositis
(Answer: E)
60. Which of the following is incorrect regarding dermatomyositis?
A) Primarily affects distal muscles
B) 20% of patients may have SLE, scleroderma, etc.
C) Antibodies against Jo-1 antigen, a subunit of histidyl-tRNA synthetase, are
present.
D) Gottron's lesion and heliotrope discoloration are present.
E) Elevated creatine kinase and aldolase levels are observed.
Dermatomyositis begins proximally, affecting the shoulder and pelvis, and
then progresses distally. In addition to the correct options, research suggests a 6-45%
risk of malignancy (lung, ovary, stomach) in patients with dermatomyositis. Muscle
biopsy reveals peri-fascicular atrophy, a hallmark feature.
(Answer: A)
61. Which of the following infections is least likely to occur in cellular
immunodeficiency?
A) CMV
B) Pneumocystis carinii
112 Pathology Questions and Answers, Prof.Dr. Murat ALPER
C) Candida esophagitis
D) Herpes zoster
E) Pneumococcal pneumonia
Cellular immunodeficiency leads to a diminished response against viruses,
fungi, and intracellular bacteria, with frequent infections caused by CMV,
Pneumocystis carinii, Candida, and Herpes zoster. Pneumococcal pneumonia is not
typically associated with cellular immunodeficiency, as the immune response against
pneumococci is primarily humoral. In humoral immunodeficiency, the response to
bacterial infections is reduced.
(Answer: E)
62. The alternative complement pathway is involved in which of the following
diseases?
A) Immune complex disease
B) Paroxysmal nocturnal hemoglobinuria
C) Sickle cell anemia
D) Beta-thalassemia major
E) Beta-thalassemia minor
Paroxysmal nocturnal hemoglobinuria (PNH) is the only hemolytic anemia
caused by an acquired defect in the complement pathway. The membrane defect
affects not only erythrocytes but also platelets and granulocytes. In PNH, CD55, CD59,
CD8, and C3 convertase are defective. PNH, a clonal disorder of multipotent stem
cells, may progress to aplastic anemia and acute leukemia. Iron deficiency anemia,
susceptibility to infections, and episodes of venous thrombosis, particularly in
hepatic, portal, and cerebral veins, are observed in PNH.
(Answer: B)
63. Which test would you request to demonstrate T lymphocytes?
A) MIF test
B) E rosette test
C) Immunoelectrophoresis
113 Pathology Questions and Answers, Prof.Dr. Murat ALPER
D) HLA typing
E) Gruber-Widal test
T lymphocytes form rosettes with sheep erythrocytes. For B lymphocytes to
form rosettes with sheep erythrocytes, complement must be present. This is also
known as the EAC test.
(Answer: B)
64. Which of the following is incorrect regarding Bruton’s disease?
A) Circulating B cells are absent or reduced.
B) Pre-B cells are absent or reduced in the bone marrow.
C) Plasma cells are absent or reduced.
D) The T cell system is normal.
E) The response to viral, fungal, and protozoal infections is generally normal.
Primary immunodeficiencies are typically inherited and become apparent
between 6 months and 2 years, as maternal antibodies decrease. Bruton’s disease is
one of the most common primary immunodeficiencies. Bruton’s (X-linked
agammaglobulinemia) involves a defect in the maturation of pre-B cells into mature B
cells, resulting in a lack of circulating antibodies (gamma globulins). The defect lies in
the Bruton tyrosine kinase (BTK) gene, which is crucial for pre-B cell maturation. BTK
is located on the X chromosome, so the disease affects males. Circulating B cells and
immunoglobulins, as well as plasma cells in tissues and Peyer’s patches in the ileum,
are absent or significantly reduced. Pre-B cells are present in the bone marrow. The
number and function of T lymphocytes are generally normal, so the response to viral,
fungal, and protozoal infections is usually unaffected. The disease becomes apparent
after 6 months, when maternal immunoglobulins decrease, leading to recurrent
bacterial infections (e.g., Staphylococcus, S. pneumoniae, H. influenzae).
Approximately 35% of patients develop autoimmune diseases such as rheumatoid
arthritis and dermatomyositis. In the absence of IgA, Giardia lamblia and certain
viruses that are normally defended by IgA frequently cause infections in the
gastrointestinal tract and eyes. Mycoplasma infections may lead to arthritis in
children. Live poliovirus vaccines rarely cause paralytic poliomyelitis.
(
Answer: B)
114 Pathology Questions and Answers, Prof.Dr. Murat ALPER
65. Which disease resembles Bruton’s disease but presents in the 2nd-3rd
decade of life, affects both sexes equally, and lacks plasma cells?
A) Hyper IgM syndrome
B) DiGeorge syndrome
C) Isolated IgA deficiency
D) Common variable immunodeficiency
E) Wiskott-Aldrich syndrome
Common variable immunodeficiency (CVID) resembles Bruton’s disease but
presents later in life, typically in the 2nd or 3rd decade, and affects both sexes
equally. All patients have hypogammaglobulinemia, usually affecting all antibody
classes, although IgG may be selectively affected. Other causes of immunodeficiency
must be ruled out before diagnosing CVID. Mature B cells are present, but plasma
cells are absent or significantly reduced, suggesting a defect in B cell differentiation.
Histologically, B cell areas (lymphoid follicles in lymph nodes, spleen, and gut) are
hyperplastic. Recurrent sinopulmonary pyogenic infections are common, with 20% of
cases involving recurrent herpes virus infections and meningoencephalitis due to
enterovirus. Persistent diarrhea due to G. lamblia may occur. Autoimmune diseases,
particularly rheumatoid arthritis, are more common. Gastric and lymphoid
malignancies are also more common. Unlike X-linked agammaglobulinemia, CVID
affects both sexes equally and presents later in life.
(Answer: D)
66. Which of the following is the most common type of immunodeficiency?
A) Bruton agammaglobulinemia
B) DiGeorge syndrome
C) Isolated IgA deficiency
D) Common variable immunodeficiency
E) Wiskott-Aldrich syndrome
Isolated IgA deficiency is the most common primary immunodeficiency
disorder, with an incidence of 1 in 700, and is the most common B cell disorder. B cells
that secrete IgA fail to mature into plasma cells. The absence of serum and secretory
IgA leads to weakened mucosal defense, resulting in recurrent respiratory,
115 Pathology Questions and Answers, Prof.Dr. Murat ALPER
gastrointestinal, and urogenital infections, chronic diarrhea (Giardia), and asthma-like
atopic disorders. IgG and IgM levels are normal or elevated. Autoimmune diseases
(e.g., SLE, RA) are common. Approximately 40% of patients have anti-IgA antibodies,
increasing the risk of allergic reactions and anaphylaxis following blood transfusion.
(Answer: C)
67. Which primary immunodeficiency results from a mutation in the CD40
ligand gene on T lymphocytes and the CD40 gene on B lymphocytes?
A) DiGeorge syndrome
B) Isolated IgA deficiency
C) Severe combined immunodeficiency
D) Hyper IgM syndrome
E) Wiskott-Aldrich syndrome
(Answer: D)
68. Which immunodeficiency disorder is characterized by a failure of T cells to
facilitate isotype switching in B cells?
A) Bruton agammaglobulinemia
B) Isolated IgA deficiency
C) Common variable immunodeficiency
D) Hyper IgM syndrome
E) DiGeorge syndrome
In response to protein antigens, IgM and IgD are initially produced, followed
by the sequential production of IgG, IgA, and IgE. This sequential production of
antibodies is known as isotype switching. Isotype switching requires the interaction
between CD40 molecules on B cells and CD40L released from CD4 T lymphocytes.
Patients with Hyper IgM syndrome produce normal or elevated levels of IgM
antibodies in response to antigens but lack the ability to produce IgG, IgA, and IgE
isotypes. The defect lies in the inability of T cells to facilitate isotype switching in B
cells, which is also influenced by CD40 interaction. Serum IgM is elevated, while IgG is
low. T and B cell counts are normal. Recurrent pyogenic infections occur due to the
low levels of opsonizing IgG antibodies, and CD40L also affects macrophages, leading
116 Pathology Questions and Answers, Prof.Dr. Murat ALPER
to increased susceptibility to Pneumocystis jirovecii infection. Patients may also
develop autoimmune hemolytic anemia, thrombocytopenia, and neutropenia due to
the elevated IgM.
(Answer: D)
69. Deletion of which region is associated with a syndrome that includes
thymic hypoplasia?
A) 5p15.2 deletion
B) 1p36 deletion
C) 22q11.2 deletion
D) 15q11.2 deletion
E) 17p11.2 deletion
(Answer: C)
70. Immunodeficiency resulting from a malformation of the 3rd and 4th
pharyngeal pouches is known as which of the following?
A) Bruton agammaglobulinemia
B) DiGeorge syndrome
C) Isolated IgA deficiency
D) Severe combined immunodeficiency
E) Hyper IgM syndrome
(Answer: B)
71. Which immunodeficiency disorder is associated with hypocalcemia and
tetany?
A) DiGeorge syndrome
B) Nezelof syndrome
C) Bruton’s disease
D) Isolated IgA deficiency
E) Wiskott-Aldrich syndrome
117 Pathology Questions and Answers, Prof.Dr. Murat ALPER
DiGeorge syndrome (thymic hypoplasia) results from a congenital
malformation affecting the 3rd and 4th pharyngeal pouches. 90% of patients have a
deletion on chromosome 22q11. The thymus, parathyroids, lip, and aortic arch are
affected. The thymus and parathyroids are absent or hypoplastic, leading to
hypocalcemic tetany. Patients have a distinctive facial appearance. Lymph nodes,
spleen, and circulating T lymphocytes are deficient. There is increased susceptibility
to viral, fungal, protozoal, and intracellular bacterial infections. B cell function and
serum immunoglobulins are usually normal. In Nezelof syndrome, the parathyroids
are normal, but there is a developmental defect in the thymus. DiGeorge syndrome
and velocardiofacial syndrome are now referred to as 22q11 deletion syndrome.
When T cell deficiency and hypocalcemia are prominent, the condition is termed
DiGeorge syndrome. When mild immunodeficiency is accompanied by significant
facial dysmorphology and cardiac abnormalities, the condition is termed
velocardiofacial syndrome. The definitive diagnosis of 22q11 deletion syndrome is
made using FISH. Schizophrenia is common in this syndrome. Hyper IgM syndrome is
characterized by a defect in isotype switching. Isolated IgA deficiency is caused by a
failure of B cells to mature into plasma cells. Bruton’s disease is characterized by a
failure of pre-B cells to mature into B cells.
(Answer: A)
72. Which disorder is characterized by an X-linked mutation in the gene
encoding IL-7 in its X-linked forms and adenosine deaminase deficiency in its
autosomal recessive forms?
A) Bruton agammaglobulinemia
B) DiGeorge syndrome
C) Severe combined immunodeficiency
D) Common variable immunodeficiency
E) Wiskott-Aldrich syndrome
(Answer: C)
73. Which of the following causes the most severe clinical disease?
A) DiGeorge syndrome
B) X-linked agammaglobulinemia
C) Isolated IgA deficiency
118 Pathology Questions and Answers, Prof.Dr. Murat ALPER
D) Common variable immunodeficiency
E) Severe combined immunodeficiency
Severe combined immunodeficiency (SCID), also known as Swiss-type
agammaglobulinemia, is characterized by deficiencies in both B and T cells, with
significant lymphopenia. However, it is primarily a T cell disorder. Without a bone
marrow transplant, most patients die within the first year of life. Opportunistic
infections, including those caused by Pseudomonas, Candida albicans, Pneumocystis
jirovecii, CMV, and HSV, are common. Autosomal recessive forms involve adenosine
deaminase deficiency, resulting in the accumulation of deoxy-ATP metabolites, which
are toxic to lymphocytes. In SCID, the thymus is frequently hypoplastic, and lymph
nodes and lymphoid tissues (e.g., tonsils, intestines, appendix) are atrophic.
(Answer: E)
74. Which condition is characterized by thrombocytopenia, eczema, and
immunodeficiency?
A) Bruton agammaglobulinemia
B) DiGeorge syndrome
C) Isolated IgA deficiency
D) Common variable immunodeficiency
E) Wiskott-Aldrich syndrome
Wiskott-Aldrich syndrome, an X-linked disorder, is characterized by
thrombocytopenia, eczema, and immunodeficiency. Initially, the thymus is normal,
but over time, there is progressive T cell deficiency in the blood and T cell areas, and
sometimes B cell deficiency. Recurrent infections and early death are common. IgM
levels are low, IgG is typically normal, and paradoxically, IgA and IgE levels are often
elevated. The risk of developing malignant lymphoma is increased. Bone marrow
transplantation is the only treatment for this condition.
(Answer: E)
75. Which component is deficient in hereditary angioedema?
A) C1 inhibitor
B) CIq
119 Pathology Questions and Answers, Prof.Dr. Murat ALPER
C) C3 activator
D) C3 inhibitor
E) C5 and C9
Complement components are synthesized in the liver. C2 deficiency is the
most common complement deficiency. Hereditary angioedema is characterized by
localized, recurrent episodes of edema, particularly affecting the mucous membranes
of the face and oropharynx. There is a deficiency in C1 inhibitor (C-INH), with reduced
levels of C2 and C4. C2 and C4 deficiencies are typically asymptomatic, as most
infections can be controlled through the alternative complement pathway. Screening
for hereditary angioedema involves
measuring C4 levels, and treatment involves administering androgens. In the
classic pathway, C5-C9 deficiency leads to recurrent Neisseria (gonococcus,
meningococcus) infections. C3 plays a crucial role in both the classic and alternative
pathways, and its deficiency results in increased susceptibility to pyogenic bacterial
infections. C1q, C2, and C4 deficiencies are associated with a higher risk of immune
complex diseases like systemic lupus erythematosus, possibly due to impaired
clearance of immune complexes.
(Answer: A)
76. Which systemic disease is characterized by viral and fungal infections in
the oral cavity, along with Kaposi's sarcoma and hairy leukoplakia?
A) Infectious mononucleosis
B) Rendu-Osler-Weber syndrome
C) Pemphigus
D) HIV infection
E) Dermatomyositis
(Answer: D)
77. Which of the following immune system functions is not expected to be
impaired in AIDS?
A) Hypogammaglobulinemia
B) Decreased IL-2 production
120 Pathology Questions and Answers, Prof.Dr. Murat ALPER
C) Decreased memory T cells
D) Reduced proliferative response to mitogens
E) Decreased delayed-type hypersensitivity
(Answer: A)
78. Which two tumors are most commonly associated with AIDS?
A) Kaposi's sarcoma and leukemia
B) Kaposi's sarcoma and Hodgkin's lymphoma
C) Kaposi's sarcoma and Non-Hodgkin's lymphoma
D) Lymphoma and squamous cell carcinoma
E) Leukemia and lymphoma
(Answer: C)
79. Kaposi's sarcoma in AIDS patients originates from which cells?
A) T lymphocytes
B) B lymphocytes
C) Vascular endothelial cells
D) Squamous epithelial cells
E) Melanocytes
Approximately 20% of AIDS patients develop Kaposi's sarcoma, a spindle cell
tumor derived from vascular endothelial cells. Kaposi’s sarcoma is associated with
HHV-8 (KSHV). Other common tumors in AIDS patients include Non-Hodgkin
lymphoma (B cell immunoblastic), Hodgkin lymphoma, and squamous cell carcinoma
of the cervix in women and the anus in men. Viruses like HHV-8, HPV, and EBV are
implicated in these malignancies. CNS lymphomas and lymphomas in body cavities
are also common. The most common opportunistic infection in AIDS patients is
Pneumocystis jirovecii. The CD4/CD8 ratio, which is normally 2, drops to as low as 0.5
in AIDS (this ratio can also be seen in some viral infections and is not specific to AIDS).
Interestingly, AIDS is associated with polyclonal B cell activation and
hypergammaglobulinemia. The two primary target organs in AIDS are the immune
system and the nervous system. Although male homosexual intercourse remains a
121 Pathology Questions and Answers, Prof.Dr. Murat ALPER
common mode of HIV transmission, heterosexual transmission has become more
prominent in recent years. HIV screening using the p24 antigen is advantageous as it
can be detected before the development of humoral antibodies. The CD4 molecule
has a high affinity for HIV as a receptor. In addition to binding to CD4, the HIV
envelope glycoprotein gp120 interacts with the coreceptors CXCR4 (primarily on T
cells) and CCR5 (primarily on macrophages) to facilitate viral entry. Gp41 undergoes
structural changes that allow the virus to enter the cell. The R5 strain of HIV is
responsible for transmission in over 90% of cases.
(Answer: C)
80. Which condition causes primary amyloidosis?
A) Multiple myeloma
B) Tuberculosis
C) Rheumatoid arthritis
D) Medullary thyroid carcinoma
E) Reactive systemic amyloidosis
(Answer: A)
81. Which of the following is incorrect regarding amyloid?
A) Rectal biopsy is used for diagnosis
B) Stains with Congo red
C) AL protein accumulates in primary amyloidosis
D) AL protein accumulates in reactive amyloidosis
E) Primary amyloidosis occurs in multiple myeloma.
(Answer: D)
82. A patient undergoing hemodialysis for chronic renal failure presents with
swelling and pain in the wrist. Biopsy of the wrist shows eosinophilic material that
stains positively with Congo red. The precursor of the material accumulated in this
patient is which of the following?
A) Immunoglobulin light chain
B) Transthyretin
122 Pathology Questions and Answers, Prof.Dr. Murat ALPER
C) ß2-microglobulin
D) Calcitonin
E) Apolipoprotein
(Answer: C)
83. Which of the following is not an amyloid fibril protein?
A) Transthyretin
B) β2-microglobulin
C) Calcitonin
D) Prealbumin
E) Transferrin
(Answer: E)
84. Which protein accumulates in tissues in familial Mediterranean fever?
A) Immunoglobulin light chains
B) Atrial natriuretic factor
C) Calcitonin
D) Beta 2 microglobulin
E) Serum amyloid-associated protein
(Answer: E)
85. Which type of amyloid accumulates in patients undergoing long-term
dialysis?
A) Beta 2-microglobulin
B) Heavy chain
C) Transthyretin
D) Light chain
E) Iron accumulation
(Answer: A)
123 Pathology Questions and Answers, Prof.Dr. Murat ALPER
86. Which of the following is not a cause of reactive systemic amyloidosis?
A) Tuberculosis
B) Rheumatoid arthritis
C) Aging
D) Bronchiectasis
E) Chronic osteomyelitis
Amyloid is a beta-pleated sheet protein that stains brick-red with Congo red
and exhibits apple-green birefringence under polarized light. Crystal violet staining
can also be used. There are three major classes of amyloid:
1) AL (amyloid light chain): derived from immunoglobulin light chains.
2) AA (amyloid-associated): a non-immunoglobulin protein synthesized in the
liver.
3) Ab: amyloid found in the cerebral lesions of Alzheimer's disease.
In primary amyloidosis, AL protein accumulates, typically seen in multiple
myeloma. In reactive systemic amyloidosis (secondary amyloidosis), AA protein
accumulates. Chronic inflammatory conditions such as tuberculosis, bronchiectasis,
chronic osteomyelitis, ulcerative colitis, rheumatoid arthritis, ankylosing spondylitis,
tumors (e.g., medullary thyroid carcinoma, pancreatic islet cell tumors,
pheochromocytoma, renal cell carcinoma, Hodgkin's lymphoma), etc., are associated
with reactive systemic amyloidosis. Beta-2 microglobulin accumulates in patients with
chronic renal failure and dialysis. Long-term dialysis patients often accumulate beta-2
microglobulin in wrist carpal ligaments, leading to carpal tunnel syndrome.
Heredofamilial amyloidosis involves the accumulation of AA protein, most commonly
in familial Mediterranean fever (FMF). FMF is most prevalent among Armenians,
Jews, and Arabs. Amyloid also accumulates in medullary thyroid carcinoma,
pancreatic islet cell tumors, and pheochromocytoma. In senile cardiac and cerebral
amyloidosis (associated with Alzheimer's disease), transthyretin and beta-amyloid
protein accumulate. In amyloidosis of the kidneys, renal involvement is the most
common and severe organ involvement. Rectal, gingival, and abdominal fat biopsies
are used for diagnosis. In the spleen, there are two patterns of amyloid deposition:
sago spleen, where amyloid accumulates in splenic follicles, and lardaceous spleen,
where amyloid accumulates in the walls of primary splenic sinuses. In the kidneys,
124 Pathology Questions and Answers, Prof.Dr. Murat ALPER
amyloid primarily accumulates in the glomeruli and vessels. In the liver, it primarily
affects the space of Disse.
(Answer: C)
87. Which of the following proteins is involved in the pathogenesis of
dementia in a 40-year-old patient with Down syndrome?
A) Prealbumin
B) Beta-protein
C) Calcitonin
D) Light chains
E) Serum amyloid-associated protein
Beta-protein is encoded on chromosome 21, which is trisomic in Down
syndrome. This protein is converted into amyloid, which is toxic to brain cells, leading
to Alzheimer's disease. Dementia occurs in Down syndrome patients who survive into
old age due to the lower incidence of cardiovascular complications. Prealbumin is
associated with restrictive cardiomyopathy caused by amyloidosis. Calcitonin is
converted to amyloid in medullary thyroid carcinoma. Light chains are seen in
multiple myeloma. Serum amyloid-associated protein is an acute-phase reactant that
can serve as a precursor to amyloid in secondary amyloidosis, which occurs in
conditions such as tuberculosis, leprosy, chronic osteomyelitis, and rheumatoid
arthritis.
(Answer: B)
88. In which of the following conditions is amyloidosis least likely to occur?
A) Congestive cardiomyopathy
B) Peripheral neuropathy
C) Nephrotic syndrome
D) Alzheimer's disease
E) Bronchiectasis
Amyloidosis is commonly seen in restrictive cardiomyopathy, peripheral
neuropathy (e.g., carpal tunnel syndrome), nephrotic syndrome (with light chains in
primary amyloidosis), Alzheimer's disease (with beta-protein amyloid encoded on
125 Pathology Questions and Answers, Prof.Dr. Murat ALPER
chromosome 21), and bronchiectasis. Among these, congestive (dilated)
cardiomyopathy is the least likely to be associated with amyloidosis.
(Answer: A)
89. Which disease causes urticarial lesions due to the development of immune
complexes?
A) IMN
B) Influenza
C) Typhoid
D) Mumps
E) Hepatitis B virus
Hepatitis B infection has extrahepatic manifestations, including a serum
sickness-like syndrome with a maculopapular rash, polyarthralgia, and less
commonly, arthritis and urticaria. Polyarteritis nodosa (PAN) occurs in 30-40% of HBV-
positive cases. Glomerulonephritis is another manifestation.
(Answer: E)
126 Pathology Questions and Answers, Prof.Dr. Murat ALPER
SECTION 7: NEOPASIA
İşte istenilen değişikliklerle sorular ve yanıtlar:
---
1. Which of the following is a benign tumor?
A) Mesothelioma
B) Seminoma
C) Papilloma
D) Melanoma
E) Lymphoma
Answer: C
---
2. Which of the following statements about adenomas is correct?
A) A benign tumor arising from glandular epithelium
B) A malignant tumor arising from glandular epithelium
C) A benign tumor arising from stratified squamous epithelium
D) A malignant tumor arising from mesoderm
E) A benign tumor arising from connective tissue
Answer: A
---
3. Which of the following is false?
A) Tumors with dense fibrous stroma are called scirrhous tumors.
127 Pathology Questions and Answers, Prof.Dr. Murat ALPER
B) Sarcoma is a malignant tumor of mesenchymal origin.
C) Blastomas are tumors originating from embryonic cells.
D) Hamartomas contain tissues from three germ layers.
E) Epithelial malignant neoplasms are called carcinomas.
Answer: D
---
4. What is the term for the presence of tissue or an organ in an abnormal
location?
A) Differentiation
B) Heterotopia
C) Acanthosis
D) Hypertrophy
E) Hyperplasia
Answer: B
---
5. What is it called when pancreatic tissue is found in the stomach wall?
A) Hamartoma
B) Choristoma
C) Metaplasia
D) Chloroma
E) Adenoma
Answer: B
128 Pathology Questions and Answers, Prof.Dr. Murat ALPER
---
6. What is the name for tumors that contain elements from three germ layers?
A) Hamartoma
B) Choristoma
C) Teratoma
D) Carcinoma
E) Sarcoma
Answer: C
---
7. Which tumor contains multiple neoplastic cell types originating from more
than one germ layer?
A) Pleomorphic adenoma
B) Malignant mixed tumor of the salivary gland
C) Immature teratoma
D) Wilms tumor
E) Nevus
Answer: C
---
8. What is the term for the loss of cellular uniformity and architectural
organization?
A) Apoptosis
B) Necrosis
C) Hyperplasia
129 Pathology Questions and Answers, Prof.Dr. Murat ALPER
D) Metaplasia
E) Dysplasia
Answer: E
---
9. What is the term for the variability in cell shape, size, and staining
characteristics within a tumor?
A) Atypia
B) Pleomorphism
C) Anaplasia
D) Dysplasia
E) Undifferentiation
Answer: B
---
10. Which of the following refers to the morphological resemblance of tumor
cells to the cell of origin?
A) Dysplasia
B) Differentiation
C) Metaplasia
D) Atypia
E) Anaplasia
Answer: B
---
130 Pathology Questions and Answers, Prof.Dr. Murat ALPER
11. In which condition is poor differentiation a key feature of the tumor?
A) Scirrhous carcinoma
B) Carcinoma in situ
C) Anaplastic carcinoma
D) Medullary carcinoma
E) Carcinosarcoma
Answer: C
---
12. What is the most definitive feature that distinguishes benign from
malignant tumors?
A) Number of mitoses
B) Presence of a capsule
C) Metastasis
D) Atypical mitoses
E) Necrosis
Answer: C
---
13. Which of the following is a definitive criterion for malignancy in a tumor?
A) Presence of metastasis
B) High mitotic rate
C) Lack of a capsule
D) Rapid growth
E) Presence of necrosis
131 Pathology Questions and Answers, Prof.Dr. Murat ALPER
Answer: A
---
14. Which of the following is more significant in determining the malignancy
of a tumor?
A) Presence of necrosis
B) Lack of a capsule
C) Infiltration into surrounding organs
D) Atypical mitoses
E) Number of mitoses
Answer: C
---
15. What is the most important prognostic factor in cancer?
A) Tumor stage
B) Tumor grade
C) Vascular invasion
D) Mitotic index
E) Lymphatic invasion
Answer: A
---
16. Which of the following malignant tumors is generally considered not to
metastasize?
132 Pathology Questions and Answers, Prof.Dr. Murat ALPER
A) Squamous cell carcinoma
B) Basal cell carcinoma
C) Adenoid cystic carcinoma
D) Seminoma
E) Leiomyosarcoma
Answer: B
---
17. What is the definition of desmoplasia?
A) Stromal invasion
B) Tumor rich in fibrous tissue
C) Increased anaplastic cells
D) Stroma rich in fat
E) Excessive vascular tissue
Answer: B
---
18. Which of the following contains both mesenchymal and epithelial
malignant cells?
A) Leiomyosarcoma
B) Endometrial stromal sarcoma
C) Rhabdomyosarcoma
D) Carcinosarcoma
E) Adenosarcoma
Answer: D
133 Pathology Questions and Answers, Prof.Dr. Murat ALPER
---
19. Which of the following is synonymous with intraepithelial carcinoma?
A) Dysplasia
B) Bowen disease
C) Microinvasive carcinoma
D) Scirrhous carcinoma
E) Early gastric carcinoma
Answer: B
---
20. Which tissue is most resistant to metastasis?
A) Bone
B) Connective tissue
C) Epithelium
D) Nerve
E) Cartilage
Answer: E
---
21. Which of the following increases earliest in tumor angiogenesis?
A) VEGF
B) HIF-1 alpha
C) TGF-beta
134 Pathology Questions and Answers, Prof.Dr. Murat ALPER
D) Angiostatin
E) Insulin-like growth factor
Answer: B
---
22. Which of the following can stimulate angiogenesis?
A) Thrombospondin-1
B) Angiostatin
C) Endostatin
D) Vascularostatin
E) VEGF
Answer: E
---
23. Which of the following is known as the "death receptor"?
A) CD95
B) P53
C) BCL2
D) BAX
E) BID
Answer: A
---
135 Pathology Questions and Answers, Prof.Dr. Murat ALPER
24. Through which pathway do epithelial tumors most commonly spread?
A) Hematogenous
B) Lymphatic
C) Implantation
D) Via cerebrospinal fluid
E) Contact
Answer: B
---
25. Which of the following tumors can metastasize through cerebrospinal
fluid?
A) Meningioma
B) Medulloblastoma
C) Oligodendroglioma
D) Astrocytoma
E) Schwannoma
Answer: B
---
26. Which of the following tumors most commonly spreads via veins?
A) Ependymoma
B) Medulloblastoma
C) Adenocarcinoma
D) Squamous cell carcinoma
E) Renal cell carcinoma
136 Pathology Questions and Answers, Prof.Dr. Murat ALPER
Answer: E
---
27. Primary tumors of which of the following organs typically metastasize by
seeding?
A) Lung and kidney
B) Ovary and brain
C) Colon and uterus
D) Liver and cervix
E) Prostate and bladder
Answer: B
---
28. What is the term for the first regional lymph node to which a primary
tumor drains?
A) Sentinel lymph node
B) Sinus histiocytosis
C) Lymphadenitis
D) Skip metastasis
E) Follicular hyperplasia
Answer: A
---
29. In which of the following sites is primary cancer more common than
metastatic cancer?
137 Pathology Questions and Answers, Prof.Dr. Murat ALPER
A) Lung
B) Ovary
C) Bone
D) Liver
E) Lymph node
Answer: B
---
30. Which of the following is the most likely primary site for a metastatic
cancer found in the brain, adrenal glands, and liver?
A) Rectosigmoid
B) Stomach
C) Esophagus
D) Breast
E) Lung
Answer: E
---
31. Which of the following is not considered in the grading of malignant
tumors?
A) Cell origin
B) Degree of anaplasia
C) Degree of differentiation
D) Extent and distribution of the disease
E) Number of mitoses
138 Pathology Questions and Answers, Prof.Dr. Murat ALPER
Answer: D
---
32. Which of the following is not used in cancer staging?
A) Primary lesion size
B) Lymph node involvement
C) Anaplasia
D) Metastasis
E) Local invasion
Answer: C
---
33. Which of the following plays the most significant role in cancer cachexia?
A) Interferon-gamma
B) Ubiquitin-proteasome
C) Fas ligand
D) Interleukin-12
E) TGF-beta
Answer: B
---
34. Which of the following is used to ripen fruits and may cause leukemia?
A) Chromium
B) Cadmium
139 Pathology Questions and Answers, Prof.Dr. Murat ALPER
C) Nickel
D) Ethylene oxide
E) Beryllium
Answer: D
---
35. Which of the following pairings is incorrect?
A) Asbestos - Mesothelioma
B) Beta-naphthylamine - Bladder carcinoma
C) Benzene - Acute myeloid leukemia
D)
Vinyl chloride - Liver angiosarcoma
E) Arsenic - Lymphoma
Answer: E
---
36. Which of the following is not a defective DNA repair syndrome?
A) Xeroderma pigmentosum
B) Bloom syndrome
C) Hereditary nonpolyposis colon cancer
D) Fanconi anemia
E) Ataxia-telangiectasia
Answer: C
140 Pathology Questions and Answers, Prof.Dr. Murat ALPER
---
37. Which of the following is not a hereditary predisposition to cancer?
A) Xeroderma pigmentosum
B) Bloom syndrome
C) X-linked agammaglobulinemia
D) Fanconi anemia
E) Ataxia-telangiectasia
Answer: C
---
38. Which of the following has the lowest statistical risk of developing into a
malignant neoplasm?
A) Cervical squamous dysplasia
B) Uterine leiomyomas
C) Chronic ulcerative colitis
D) Endometrial atypical hyperplasia
E) Hepatic cirrhosis
Answer: B
---
39. In which of the following conditions does the risk of cancer not increase?
A) Edge of a chronic skin fistula
B) Endometrial hyperplasia
C) Chronic atrophic gastritis
141 Pathology Questions and Answers, Prof.Dr. Murat ALPER
D) Solar elastosis
E) Solar keratosis
Answer: D
---
40. Which of the following is a lesion more likely to progress to cancer?
A) Uterine leiomyoma
B) Prostatic hyperplasia
C) Actinic keratosis
D) Fibroadenoma
E) Capillary hemangioma
Answer: C
---
41. Which of the following is a likely precursor lesion to cancer?
A) Endometrial polyp
B) Complete mole
C) Lichen sclerosus
D) Intraductal papilloma
E) Traumatic fat necrosis
Answer: B
---
142 Pathology Questions and Answers, Prof.Dr. Murat ALPER
42. Which of the following is not a hereditary neoplasm?
A) Retinoblastoma
B) Breast cancer
C) Colon cancer
D) Osteosarcoma
E) MEN type II
Answer: D
---
43. Which of the following is not an autosomal dominant cancer syndrome?
A) Retinoblastoma
B) Li-Fraumeni syndrome
C) Familial adenomatous polyposis
D) Ataxia-telangiectasia
E) Multiple endocrine neoplasia type II
Answer: D
---
44. In which autosomal dominant tumor is the risk of osteosarcoma
increased?
A) Neurofibromatosis
B) Wilms tumor
C) Retinoblastoma
D) Hereditary nonpolyposis colon cancer
E) Familial adenomatous polyposis coli
143 Pathology Questions and Answers, Prof.Dr. Murat ALPER
Answer: C
---
45. Which of the following tumors does not show dominant inheritance?
A) Medullary thyroid carcinoma
B) Wilms tumor
C) Neuroblastoma
D) Retinoblastoma
E) Angiosarcoma
Answer: E
---
46. The Philadelphia chromosome is characteristic of which of the following
tumors?
A) Burkitt lymphoma
B) Hodgkin lymphoma
C) Acute myeloid leukemia
D) Chronic lymphocytic leukemia
E) Chronic myeloid leukemia
Answer: E
---
47. Which of the following is not involved in the development of a tumor with
a viral etiology?
A) Gastric MALToma
144 Pathology Questions and Answers, Prof.Dr. Murat ALPER
B) Sinonasal papilloma
C) Cervical in situ carcinoma
D) Nasopharyngeal carcinoma
E) Hepatocellular carcinoma
Answer: A
---
48. Which of the following tumor-virus associations is incorrect?
A) EBV - Burkitt lymphoma
B) Human papillomavirus - Cervical carcinoma
C) EBV - Nasopharyngeal carcinoma
D) HTLV-1 - T-cell leukemia/lymphoma
E) Hepatitis A virus - Hepatocellular carcinoma
Answer: E
---
49. Which of the following is an oncogenic RNA virus?
A) HPV
B) EBV
C) HTLV-1
D) HHV-8
E) Merkel cell polyomavirus
Answer: C
145 Pathology Questions and Answers, Prof.Dr. Murat ALPER
---
50. In which anatomical location is HPV-associated squamous cell carcinoma
more likely to occur than in others?
A) Oropharynx
B) Nasopharynx
C) Larynx
D) Paranasal sinuses
E) Nasal cavity
Answer: A
---
51. Which of the following HPV types are high-risk?
A) HPV types 1, 2
B) HPV types 2, 4
C) HPV types 7, 8
D) HPV types 6, 11
E) HPV types 16, 18, 31
Answer: E
---
52. Which of the following can kill tumor cells without prior sensitization?
A) Natural killer cells
B) Cytotoxic T lymphocytes
C) Macrophages
D) Humoral mechanisms
146 Pathology Questions and Answers, Prof.Dr. Murat ALPER
E) Giant cells
Answer: A
---
53. Which cancer is most likely to increase in immunocompromised
individuals?
A) Non-Hodgkin lymphoma
B) Hodgkin lymphoma
C) Small cell carcinoma
D) Squamous cell carcinoma
E) Breast carcinoma
Answer: A
---
54. Which of the following paraneoplastic syndrome associations is incorrect?
A) Cerebellar hemangioblastoma - Erythropoietin
B) Small cell carcinoma - ACTH
C) Bronchial squamous cell carcinoma - Parathyroid hormone
D) Intracranial tumors - ADH
E) Renal cell carcinoma - ACTH
Answer: E
---
147 Pathology Questions and Answers, Prof.Dr. Murat ALPER
55. Which organ’s tumor is most likely to cause secondary polycythemia and
hypercalcemia?
A) Lung
B) Liver
C) Kidney
D) Thyroid
E) Breast
Answer: C
---
56. Which of the following tumor marker associations is incorrect?
A) CEA - Colorectal carcinoma
B) AFP - Hepatocellular carcinoma
C) AFP - Yolk sac tumor
D) HCG - Endodermal sinus tumor
E) CEA - Pancreatic carcinoma
Answer: D
---
57. Which of the following is not used to assess metastatic potential in
immunohistochemical analysis?
A) Laminin
B) Type IV collagenase
C) Cathepsin D
D) Ki-67
E) Keratin
148 Pathology Questions and Answers, Prof.Dr. Murat ALPER
Answer: E
---
58. Which of the following tumor and ultrastructural findings is correct?
A) Small cell lung carcinoma: Dense neurosecretory granules
B) Angiosarcoma: Tonofilaments
C) Rhabdomyosarcoma: Birbeck granules
D) Neuroblastoma: Melanosomes
E) Histiocytosis X: Weibel-Palade bodies
Answer: A
---
59. Which of the following associations between cancer, marker, and
pathogenic factor is correct?
A) Hepatocellular carcinoma: Beta2-microglobulin; Aflatoxin
B) Breast cancer: CA125; Ionizing radiation
C) Pancreatic carcinoma: Bombesin; Polycyclic hydrocarbons
D) Bladder carcinoma: CEA; Phenacetin
E) Choriocarcinoma: Beta-HCG; Severe alpha-thalassemia
Answer: E
---
60. What is the most common skin cancer?
A) Squamous cell carcinoma
149 Pathology Questions and Answers, Prof.Dr. Murat ALPER
B) Basal cell carcinoma
C) Malignant melanoma
D) Adenoid cystic carcinoma
E) Angiosarcoma
Answer: B
---
61. Which of the following is incorrect about squamous cell carcinoma?
A) Leukoplakia is a precursor lesion.
B) Fair-skinned individuals working outdoors are at higher risk.
C) Solar elastosis is a risk factor.
D) Long-standing chronic inflammation with discharge is a risk factor.
E) Radiation is a risk factor.
Answer: C
---
62. Where is basal cell carcinoma primarily located?
A) Sole of the foot
B) Cervix
C) Lip
D) Palm of the hand
E) Ear
Answer: E
150 Pathology Questions and Answers, Prof.Dr. Murat ALPER
---
63. In which tumor is "rodent ulcer" typically seen?
A) Seminoma
B) Squamous cell carcinoma
C) Fibrosarcoma
D) Basal cell carcinoma
E) Choriocarcinoma
Answer: D
---
64. Which of the following has the highest likelihood of transforming into
malignant melanoma?
A) Intradermal nevus
B) Junctional nevus
C) Dysplastic nevus
D) Blue nevus
E) Compound nevus
Answer: C
---
65. Which of the following is the most important prognostic factor in
malignant melanoma?
A) Depth of the lesion
B) Anatomical site
C) Number of mitoses
151 Pathology Questions and Answers, Prof.Dr. Murat ALPER
D) Lymphocytic response
E) Bleeding and ulceration
Answer: A
---
66. What are the three most common tumors in children under five years old,
in decreasing order?
A) Leukemia - Lymphoma - Brain tumors
B) Leukemia - Central nervous system tumors - Wilms tumor
C) Lymphoma - Brain tumors - Kidney tumors
D) Brain tumors - Lymphoma -
Kidney tumors
E) Leukemia - Neuroblastoma - Soft tissue tumors
Answer: B
---
67. What are the two most common tumors in children aged five to nine
years?
A) Leukemia - Brain tumors
B) Leukemia - Lymphoma
C) Leukemia - Kidney tumors
D) Brain tumors - Lymphoma
E) Brain tumors - Kidney tumors
Answer: A
152 Pathology Questions and Answers, Prof.Dr. Murat ALPER
---
68. What are the two most common tumors in individuals aged 10 to 20
years?
A) Lymphoma - Brain tumors
B) Leukemia - Brain tumors
C) Lymphoma - Leukemia
D) Lymphoma - Ovary, testis tumors
E) Leukemia - Bone tumors
Answer: C
---
69. What is the most common soft tissue sarcoma that occurs in the head,
neck, and urogenital region during childhood and adolescence?
A) Rhabdomyosarcoma
B) Leiomyosarcoma
C) Angiosarcoma
D) Synovial sarcoma
E) Fibrosarcoma
Answer: A
---
70. What is the most common malignant soft tissue tumor in childhood?
A) Rhabdomyosarcoma
B) Leiomyosarcoma
153 Pathology Questions and Answers, Prof.Dr. Murat ALPER
C) Malignant fibrous histiocytoma
D) Fibrosarcoma
E) Liposarcoma
Answer: A
---
71. What are the three most common tumors in men, in decreasing order?
A) Lung - Stomach - Colon and rectum
B) Prostate - Lung - Pancreas
C) Prostate - Lung - Colon and rectum
D) Prostate - Urinary tract - Lung
E) Lung - Prostate - Melanoma
Answer: C
---
72. What are the three most common tumors in women, in decreasing order?
A) Colon and rectum - Breast - Pancreas
B) Breast - Lung - Colon and rectum
C) Breast - Ovary - Cervix
D) Breast - Uterus - Lung
E) Lung - Colon and rectum - Uterus
Answer: B
---
154 Pathology Questions and Answers, Prof.Dr. Murat ALPER
73. What are the three most common causes of cancer death in men, in
decreasing order?
A) Lung - Liver - Colon
B) Colon and rectum - Prostate - Pancreas
C) Lung - Colon and rectum - Prostate
D) Prostate - Lung - Pancreas
E) Lung - Leukemia - Prostate
Answer: C
---
74. What are the three most common causes of cancer death in women, in
decreasing order?
A) Lung - Breast - Colon and rectum
B) Breast - Lung - Lymphoma
C) Breast - Ovary - Uterus
D) Lung - Uterus - Breast
E) Breast - Colon and rectum - Ovary
Answer: A
---
75. In a 65-year-old male with pleural effusion, where are the primary cancer
cells most likely to be located?
A) Mesothelioma
B) Colon carcinoma
C) Lung carcinoma
155 Pathology Questions and Answers, Prof.Dr. Murat ALPER
D) Prostate carcinoma
E) Lymphoma
Answer: C
---
76. A 30-year-old male with polycythemia and motor difficulties has multiple
family members with brain and kidney tumors. A brain CT scan reveals a vascular
lesion in the cerebellum. What is the most likely diagnosis?
A) Medulloblastoma
C) Astrocytoma
D) Anaplastic astrocytoma
E) Hemangioblastoma
Answer: E
---
77. Which of the following is true about oral cavity carcinomas?
A) They are mostly adenocarcinomas.
B) They are more common in women.
C) They are most common on the upper lip.
D) Erythroplakia is a predisposing condition.
E) They are most common on the hard palate.
Answer: D
---
156 Pathology Questions and Answers, Prof.Dr. Murat ALPER
78. In which of the following tumors are Flexner-Wintersteiner rosettes seen?
A) Neuroblastoma
B) Wilms tumor
C) Hepatoblastoma
D) Retinoblastoma
E) Medulloblastoma
Answer: D
---
79. In infants and children, where are teratomas most commonly located?
A) Head and neck
B) Mediastinum
C) Retroperitoneum
D) Gonads
E) Sacrococcygeal region
Answer: E
---
80. What is the most common tumor in infants?
A) Hemangioma
B) Lymphangioma
C) Teratoma
D) Neuroblastoma
E) Ganglion
157 Pathology Questions and Answers, Prof.Dr. Murat ALPER
Answer: A
---
81. In which location is cystic hygroma most commonly found?
A) Retroperitoneum
B) Wrist
C) Ankle
D) Neck and axilla
E) Back
Answer: D
---
82. What is the most common mesenchymal tumor in the abdomen?
A) Lipoma
B) Hemangioma
C) Gastrointestinal stromal tumor
D) Lymphangioma
E) Fibroma
Answer: C
---
83. Which of the following is a neoplastic disease of T lymphocytes?
A) Multiple myeloma
B) AML
158 Pathology Questions and Answers, Prof.Dr. Murat ALPER
C) CML
D) Macroglobulinemia
E) Mycosis fungoides
Answer: E
---
84. Which of the following tumors does not show spontaneous regression?
A) Neuroblastoma
B) Choriocarcinoma
C) Retinoblastoma
D) Malignant melanoma
E) Thyroid papillary carcinoma
Answer: E
---
85. Which soft tissue tumor typically consists of small, round cells the size of
lymphocytes?
A) Leiomyosarcoma
B) Fibrosarcoma
C) Undifferentiated pleomorphic sarcoma
D) Malignant schwannoma
E) Primitive neuroectodermal tumor
Answer: E
---
159 Pathology Questions and Answers, Prof.Dr. Murat ALPER
86. Which of the following is not part of the group of small round blue cell
tumors?
A) Rhabdomyosarcoma
B) Ewing sarcoma/PNET
C) Neuroblastoma
D) Lymphoma
E) Angiosarcoma
Answer: E
---
87. A mutation in which of the following DNA repair genes is associated with
an increased risk of breast cancer in men and women, as well as prostate, ovarian,
pancreatic, gastric, and bile duct cancers?
A) BRCA1
B) BRCA2
C) APC
D) TP53
E) ATM
Answer: B
---
88. What is the name of the condition associated with mutations in the p53
gene located on the short arm of chromosome 17, leading to the development of
multiple tumors such as leukemia, lymphoma, osteosarcoma, and breast carcinoma?
A) Li-Fraumeni syndrome
B) Prader-Willi syndrome
160 Pathology Questions and Answers, Prof.Dr. Murat ALPER
C) Angelman syndrome
D) Beckwith-Wiedemann syndrome
E) Denys-Drash syndrome
Answer: A
---
89. Mutations in the ras oncogene result in the production of a protein with
which of the following characteristics?
A) Decreased GTPase activity
B) Decreased reverse transcriptase activity
C) Increased protein phosphatase activity
D) Increased response to growth factors
E) Increased tyrosine kinase activity
Answer: A
---
90. Which of the following is a tumor suppressor gene that plays a role in
inhibiting signal transduction?
A) KRAS
B) APC
C) BRAF
D) RET
E) Cyclin D
Answer: B
161 Pathology Questions and Answers, Prof.Dr. Murat ALPER
---
91. In which of the following is the inactivation of a suppressor gene least
likely?
A) Li-Fraumeni multicancer syndrome
B) HPV-induced cancers
C) Retinoblastoma
D) Familial adenomatous polyposis coli
E) Follicular lymphoma
Answer: E
---
92. In which case do both oncogenes get activated through translocation?
A) ras and erbB2
B) sis and N-myc
C) erbB2 and abl
D) abl and c-myc
E) c-myc and ras
Answer: D
---
93. Tumors are generally clonal with which characteristic?
A) Polyclonal
B) Bi- and monoclonal
C) Bi- and polyclonal
D) Tri- and biclonal
162 Pathology Questions and Answers, Prof.Dr. Murat ALPER
E) Monoclonal
Answer: E
---
94. Which of the following is not a structural genetic abnormality seen in
malignancies?
A) Translocation
B) Inversion
C) Deletion
D) Insertion
E) Trisomy
Answer: E
---
95. Which tumor suppressor gene mutation is associated with kidney cancers,
pheochromocytoma, and renal cysts?
A) VHL
B) WT1
C) NF1
D) APC
E) RB
Answer: A
---
163 Pathology Questions and Answers, Prof.Dr. Murat ALPER
96. Which of the following conditions is characterized by cysts in the liver,
kidneys, and pancreas, along with numerous angiomas?
A) Von Hippel-Lindau disease
B) Von Recklinghausen disease
C) Marfan syndrome
D) Familial hypercholesterolemia
E)
Tuberous sclerosis
Answer: A
---
97. In the development of neoplasms due to cellular DNA damage caused by
ionizing radiation, mutations in which gene play a significant role?
A) c-abl
B) k-ras
C) c-myc
D) p53
E) bcl-2
Answer: D
---
98. Which type of radiation is not associated with cancer induction?
A) Ultraviolet
B) Alpha rays
C) Infrared
164 Pathology Questions and Answers, Prof.Dr. Murat ALPER
D) Beta rays
E) Protons
Answer: C
---
99. Which of the following is the most frequently observed transcription factor
oncogene in human tumors?
A) MYC
B) MYB
C) JUN
D) FOS
E) REL
Answer: A
---
### REVIEW
(Review the topics below and write down what you remember...)
- Characteristics of benign tumors?
- Characteristics of malignant tumors?
- Differences between benign and malignant tumors?
- Differentiation and anaplasia?
- Metastasis?
- Metastasis pathways?
- Cancer epidemiology?
- Cancer and heredity?
165 Pathology Questions and Answers, Prof.Dr. Murat ALPER
- Hereditary cancers?
- Preneoplastic diseases?
- Cancer and oncogenes?
- Tumor suppressor genes?
- The concept of angiogenesis?
- Carcinogenic agents?
- Radiation and carcinogenesis?
- Viral and microbial carcinogens?
- Local effects of tumors?
- Systemic effects of tumors?
- Paraneoplastic syndromes?
- Grading?
- Staging?
- Laboratory diagnosis of cancer?
- Immunohistochemical markers?
- Tumor markers?
- Tumor nomenclature?
166 Pathology Questions and Answers, Prof.Dr. Murat ALPER
SECTION 8: GENETICS
Sure, here is the revised translation without the content in parentheses and
with the additional questions translated:
---
1. Which of the following diseases is more common in live-born infants?
A) Polycystic kidney disease
B) Marfan syndrome
C) Cystic fibrosis
D) Phenylketonuria
E) Familial hypercholesterolemia
Answer: E
---
2. What type of genetic defect is found in sickle cell anemia?
A) Point mutation
B) Frameshift mutations
C) Trinucleotide repeat mutations
D) Deletion
E) Nonsense mutations
Answer: A
---
3. Which of the following diseases is caused by trinucleotide repeat mutation?
167 Pathology Questions and Answers, Prof.Dr. Murat ALPER
A) Mitochondrial myopathies
B) Duchenne muscular dystrophy
C) Congenital muscular dystrophy
D) Myotonic dystrophy
E) Guillain-Barré syndrome
Answer: D
---
4. Which of the following is an example of a trinucleotide repeat mutation?
A) Sickle cell anemia
B) Fragile X syndrome
C) Hemophilia
D) Galactosemia
E) Hereditary spherocytosis
Answer: B
---
5. What type of inheritance is generally seen in storage diseases?
A) Autosomal dominant
B) Autosomal codominant
C) Autosomal recessive
D) X-linked recessive
E) X-linked dominant
Answer: C
168 Pathology Questions and Answers, Prof.Dr. Murat ALPER
---
6. In which of the following diseases is lens subluxation common?
A) Familial hypercholesterolemia
B) Marfan syndrome
C) Phenylketonuria
D) Huntington's disease
E) Cystic fibrosis
Answer: B
---
7. Which of the following statements is incorrect about Marfan syndrome?
A) It is autosomal dominant
B) There is a defect in elastic tissue and microfibrils
C) Lens subluxation is common
D) Floppy valve is common
E) The average lifespan is 10 years
Answer: E
---
8. In both homocystinuria and Marfan syndrome, arachnodactyly and lens
dislocation are seen. This is an example of which of the following?
A) Pleiotropy
B) Variable expressivity
C) Genetic heterogeneity
169 Pathology Questions and Answers, Prof.Dr. Murat ALPER
D) Penetrance
E) Nondisjunction
Answer: C
---
9. Which of the following diseases is autosomal dominant and results in
spontaneous rupture in the vascular and intestinal systems due to a defect in type III
collagen production?
A) Hunter syndrome
B) Hurler syndrome
C) Osteogenesis imperfecta
D) Ehlers-Danlos disease vascular type
E) Alport disease
Answer: D
---
10. Which of the following diseases does not exhibit autosomal recessive
inheritance?
A) Phenylketonuria
B) Sickle cell anemia
C) Von Willebrand disease
D) Congenital adrenal hyperplasia
E) Hemochromatosis
Answer: C
170 Pathology Questions and Answers, Prof.Dr. Murat ALPER
---
11. Which of the following is the most common cause of premature
atherosclerosis and death by the age of 15, inherited in an autosomal dominant
pattern?
A) Marfan syndrome
B) Familial hypercholesterolemia
C) Tay-Sachs disease
D) Cystic fibrosis
E) Mucopolysaccharidoses
Answer: B
---
12. Which of the following is incorrect regarding phenylketonuria?
A) It is autosomal recessive
B) There is a deficiency in phenylalanine hydroxylase
C) Symptoms can be prevented by diet
D) Mental retardation can develop
E) The mother's phenylalanine level has no effect on the fetus
Answer: E
---
13. Which of the following diseases is associated with Lisch nodules and is
autosomal dominant?
A) Li-Fraumeni syndrome
B) Von Hippel Lindau disease
171 Pathology Questions and Answers, Prof.Dr. Murat ALPER
C) Neurofibromatosis type I
D) Neuroblastoma
E) Marfan syndrome
Answer: C
---
14. In a 2-year-old child who died of chronic disease secondary to recurrent
infections, the autopsy shows dilatation in the ducts of the pancreas and salivary
glands, squamous metaplasia in the lining epithelial cells, fibrosis, hepatic steatosis,
and biliary cirrhosis. What is the most likely diagnosis?
A) Hemochromatosis
B) Cystic fibrosis
C) Tyrosinemia
D) Glycogen storage disease
E) Wilson disease
Answer: B
---
15. Which of the following statements about cystic fibrosis is incorrect?
A) Recurrent pulmonary infections
B) Pancreatic insufficiency
C) Impaired magnesium transport
D) High sweat NaCl levels
E) Occurs in 1 in 2500 live births
Answer: C
172 Pathology Questions and Answers, Prof.Dr. Murat ALPER
---
16. Which of the following is not seen in cystic fibrosis?
A) Meconium ileus
B) Malabsorption
C) Lung abscess
D) Infertility
E) Hypothyroidism
Answer: E
---
17. Painful muscle cramps and pain in the muscles after exercise are typical of
which of the following diseases?
A) Von Gierke disease
B) Pompe disease
C) Cori disease
D) McArdle disease
E) Galactosemia
Answer: D
---
18. Which of the following diseases is characterized by hepatomegaly,
renomegaly, and the development of hepatic adenomas in long-term survivors?
A) Cori disease
B) McArdle disease
173 Pathology Questions and Answers, Prof.Dr. Murat ALPER
C) Von Gierke disease
D) Gaucher disease
E) Pompe disease
Answer: C
---
19. In which of the following diseases can cardiomegaly, hypotonia in the
muscles, and splenomegaly lead to death before the age of three?
A) Cori disease
B) McArdle disease
C) Von Gierke disease
D) Gaucher disease
E) Pompe disease
Answer: E
---
20. Which of the following diseases is characterized by hepatosplenomegaly,
fever, and neurological disorders?
A) Hurler disease
B) Tay-Sachs disease
C) Niemann-Pick disease
D) Gaucher disease
E) Ehlers-Danlos type IX
Answer: C
174 Pathology Questions and Answers, Prof.Dr. Murat ALPER
---
21. Which of the following diseases is associated with gout and self-
mutilation?
A) Fabry disease
B) Hunter syndrome
C) Hurler syndrome
D) Lesch-Nyhan syndrome
E) Cori disease
Answer: D
---
22. Which of the following diseases is characterized by renal failure and
angiokeratomas?
A) Hunter syndrome
B) Gaucher disease
C) Von Gierke disease
D) Systemic lupus erythematosus
E) Fabry disease
Answer: E
---
23. Which of the following diseases is characterized by infantile cataracts,
mental retardation, and cirrhosis?
A) Alkaptonuria
B) Cori disease
175 Pathology Questions and Answers, Prof.Dr. Murat ALPER
C) Diabetes mellitus
D) Galactosemia
E) Pompe disease
Answer: D
---
24. Which of the following is not associated with galactosemia?
A) Hepatosplenomegaly
B) Galactose 1-phosphate uridyl transferase deficiency
C) Development of cirrhosis in a milk-fed infant
D) Development of Wilms tumor
E) Lens opacification
Answer: D
---
25. Which of the following does not exhibit autosomal recessive inheritance?
A) Phenylketonuria
B) Sickle cell anemia
C) Von Willebrand disease
D) Congenital adrenal hyperplasia
E) Hemochromatosis
Answer: C
---
176 Pathology Questions and Answers, Prof.Dr. Murat ALPER
26. Which of the following does not exhibit autosomal dominant inheritance?
A) Cystic fibrosis
B) Huntington's chorea
C) Acute intermittent porphyria
D) Achondroplasia
E) Hereditary spherocytosis
Answer: A
---
27. Which of the following diseases does not exhibit X-linked recessive
inheritance?
A) Duchenne muscular dystrophy
B) Chronic granulomatous disease
C) Diabetes insipidus
D) Vitamin D-resistant rickets
E) Fragile X syndrome
Answer: D
---
28. Which of the following is more valuable for distinguishing Hunter
syndrome from Hurler syndrome?
A) Accumulation of heparan sulfate
B) Accumulation of dermatan sulfate
C) X-linked recessive inheritance
D) Presence of corneal clouding
177 Pathology Questions and Answers, Prof.Dr. Murat ALPER
E) Milder clinical course
Answer: C
---
29. Which of the following causes skeletal deformities known as gargoylism?
A) Marfan syndrome
B) Ehlers-Danlos type I
C) Ehlers-Danlos type IV
D) Niemann-Pick syndrome
E) Hurler syndrome
Answer: E
---
30. Which of the following is not among the main symptoms of
mucopolysaccharidoses?
A) Hepatomegaly
B) Muscle weakness
C) Skeletal deformities
D) Coronary artery disease
E) Cardiac
valvular lesions
Answer: B
---
178 Pathology Questions and Answers, Prof.Dr. Murat ALPER
31. Which of the following is not a trinucleotide repeat disorder?
A) Fragile X syndrome
B) Huntington's disease
C) Friedreich's ataxia
D) Myotonic dystrophy
E) Prader-Willi syndrome
Answer: E
---
32. Which of the following is common in Fragile X syndrome?
A) Macroorchidism
B) Normal intelligence
C) Autosomal dominant inheritance
D) Fractures of long bones
E) Recurrent infections
Answer: A
---
33. Which of the following enzymes is deficient in essential fructosuria?
A) 1, 6 diphosphatase
B) Aldolase B
C) Phosphofructokinase
D) Phosphohexose isomerase
E) Fructokinase
179 Pathology Questions and Answers, Prof.Dr. Murat ALPER
Answer: E
---
34. In a 3-year-old child with hypoglycemia, hepatomegaly, and a baby face
appearance, which of the following should be considered first?
A) Von Gierke disease
B) Gaucher disease
C) Pompe disease
D) Sanfilippo disease
E) Tay-Sachs disease
Answer: A
---
35. In which of the following diseases is the affected gene involved in the
ubiquitin-proteasome proteolytic pathway, with the paternal chromosome being
defective?
A) Fragile X syndrome
B) Prader-Willi syndrome
C) Angelman syndrome
D) Turner syndrome
E) Klinefelter syndrome
Answer: C
---
180 Pathology Questions and Answers, Prof.Dr. Murat ALPER
36. In an obese patient with mental retardation, hypogonadism, and short
hands and feet, a deletion on chromosome 15 is identified. What is the most likely
diagnosis?
A) Fragile X syndrome
B) Prader-Willi syndrome
C) Angelman syndrome
D) Turner syndrome
E) Klinefelter syndrome
Answer: B
---
37. Congenital microcephaly seen at birth is not caused by which of the
following?
A) Cytomegalovirus
B) Herpes simplex virus
C) Gaucher disease
D) Maternal alcohol consumption
E) Rubella virus
Answer: C
---
38. Which of the following is associated with Gaucher disease?
A) Mucopolysaccharidoses
B) Cherry-red spot in the retina
C) Sphingomyelinase deficiency
D) Childhood death
181 Pathology Questions and Answers, Prof.Dr. Murat ALPER
E) Thrombocytopenia and often massive splenomegaly
Answer: E
---
39. In which of the following diseases is a clinical picture dominated by liver
enlargement and hypoglycemia seen?
A) Von Gierke disease
B) Hurler syndrome
C) Hunter syndrome
D) Gaucher disease
E) Niemann-Pick disease
Answer: A
---
40. Which genetic disease is associated with pathological fractures and bone
pain?
A) Von Gierke disease
B) Hurler syndrome
C) Hunter syndrome
D) Gaucher disease
E) Niemann-Pick disease
Answer: D
---
182 Pathology Questions and Answers, Prof.Dr. Murat ALPER
41. Which genetic disease is characterized by pancytopenia or
thrombocytopenia secondary to hypersplenism?
A) Von Gierke disease
B) Hurler syndrome
C) Hunter syndrome
D) Gaucher disease
E) Niemann-Pick disease
Answer: D
---
42. In a patient with cleft lip and palate, polydactyly, and cardiac and lung
defects, trisomy 13 is detected. What is the most likely diagnosis?
A) Down syndrome
B) Cri-du-chat syndrome
C) Patau syndrome
D) Edward syndrome
E) Klinefelter syndrome
Answer: C
---
43. A patient with 47 chromosomes who has a predisposition to leukemia and
develops Alzheimer's disease in middle age most likely has which of the following
diagnoses?
A) Cri-du-chat syndrome
B) Down syndrome
C) Edward syndrome
D) Patau syndrome
183 Pathology Questions and Answers, Prof.Dr. Murat ALPER
E) Turner syndrome
Answer: B
---
44. Which tumor is most commonly associated with Down syndrome in
children?
A) Retinoblastoma
B) Medulloblastoma
C) Dysgerminoma
D) Leukemia
E) Embryonal carcinoma
Answer: D
---
45. In which of the following diseases is endocardial cushion defect more
common?
A) Turner syndrome
B) Klinefelter syndrome
C) Down syndrome
D) Patau syndrome
E) Edward syndrome
Answer: C
---
184 Pathology Questions and Answers, Prof.Dr. Murat ALPER
46. Which of the following is the most prominent sign of Turner syndrome at
birth?
A) Short stature
B) Streak gonads
C) Delayed sexual development
D) Mental retardation
E) Edema of the hands and feet
Answer: E
---
47. What does a 47 XXY karyotype indicate?
A) Edward syndrome
B) Klinefelter syndrome
C) Patau syndrome
D) Turner syndrome
E) Prader-Willi syndrome
Answer: B
---
48. During fetal development, when is organogenesis (excluding the brain)
completed?
A) 4 weeks after ovulation
B) Between the 4th and 8th weeks
C) Between the 12th and 16th weeks
D) Between the 16th and 24th weeks
E) Between the 24th and 36th weeks
185 Pathology Questions and Answers, Prof.Dr. Murat ALPER
Answer: B
---
49. Where does fetal hematopoiesis initially begin?
A) Yolk sac
B) Liver
C) Lymph nodes
D) Heart
E) Bone marrow
Answer: A
---
50. Which of the following is not a type of hemoglobin produced by the fetus
and embryo?
A) Gower 1
B) Gower 2
C) Hemoglobin A
D) Hemoglobin F
E) Hemoglobin C
Answer: E
---
51. Which of the following sequences correctly describes the development
stages of the excretory system during the embryonic period?
186 Pathology Questions and Answers, Prof.Dr. Murat ALPER
A) Nephrogenic cord, pronephros, mesonephros
B) Pronephros, metanephros, mesonephros
C) Metanephros, pronephros, mesonephros
D) Pronephros, mesonephros, metanephros
E) Mesonephros, pronephros, metanephros
Answer: D
---
52. What is the effect of estrogens on the fetus-embryo?
A) Differentiates the genital tubercle towards female external genitalia
B) Initiates the development of the Müllerian duct
C) Causes the regression of Wolffian ducts
D) Modifies sweat glands to stimulate breast development
E) None of the above
Answer: E
---
53. Amniotic bands result from the rupture of the amnion and can wrap
around, compress, or attach to parts of the developing fetus. This is an example of
which of the following?
A) Malformation
B) Disruption
C) Deformation
D) Sequence
E) Agenesis
187 Pathology Questions and Answers, Prof.Dr. Murat ALPER
Answer: B
---
54. A ventricular septal defect caused by a teratogen would likely result from
the mother's exposure to the teratogen at which time?
A) 5th week
B) 8th week
C) 10th week
D) 13th week
E) 15th week
Answer: A
---
55. Which of the following best describes the structure of nucleic acids like
DNA and RNA?
A) They consist of a base, a phosphate group, and a sugar group
B) They are isomers of polypeptides and proteins
C) They contain two types of purines and two types of pyrimidines
D) They are composed of complementary double helices
E) They consist of a phosphate group, a 5-carbon sugar, and a nitrogenous
base
Answer: E
---
### REVIEW
188 Pathology Questions and Answers, Prof.Dr. Murat ALPER
(Review the topics below and write down what you remember...)
- Frequency of genetic diseases?
- Teratogens
- Marfan syndrome?
- Familial hypercholesterolemia
- Autosomal dominant diseases?
- Autosomal recessive diseases?
- X-linked diseases?
- Ehlers-Danlos syndrome?
- Lysosomal storage diseases?
- Mucopolysaccharidoses?
- Glycogen storage diseases?
- Alkaptonuria?
- Neurofibromatosis type I?
- Neurofibromatosis type II?
- Down syndrome?
- Edwards syndrome?
- Klinefelter syndrome?
- Patau syndrome
- XYY syndrome?
- Turner syndrome?
- Hermaphroditism?
- Pseudohermaphroditism?
- Trinucleotide repeat mutations?
- Fragile X syndrome?
- Prader-Willi syndrome?
- Angelman syndrome?
- Cri du chat syndrome?
189 Pathology Questions and Answers, Prof.Dr. Murat ALPER
- Fabry disease
- Hunter syndrome
- Hurler syndrome
- Lesch-Nyhan syndrome
- Cori disease
- Osteogenesis imperfecta
- Ehlers-Danlos vascular type
- Alport disease
- Cystic fibrosis
- Hemochromatosis
190 Pathology Questions and Answers, Prof.Dr. Murat ALPER
SECTION 9: ENVIROMENTAL PATHOLOGY
Here is the translation of the questions you provided:
---
1. Which of the following cancers is not associated with smoking?
A) Laryngeal cancer
B) Thymoma
C) Pancreatic cancer
D) Bladder cancer
E) Esophageal cancer
Answer: B
---
2. Which of the following statements is incorrect regarding Caplan syndrome?
A) It can be caused by coal dust.
B) It can be caused by silica.
C) Rheumatoid arthritis is seen.
D) Ankylosing spondylitis is seen.
E) It can be caused by asbestosis.
Answer: D
---
3. In which of the following pneumoconioses are granulomas seen?
A) Asbestosis
191 Pathology Questions and Answers, Prof.Dr. Murat ALPER
B) Anthracosis
C) Silicosis
D) Siderosis
E) Berylliosis
Answer: E
---
4. Which of the following pairings is incorrect?
A) Silicosis - Polarized particles in nodules
B) Berylliosis - Non-caseating granuloma
C) Asbestosis - Pleural calcification
D) Pneumoconiosis - Centrilobular emphysema
E) Berylliosis - Eggshell calcification
Answer: E
---
5. Which of the following tumors is most likely to occur in individuals working
in industries exposed to asbestos, such as shipbuilding?
A) Non-Hodgkin lymphoma
B) Malignant mesothelioma
C) Laryngeal cancer
D) Sinonasal carcinoma
E) Bladder cancer
Answer: B
192 Pathology Questions and Answers, Prof.Dr. Murat ALPER
---
6. Which of the following statements is incorrect regarding asbestosis?
A) Chrysotile is the most common cause of asbestosis.
B) Amphiboles are more commonly associated with mesothelioma.
C) Ferruginous bodies are seen.
D) Pleural calcification is a common finding.
E) It can lead to pancreatic carcinoma.
Answer: E
---
7. Which of the following substances is responsible for lung cancer in
individuals living near nuclear waste sites or nuclear factories?
A) Radon
B) Cadmium
C) Ozone
D) Nitrogen dioxide
E) Sulfur dioxide
Answer: A
---
8. Which of the following substances acts like a hapten, inducing cellular
immunity?
A) Nicotine
B) Beryllium
C) Formaldehyde
193 Pathology Questions and Answers, Prof.Dr. Murat ALPER
D) Silicon
E) Asbestos
Answer: B
---
9. Which of the following hormone pairings is incorrect?
A) Estrogen increases endometrial cancer.
B) Estrogen-progesterone increases endometrial cancer.
C) Oral contraceptives reduce ovarian cancer.
D) Estrogen increases breast cancer.
E) None of the above
Answer: B
---
10. Which of the following vitamin deficiencies can lead to severe brain
damage in chronic alcoholics?
A) Thiamine
B) Pyridoxine
C) Folic acid
D) Pantothenic acid
E) Cyanocobalamin
Answer: A
---
194 Pathology Questions and Answers, Prof.Dr. Murat ALPER
11. Which of the following disorders is associated with alcohol consumption?
A) CNS: Putamen necrosis
B) Gastrointestinal: Esophageal stricture
C) Hepatobiliary: Hemochromatosis
D) Gastrointestinal: Malabsorption
E) Hematological: Pernicious anemia
Answer: D
---
12. Which of the following is not directly caused by the toxic effects of chronic
alcoholism?
A) Alcoholic cirrhosis
B) Cerebral atrophy
C) Wernicke-Korsakoff syndrome
D) Congestive cardiomyopathy
E) Fatty liver change
Answer: C
---
13. Wernicke-Korsakoff syndrome is caused by the consumption of which
substance?
A) Ethanol
B) Methanol
C) Marijuana
D) Heroin
E) Acetone
195 Pathology Questions and Answers, Prof.Dr. Murat ALPER
Answer: A
---
14. Individuals working near high-voltage power lines or living in such areas
have an increased risk of developing which type of cancer?
A) Leukemia
B) Squamous cell carcinoma
C) Small cell lung carcinoma
D) Soft tissue sarcomas
E) Atrial myxoma
Answer: A
---
15. Which of the following tumors is not associated with the etiology of
radiotherapy?
A) Papillary thyroid carcinoma
B) Malignant fibrous histiocytoma
C) Chronic lymphocytic leukemia (CLL)
D) Squamous cell carcinoma
E) Leukemia
Answer: C
---
16. What is the most important target of ionizing radiation in a living cell?
196 Pathology Questions and Answers, Prof.Dr. Murat ALPER
A) Lipids in the cell membrane
B) DNA
C) Proteins in the cell membrane
D) Mitochondria
E) Ribosomes
Answer: B
---
19. Which type of radiation penetrates deeper into tissues?
A) Alpha
B) Beta
C) Proton
D) X-ray
E) Gamma
Answer: E
---
20. Which of the following is more harmful to human tissues when given in
equal doses?
A) Alpha
B) Beta
C) Proton
D) X-ray
E) Gamma
Answer: A
197 Pathology Questions and Answers, Prof.Dr. Murat ALPER
---
21. Which type of radiation causes the most damage to cells and has a high
Linear Energy Transfer (LET)?
A) X-ray
B) Gamma ray
C) Alpha particle
D) Beta particle
E) All cause equal damage
Answer: C
---
22. Which of the following is not a characteristic of neuroblastoma?
A) Produces catecholamines
B) Differentiates into a more mature form
C) Oncogene amplification
D) Most commonly seen in infancy and childhood
E) Most commonly located in the posterior cranial fossa
Answer: E
---
23. Which of the following tissues is most sensitive to ionizing radiation?
A) Bone marrow
B) Hepatocytes
C) Heart muscle
198 Pathology Questions and Answers, Prof.Dr. Murat ALPER
D) Nerve cells
E) Kidney
Answer: A
---
24. Which of the following pairings of chemical and effect is correct?
A) Acetaminophen - Hepatocellular carcinoma
B) Methyl alcohol - Kidney failure
C) Cocaine - Excessive lacrimation and salivation
D) Ethanol - CNS stimulant
E) CO - Changes in basal ganglia
Answer: E
---
25. Which of the following pairings of chemical and effect is directly correct?
A) Benzene: Liver necrosis
B) Lead: Encephalopathy in children
C) Ethylene glycol - Conversion to acetone
D) Unbalanced estrogen - Liver adenoma
E) Organophosphates - Mydriasis
Answer: B
---
199 Pathology Questions and Answers, Prof.Dr. Murat ALPER
26. Which of the following statements is incorrect?
A) Marasmus is a protein-energy malnutrition.
B) Kwashiorkor is primarily due to protein deficiency.
C) In marasmus, there is a loss of muscle and fat tissue.
D) Hepatosplenomegaly is prominent in kwashiorkor.
E) The edema in marasmus is more severe.
Answer: E
---
27. Isopropyl alcohol causes which of the following?
A) Hepatic angiosarcoma
B) Nasal cancer
C) Lung cancer
D) Bladder cancer
E) Leukemia
Answer: B
---
28. Which of the following is associated with marasmus rather than
kwashiorkor?
A) Decreased muscle mass and subcutaneous fat
B) Edema
C) Fatty liver
D) Hypoalbuminemia
E) Diarrhea
200 Pathology Questions and Answers, Prof.Dr. Murat ALPER
Answer: A
---
29. Which of the following cancers does not increase in obese men?
A) Esophagus
B) Thyroid
C) Colon
D) Kidney
E) Testis
Answer: B
---
30. Which of the following molecules, produced in adipose tissue and known
as the "guardian angel against obesity," burns fat?
A) Leptin
B) Adiponectin
C) Ghrelin
D) Cachectin
E) Cocaine
Answer: B
---
31. Which of the following cytokines affects obesity?
A) Leptin
B) Interleukin
201 Pathology Questions and Answers, Prof.Dr. Murat ALPER
C) Interleukin 12
D) Interferon gamma
E) TNF
Answer: A
---
32. Which of the following vitamins, when taken in high doses, causes
papilledema, headache, vomiting, and stupor?
A) Vitamin A
B) Cyanocobalamin
C) Vitamin D
D) Vitamin E
E) Ascorbic acid
Answer: A
---
33. Which of the following can cause microtia, anotia, facial bone and calvarial
anomalies, micrognathia, aortic arch anomalies, and thymus aplasia in the embryo?
A) Retinoic acid
B) Herpes virus
C) Folic acid
D) Vitamin E
E) Parvovirus B19
Answer: A
202 Pathology Questions and Answers, Prof.Dr. Murat ALPER
---
34. Which of the following diseases does not lead to vitamin B12 deficiency?
A) Crohn's disease
B) Celiac disease
C) Total gastrectomy
D) Infestation with Diphylloboth
rium latum
E) Colon tumor
Answer: E
---
35. Which of the following symptoms is not seen in pellagra?
A) Dermatitis
B) Diarrhea
C) Dementia
D) Peripheral neuropathy
E) Death
Answer: D
---
36. Which deficiency is most commonly seen in total parenteral nutrition?
A) Chromium
B) Copper
203 Pathology Questions and Answers, Prof.Dr. Murat ALPER
C) Manganese
D) Zinc
E) Selenium
Answer: D
---
37. Which of the following is expected in a deficiency of a water-soluble
vitamin?
A) Osteomalacia
B) Follicular hyperkeratosis
C) Congestive cardiomyopathy
D) Mottled teeth
E) Dysgeusia
Answer: C
---
38. Which of the following chemical agents causes necrosis in the bone
marrow, leukopenia, thrombocytopenia, anemia, and increased susceptibility to
infections, with some patients developing leukemia?
A) Carbon tetrachloride
B) Benzene
C) Halothane
D) Nitrous oxide
E) Nitrosamine
Answer: B
204 Pathology Questions and Answers, Prof.Dr. Murat ALPER
---
39. A patient with acrodermatitis enteropathica and poor wound healing is
likely deficient in which of the following?
A) Fluoride
B) Zinc
C) Copper
D) Magnesium
E) Sodium
Answer: B
---
40. Which of the following best explains the role of dietary fiber in preventing
colon cancer?
A) Increases stool transit time
B) Reduces the incidence of diverticulosis
C) Increases stool bulk
D) Increases excretion of lithocholic acid
E) Reduces reabsorption of estrogen
Answer: D
---
41. A 40-year-old patient from Kütahya, being treated for acute
bronchopneumonia, mentions that he works in a ceramic factory. What should be the
next step?
A) Ask if there is asbestos in the factory
205 Pathology Questions and Answers, Prof.Dr. Murat ALPER
B) Ask if he previously worked in a coal mine
C) Ask about steroid use
D) Ask about tuberculosis medication use
E) Perform a skin test for beryllium
Answer: E
---
42. A 40-year-old male farmer presents to the emergency department with
acute pulmonary edema. He has no other complaints, and his chest X-ray shows no
cardiac enlargement. What is the most likely diagnosis?
A) Chronic bronchitis due to dust exposure
B) Silo filler's disease
C) Asbestosis
D) Hypersensitivity to resins
E) Berylliosis
Answer: B
---
43. Which tumor is likely to develop in the daughter of a woman who used
diethylstilbestrol during pregnancy?
A) Squamous cell carcinoma
B) Basosquamous carcinoma
C) Clear cell carcinoma
D) Botryoid rhabdomyosarcoma
E) Bowen's disease
206 Pathology Questions and Answers, Prof.Dr. Murat ALPER
Answer: C
---
44. Nickel is an etiological agent in the cancer of which organ?
A) Skin
B) Liver
C) Nasal sinuses
D) Prostate
E) Pancreas
Answer: C
---
45. Which of the following causes large eosinophilic droplets in renal tubular
epithelial cells and colicky abdominal pain?
A) Arsenic
B) Cadmium
C) Lead
D) Nitrite
E) Uranium
Answer: C
---
46. In which of the following conditions is increased urinary excretion of
aminolevulinic acid observed?
A) Lead poisoning
207 Pathology Questions and Answers, Prof.Dr. Murat ALPER
B) Pancreatic carcinoma
C) Chronic pyelonephritis
D) Vitamin C intoxication
E) Ulcerative colitis
Answer: A
---
47. Which chemical agent causes severe hypoxic injury with "cherry-red"
discoloration of the skin, blood, and internal organs?
A) Methyl alcohol
B) Carbon tetrachloride
C) Carbon monoxide
D) Mercury
E) Cyanide
Answer: C
---
48. Which chemical agent causes gastrointestinal ulceration, renal tubular
necrosis, and calcification?
A) Lead
B) Cyanide
C) Mercury
D) Methyl alcohol
E) 1,3-Butadiene
Answer: C
208 Pathology Questions and Answers, Prof.Dr. Murat ALPER
---
49. Which of the following substances causes nasal ulceration, septal
perforation, and fatal arrhythmias?
A) Methyl alcohol
B) Carbon monoxide
C) Marijuana
D) Cocaine
E) Lead
Answer: D
---
50. Which vitamin deficiency may manifest as confabulation?
A) Riboflavin
B) Thiamine
C) Niacin
D) Folic acid
E) Ascorbic acid
Answer: B
---
51. Which of the following vitamin deficiencies is characterized by glossitis,
corneal vascularization, cheilosis, and seborrheic dermatitis?
A) Riboflavin
B) Vitamin D
209 Pathology Questions and Answers, Prof.Dr. Murat ALPER
C) Vitamin E
D) Folic acid
E) Ascorbic acid
Answer: A
---
52. Which of the following conditions is associated with subperiosteal
hemorrhage, bleeding gums, and cutaneous perifollicular hemorrhage?
A) Beriberi
B) Keratomalacia
C) Pellagra
D) Rickets
E) Scurvy
Answer: E
---
53. A 70-year-old widower who has been surviving on a diet of tea and toast
complains of gum bleeding while brushing his teeth. The patient also has a red
tongue, decayed teeth, and inflamed gums. What is the most likely mechanism of
these findings?
A) ATP deficiency
B) Cofactor deficiency for collagenase
C) Deficiency in lysine and proline hydroxylation
D) Cofactor deficiency for lysyl oxidase
E) Cofactor deficiency for pyruvate dehydrogenase
Answer: C
210 Pathology Questions and Answers, Prof.Dr. Murat ALPER
---
54. Which of the following vitamins cannot be synthesized endogenously?
A) Vitamin D
B) Biotin
C) Vitamin C
D) Vitamin K
E) Niacin
Answer: C
---
55. In which vitamin deficiency is spinocerebellar degeneration seen?
A) Vitamin A deficiency
B) Vitamin D deficiency
C) Vitamin E deficiency
D) Vitamin K deficiency
E) Thiamine deficiency
Answer: C
---
56. A worker at a plastic pipe manufacturing plant develops a mass in the liver.
A biopsy reveals a spindle cell neoplasm, immunohistochemically positive for
vimentin and negative for cytokeratin. Which of the following substances could be
implicated in the development of this neoplasm?
A) Radon
211 Pathology Questions and Answers, Prof.Dr. Murat ALPER
B) Benzene
C) Cyclophosphamide
D) Vinyl chloride
E) Asbestos
Answer: D
---
57. Which of the following carcinogens is associated with angiosarcoma?
A) Vinyl chloride
B) Nickel
C) Naphthylamine
D) Benzene
E) Cadmium
Answer: A
---
58. Which of the following is not a side effect of vincristine, used in the
treatment of germ cell tumors?
A) Muscle weakness
B) Acute left ventricular failure
C) Paralytic ileus
D) Neuropathy
E) Inappropriate ADH secretion
Answer: B
212 Pathology Questions and Answers, Prof.Dr. Murat ALPER
---
59. Which vitamin is synthesized with the involvement of the skin, liver, and
kidneys?
A) Vitamin A
B) Vitamin B12
C) Vitamin K
D) Vitamin D
E) Vitamin E
Answer: D
---
60. Which of the following may prevent the development of atheroma?
A) Vitamin A
B) Vitamin B12
C) Vitamin K
D) Vitamin E
E) Biotin
Answer: D
---
61. Which vitamin, when taken in high doses over a long period, causes
gonadal dysfunction, creatinuria, and gastrointestinal disturbances?
A) Vitamin A
B) Vitamin B
C) Vitamin C
213 Pathology Questions and Answers, Prof.Dr. Murat ALPER
D) Vitamin D
E) Vitamin E
Answer: E
---
62. Which of the following conditions is not associated with vitamin A
hypovitaminosis?
A) Pseudomembranous colitis
B) Chronic pancreatitis
C) Whipple's disease
D) Celiac sprue
E) Abetalipoproteinemia
Answer: A
---
Review Topics:
- Tumors associated with radiotherapy
- Substances found in cigarette smoke
- Caplan syndrome
- Asbestos
- Silicosis
- Effects of estrogen
- Effects of alcohol
- Effects of ionizing radiation
- Types of radiation
214 Pathology Questions and Answers, Prof.Dr. Murat ALPER
- Cellular sensitivity to radiation
- Kwashiorkor
- Marasmus
- Vitamin deficiencies in total parenteral nutrition
- Hypervitaminosis and symptoms
- Hypovitaminosis and symptoms
- Benzene
- Zinc
- Beryllium
- Diethylstilbestrol
- Lead
- Mercury
- Carbon monoxide
- Cocaine
- Heroin
- Thorotrast
- Retinoic acid
- Parvovirus
215 Pathology Questions and Answers, Prof.Dr. Murat ALPER
SECTION 10: INFECTIOUS DISEASE
Here is the translation of the questions and review content you provided:
---
1. Which of the following microorganisms spreads through erythrocytes?
A) Babesia
B) Herpes virus
C) Mycobacterium
D) Leishmania
E) Toxoplasma
Answer: A
Hepatitis B virus, poliovirus, most bacteria, fungi, trypanosomes, and all
helminths are transported freely in plasma. Herpes virus, HIV, CMV, Mycobacterium,
Leishmania, and Toxoplasma are carried within leukocytes. Plasmodium and Babesia
are carried by erythrocytes.
---
2. Which of the following viruses can remain latent in the dorsal root ganglia
for a long time?
A) Cytomegalovirus
B) Epstein-Barr virus
C) Herpes Zoster
D) Human papillomavirus
E) Reovirus
Answer: C
216 Pathology Questions and Answers, Prof.Dr. Murat ALPER
DNA viruses like Herpes viruses have a double-stranded DNA genome and
large capsules, with eight types. The alpha group (HSV-1, HSV-2, Varicella Zoster virus)
infects epithelial cells and establishes latent infection in neurons. Varicella Zoster
virus, which causes chickenpox, remains latent in the dorsal root ganglia and can later
cause shingles (Herpes Zoster).
3. Which of the following viruses forms inclusion bodies?
A) Epstein-Barr virus
B) Human immunodeficiency virus
C) Cytomegalovirus
D) Poliovirus
E) Rotavirus
Answer: C
CMV characteristically forms large basophilic inclusion bodies resembling
"owl's eyes" in both the nucleus and cytoplasm, enlarging the cell. Rabies and
smallpox viruses also cause cytoplasmic inclusions, but most viruses do not. CMV
inclusion bodies are easily seen in urinary epithelial cells, especially in babies
suspected of CMV infection.
4. Which microorganism is an obligate intracellular organism and cannot
synthesize ATP?
A) Chlamydia
B) Mycoplasma
C) Trichophyton
D) Vibrio
E) Prion
Answer: A
Chlamydia and Rickettsia are obligate intracellular organisms. They have
limited metabolic abilities, such as the inability to synthesize ATP. Mycoplasmas,
which lack a cell wall, are the smallest free-living bacteria.
217 Pathology Questions and Answers, Prof.Dr. Murat ALPER
---
5. Which of the following hosts is responsible for transmitting Borrelia
burgdorferi, the causative agent of Lyme disease?
A) Lice
B) Ticks
C) Fleas
D) Mites
E) Rats
Answer: B
Borrelia burgdorferi, transmitted by ticks, causes Lyme disease, which has
three stages: erythema chronicum migrans, hematogenous dissemination leading to
lymphadenopathy, joint, muscle, and nerve involvement, pericarditis, myocarditis,
and meningoencephalitis, followed by destructive chronic arthritis and encephalitis in
the third stage.
---
6. Which of the following does not play a primary role in the gastrointestinal
defense against microbial invasion?
A) Mucus covering epithelial cells
B) Acid in gastric fluid
C) Pancreatic enzymes
D) Detergent action of bile salts
E) IgG
Answer: E
The primary defense mechanisms in the gastrointestinal system include gastric
acidity, mucus covering the epithelium, pancreatic enzymes, bile salts, and secreted
antibodies (IgA). IgG is involved after bacterial invasion of tissue.
218 Pathology Questions and Answers, Prof.Dr. Murat ALPER
---
7. Which pathogen causes damage without colonizing or multiplying in the
intestinal wall by releasing toxins?
A) Campylobacter
B) Mycobacterium
C) Salmonella
D) Vibrio cholerae
E) Clostridium botulinum
Answer: E
Food poisoning caused by Clostridium botulinum and some Staphylococcus
species occurs when bacteria release toxins without colonizing the intestinal wall.
Vibrio cholerae and enterotoxigenic E. coli adhere to the epithelium with flagella but
cause pathology mainly through extensive watery diarrhea.
---
8. Which organism has a mechanism of action involving cell membrane lysis
and cell depolarization?
A) Giardia
B) Ascaris
C) Trichinella
D) Entamoeba
E) Wuchereria
Answer: D
Entamoeba histolytica secretes a lectin that forms pores in the target
epithelial cell plasma membrane, causing cytolysis and cell depolarization, similar to
the action of cytotoxic T lymphocytes. This organism most commonly affects the
cecum and ascending colon, causing dysentery and liver abscesses.
219 Pathology Questions and Answers, Prof.Dr. Murat ALPER
---
9. Which pathogen can be transmitted from mother to fetus, causing severe
fetal osteochondritis and periostitis?
A) Toxoplasma
B) Rubella
C) Treponema
D) Chlamydia
E) Cytomegalovirus
Answer: C
Rubella can cause congenital heart disease, intellectual disability, cataracts,
and deafness if contracted during the first trimester. Treponema pallidum, the
causative agent of syphilis, can invade the fetus in the second trimester, causing
severe fetal osteochondritis and periostitis.
---
10. Which condition presents with generalized lymphadenopathy, sore throat,
atypical lymphocytes in peripheral blood, and a high antibody titer in the Paul-Bunnell
test?
A) Burkitt lymphoma
B) Chronic nonspecific lymphadenitis
C) Infectious mononucleosis
D) Sarcoidosis
E) Tuberculosis
Answer: C
These findings are characteristic of infectious mononucleosis caused by
Epstein-Barr virus (EBV). The virus infects B cells via the CD21 receptor and can also
cause Burkitt lymphoma and nasopharyngeal carcinoma.
220 Pathology Questions and Answers, Prof.Dr. Murat ALPER
---
11. Which of the following is a deep and extensive infection affecting multiple
follicles?
A) Furuncle
B) Carbuncle
C) Folliculitis
D) Hidradenitis suppurativa
E) Paronychia
Answer: B
A carbuncle is a deep and extensive infection affecting multiple hair follicles,
often caused by Staphylococcus aureus. It presents with painful, erythematous,
infiltrated nodular lesions, usually on the neck, back, and buttocks.
---
12. Which of the following is the name given to an acute bacterial infection of
the nail folds and surrounding tissue?
A) Furuncle
B) Carbuncle
C) Folliculitis
D) Hidradenitis suppurativa
E) Paronychia
Answer: E
Bacterial paronychia (or "dolama") is an acute bacterial infection of the nail
folds, usually caused by staphylococci. It is associated with predisposing factors like
trauma or prolonged wetting of the hands.
---
221 Pathology Questions and Answers, Prof.Dr. Murat ALPER
13. Which of the following is an acute infection of the lower dermis and
subcutaneous fat tissue?
A) Furuncle
B) Carbuncle
C) Folliculitis
D) Hidradenitis suppurativa
E) Cellulitis
Answer: E
Cellulitis is an acute infection of the lower dermis and subcutaneous fat tissue,
usually caused by group A streptococci or Staphylococcus aureus. It presents with
painful, erythematous, and swollen skin.
---
14. Which of the following is a multiple abscess formation in sweat glands?
A) Furuncle
B) Carbuncle
C) Folliculitis
D) Hidradenitis suppurativa
E) Cellulitis
Answer: D
Hidradenitis suppurativa is a chronic inflammatory condition affecting the
apocrine sweat glands, leading to painful nodules, abscesses, and scarring, often in
the armpits, groin, and under the breasts.
---
15. Which of the following pathogens causes Ritter disease?
222 Pathology Questions and Answers, Prof.Dr. Murat ALPER
A) Staphylococcus
B) Streptococcus
C) Meningococcus
D) Candida
E) Histoplasmosis
Answer: A
Ritter disease, also known as Staphylococcal scalded skin syndrome, is caused
by exfoliative toxins produced by Staphylococcus aureus, leading to widespread skin
desquamation.
---
16. Which of the following pathogens clusters and forms plaques leading to
dental caries?
A) S. aureus
B) S. viridans
C) S. mutans
D) S. pneumoniae
E) S. agalactiae
Answer: C
Streptococcus mutans is the primary bacterium responsible for dental caries.
It forms biofilms (dental plaque) on tooth surfaces and produces acid from sugar
metabolism, leading to tooth decay.
---
17. Which pathogen causes abscesses and phlegmon in the oral, retro-
pharyngeal, or even brain and lungs, either alone or with anaerobic organisms?
A) Streptococcus faecalis
223 Pathology Questions and Answers, Prof.Dr. Murat ALPER
B) Bacteroides fragilis
C) Propionibacterium
D) Prevotella
E) Bacteroides melaninogenicus
Answer: D
Prevotella is a Gram-negative rod that causes abscesses and phlegmon above
the diaphragm, particularly in the oral cavity, retro-ph
aryngeal area, brain, and lungs.
---
18. Which of the following diseases is NOT caused by Chlamydia trachomatis?
A) Urethritis
B) Arthritis
C) Inclusion conjunctivitis
D) Neonatal pneumonia
E) Ornithosis
Answer: E
Ornithosis is caused by Chlamydia psittaci, not C. trachomatis. C. trachomatis
causes diseases such as nongonococcal urethritis, Reiter's syndrome (arthritis),
inclusion conjunctivitis, and neonatal pneumonia.
---
19. Which of the following organisms causes well-defined necrotic and
hemorrhagic skin lesions known as ecthyma gangrenosum?
A) Legionella pneumonia
B) Staphylococcus aureus
224 Pathology Questions and Answers, Prof.Dr. Murat ALPER
C) Streptococcus pneumoniae
D) Cytomegalovirus
E) Pseudomonas aeruginosa
Answer: E
Pseudomonas aeruginosa, an opportunistic pathogen, can cause ecthyma
gangrenosum, especially in immunocompromised patients. It leads to necrotic skin
lesions due to vascular invasion and bacterial sepsis.
---
20. A 20-year-old woman presents with pyuria and microscopic hematuria, but
routine urine cultures are negative. What is the most likely diagnosis?
A) Bladder tumor
B) Chronic cystitis
C) Tuberculosis
D) Acute cystitis
E) Kidney stone
Answer: C
Genitourinary tuberculosis can cause pyuria and hematuria, but routine urine
cultures are usually negative. Diagnosis is made with specialized cultures or biopsy.
---
21. Which lesion is seen in the first stage of syphilis?
A) Aortic regurgitation
B) Tabes dorsalis
C) Chancre
D) Gumma
E) Condyloma lata
225 Pathology Questions and Answers, Prof.Dr. Murat ALPER
Answer: C
The primary lesion of syphilis is the chancre, a painless ulcer that appears at
the site of infection, typically on the genitals.
---
22. Which lesion is associated with the second stage of syphilis?
A) Gumma
B) Aortitis
C) Condylomata lata
D) Tabes dorsalis
E) Chancre
Answer: C
Condylomata lata are wart-like lesions that occur during the secondary stage
of syphilis, typically found in the genital and oral regions.
---
23. Which lesion can be found in all stages of syphilis?
A) Condylomata lata
B) Vasculitis
C) Chancre
D) Gumma
E) Tabes dorsalis
Answer: B
Syphilitic vasculitis, characterized by lymphocytes and plasma cells around
small blood vessels, can be seen in all stages of syphilis.
226 Pathology Questions and Answers, Prof.Dr. Murat ALPER
---
24. Which pathogen causes osteochondritis, periostitis, and keratitis in a
newborn?
A) C. trachomatis
B) CMV
C) HBV
D) Toxoplasma
E) T. pallidum
Answer: E
Treponema pallidum, the causative agent of syphilis, can cross the placenta
after the 4th month of pregnancy, causing congenital syphilis, which includes
periostitis, osteochondritis, and interstitial keratitis.
---
25. Which organism does not cause bloody diarrhea?
A) Yersinia
B) V. cholerae
C) C. jejuni
D) E. histolytica
E) Shigella
Answer: B
Vibrio cholerae causes severe watery diarrhea (rice-water stools) by increasing
cAMP levels, but it does not cause bloody diarrhea or involve neutrophils in the stool.
---
227 Pathology Questions and Answers, Prof.Dr. Murat ALPER
26. Which ciliated parasite, transmitted from pigs, causes dysentery?
A) E. histolytica
B) Trichuris trichiura
C) Balantidium coli
D) Giardia intestinalis
E) Ascaris lumbricoides
Answer: C
Balantidium coli is a ciliated parasite that can cause colitis and dysentery,
typically transmitted from pigs.
---
27. Which of the following is NOT seen in congenital toxoplasmosis?
A) Hydrocephalus
B) Chorioretinitis
C) Intracranial calcifications
D) Osteochondritis
E) Maculopapular rash
Answer: D
Osteochondritis is more typical of congenital syphilis. Congenital
toxoplasmosis is associated with hydrocephalus, chorioretinitis, intracranial
calcifications, and maculopapular rash.
---
28. A 7-month-old healthy girl suddenly develops poor feeding, constipation,
lethargy, and general weakness. The history reveals she has been fed honey mixed
with biscuits for some time. What is the most likely diagnosis?
A) Hypothyroidism
228 Pathology Questions and Answers, Prof.Dr. Murat ALPER
B) Poliomyelitis
C) Botulism
D) Mercury poisoning
E) Hyperglycemia
Answer: C
Clostridium botulinum spores, commonly found in honey, produce a toxin in
the infant's gastrointestinal tract, leading to symptoms of botulism.
---
29. A biopsy from vesicles in the genital area of a 30-year-old woman shows
purple inclusions in the nuclei of cells. What is the most likely diagnosis?
A) Cytomegalovirus infection
B) Syphilis
C) Chlamydia infection
D) Genital herpes
E) Mycoplasma infection
Answer: D
Herpes genitalis is a common sexually transmitted infection caused by HSV-1
or HSV-2. It presents with painful vesicles and intranuclear inclusions (Cowdry type A).
---
30. Which condition is NOT caused by a microorganism that requires
immediate culturing at the bedside for growth?
A) Conjunctivitis
B) Salpingitis
C) Urethritis
D) Bartholinitis
229 Pathology Questions and Answers, Prof.Dr. Murat ALPER
E) Penile ulcer
Answer: E
Neisseria gonorrhoeae, which requires immediate culturing at the bedside for
growth, does not cause penile ulcers. Penile ulcers are often caused by Treponema
pallidum (syphilis) or herpes simplex virus.
---
31. Acute gonococcal infection in women typically causes inflammation in
which of the following locations?
A) Vulva
B) Vagina
C) Endocervix
D) Ovary
E) Parametrium
Answer: C
Gonococcal infection typically affects the transitional epithelium of the urethra
and the columnar epithelium of the endocervix, leading to pelvic inflammatory
disease if untreated.
---
32. A 33-year-old woman with an ulcerated genital lesion and inguinal
lymphadenopathy has a biopsy showing suppurative necrotizing granulomatous
inflammation with a star-like appearance. What is the most likely pathogen?
A) Brucella
B) Rickettsia
C) Mycoplasma
D) Mycobacterium
E) Chlamydia
230 Pathology Questions and Answers, Prof.Dr. Murat ALPER
Answer: E
The described findings are typical of lymphogranuloma venereum, a sexually
transmitted disease caused by Chlamydia trachomatis.
---
33. In which disease is pannus formation in the eye observed?
A) Glaucoma
B) Cataract
C) Conjunctivitis
D) Trachoma
E) Wilson's disease
Answer: D
Trachoma, caused by Chlamydia trachomatis, leads to pannus formation,
which is a vascularization of the cornea, causing scarring and blindness if untreated.
---
34. Which of the following diseases is NOT caused by Mycoplasma?
A) Prostatitis
B) Upper respiratory infection
C) Primary atypical pneumonia
D) Urethritis
E) Dendritic keratoconjunctivitis
Answer: E
Dendritic keratoconjunctivitis is caused by herpes simplex virus type 1, not by
Mycoplasma. Mycoplasma causes primary atypical pneumonia and can be involved in
other conditions like urethritis and prostatitis.
231 Pathology Questions and Answers, Prof.Dr. Murat ALPER
---
35. Which organism depolarizes its cellular target and creates a pore protein,
causing cytolysis similar to cytotoxic T lymphocytes?
A) E. histolytica
B) G. lamblia
C) E. granulosus
D) C. granulomatosis
E) T. cruzi
Answer: A
Entamoeba histolytica secretes a pore-forming lectin that depolarizes the
target cell membrane, leading to cytolysis and forming ulcers in the colon mucosa.
---
36. A 35-year-old butcher develops a black pustule on his hand containing
Gram-positive bacteria and a few neutrophils. What is the most likely diagnosis?
A) Staphylococcal infection
B) Erysipelas
C) Leishmaniasis
D) Cellulitis
E) Anthrax
Answer: E
Anthrax, caused by Bacillus anthracis, often presents with a black eschar and
can be found in people working with animals such as goats, sheep, and cattle.
---
232 Pathology Questions and Answers, Prof.Dr. Murat ALPER
37. Which pathogen, producing Shiga-like toxins I and II, can
cause necrotizing enterocolitis in neutropenic patients, sometimes leading to
an epidemic?
A) Enteropathogenic E. coli
B) Enterohemorrhagic E. coli
C) Enterotoxigenic E. coli
D) E. coli O157:H7
E) Enteroinvasive E. coli
Answer: B
Enterohemorrhagic E. coli (EHEC) produces Shiga-like toxins I and II and can
cause hemolytic-uremic syndrome (HUS), especially in children and the elderly, with a
high mortality rate.
---
38. Which organism is most commonly associated with neutropenic
enterocolitis?
A) Enteropathogenic E. coli
B) Enterotoxigenic E. coli
C) Clostridium difficile
D) Clostridium septicum
E) Shigella dysenteriae type I
Answer: D
Neutropenic enterocolitis, also known as typhlitis, commonly occurs in
patients undergoing cancer chemotherapy. The most frequent pathogen is
Clostridium septicum, though CMV can also be involved.
---
233 Pathology Questions and Answers, Prof.Dr. Murat ALPER
39. What is the correct definition of cellulitis?
A) Fibrin mixed with neutrophils
B) Intense pus accumulation
C) Fluid accumulation in acute inflammation
D) Surface epithelium covered with fibrinopurulent exudate
E) Diffuse acute inflammation spreading through tissues
Answer: E
Cellulitis is characterized by diffuse acute inflammation that spreads through
tissues, usually caused by Streptococcus pyogenes or Staphylococcus aureus.
---
40. Which lung lesion is caused by Mycoplasma pneumoniae?
A) Interstitial pneumonia
B) Patchy acute inflammation
C) Lobar consolidation
D) Granulomas
E) Abscess formation
Answer: A
Mycoplasma pneumoniae typically causes interstitial pneumonia, which
affects the alveolar walls rather than the air spaces.
---
41. Which pathogen does NOT cause interstitial pneumonia?
A) Orthomyxovirus
B) Adenovirus
234 Pathology Questions and Answers, Prof.Dr. Murat ALPER
C) Chlamydia
D) Mycoplasma pneumoniae
E) S. pneumoniae
Answer: E
Streptococcus pneumoniae causes lobar or lobular pneumonia, not interstitial
pneumonia.
---
42. Which cell infiltration is observed in lung infection caused by Legionella
pneumophila?
A) Lymphocytes
B) Plasma cells
C) Macrophages
D) Basophils
E) Eosinophils
Answer: C
Legionella infection primarily stimulates a macrophage response, with early
lesions often containing phagocytosed organisms.
---
43. Which disease is NOT associated with Coxsackie B virus?
A) Herpangina
B) Epidemic myalgia
C) Pleurodynia
D) Myocarditis
E) Neonatal hepatitis
235 Pathology Questions and Answers, Prof.Dr. Murat ALPER
Answer: A
Herpangina, hand-foot-and-mouth disease, and hemorrhagic conjunctivitis are
caused by Coxsackie A virus. Coxsackie B virus causes conditions like epidemic
myalgia, pleurodynia, and neonatal hepatitis.
---
44. In which disease are Warthin-Finkeldey cells observed?
A) Scarlet fever
B) Measles
C) Rubella
D) Shingles
E) Toxoplasmosis
Answer: B
Warthin-Finkeldey multinucleated giant cells are characteristic of measles.
Measles can lead to complications like giant cell pneumonia, otitis media, and
encephalitis.
---
45. Which organism is the most common cause of croup?
A) Parainfluenza virus
B) H. influenzae type B
C) Chlamydia trachomatis
D) Respiratory syncytial virus
E) S. pneumoniae
Answer: A
236 Pathology Questions and Answers, Prof.Dr. Murat ALPER
Croup (laryngotracheobronchitis) is most commonly caused by parainfluenza
virus. Other causes include adenovirus, respiratory syncytial virus, and influenza virus.
---
46. Which disease is thought to be associated with measles virus?
A) Creutzfeldt-Jakob disease
B) Guillain-Barré syndrome
C) Kuru
D) Progressive multifocal leukoencephalopathy
E) Subacute sclerosing panencephalitis
Answer: E
Subacute sclerosing panencephalitis (SSPE) is a slow viral infection thought to
be caused by a persistent defective measles virus, lacking the M component
necessary for extracellular spread.
---
47. Which pair of rash diseases is caused by the same virus?
A) Rubella-Roseola
B) Rubella-Rubeola
C) Variola-Herpes simplex
D) Herpes simplex-Varicella
E) Herpes zoster-Varicella
Answer: E
Both herpes zoster (shingles) and varicella (chickenpox) are caused by the
Varicella-Zoster virus.
---
237 Pathology Questions and Answers, Prof.Dr. Murat ALPER
48. Which virus can cause necrotizing bronchitis and diffuse hemorrhagic
necrotizing pneumonia in adults, and sometimes lead to epidemics?
A) Rhinovirus
B) RSV
C) Rotavirus
D) Influenza A virus
E) Norwalk virus
Answer: D
Influenza A virus is the most common cause of viral pneumonia in adults and
can cause necrotizing bronchitis and pneumonia, sometimes leading to epidemics.
---
49. Which is the most common intrauterine infection?
A) Hepatitis
B) Toxoplasma
C) CMV
D) Rubella
E) Listeria monocytogenes
Answer: C
Cytomegalovirus (CMV) is the most common cause of intrauterine infections,
potentially leading to conditions like microcephaly, hearing loss, hepatosplenomegaly,
and intracranial calcifications.
---
50. A 30-year-old man develops a blister, ulcer, and lymphadenopathy after a
thorn prick while picking roses. What is the most likely pathogen?
238 Pathology Questions and Answers, Prof.Dr. Murat ALPER
A) Histoplasma capsulatum
B) Cryptococcus neoformans
C) Aspergillus flavus
D) Candida albicans
E) Sporothrix schenckii
Answer: E
Sporothrix schenckii, the causative agent of sporotrichosis (rose gardener's
disease), causes skin lesions and lymphadenopathy after exposure to contaminated
plant material.
---
51. Which parasite causes punctate keratitis, blindness, and subcutaneous
nodules?
A) Onchocerca volvulus
B) Schistosoma haematobium
C) Schistosoma mansoni
D) Leishmania donovani
E) Acanthamoeba
Answer: A
Onchocerca volvulus, the causative agent of river blindness, leads to skin
nodules, punctate keratitis, and chronic pruritic dermatitis, with potential blindness
due to sclerosis keratitis.
---
52. Which organism does NOT affect the liver?
A) Clonorchis sinensis
B) Echinococcus granulosus
239 Pathology Questions and Answers, Prof.Dr. Murat ALPER
C) Schistosoma haematobium
D) Schistosoma mansoni
E) Schistosoma japonicum
Answer: C
Schistosoma haematobium primarily affects the urinary bladder and does not
typically involve the liver, unlike the other organisms listed.
---
53. Which statement is incorrect regarding Schistosoma mansoni infestation?
A) It is a common cause of hematuria.
B) Its eggs are excreted in feces.
C) Its cycle involves a freshwater snail.
D) Liver fibrosis can develop.
E) It is transmitted through contact with contaminated water.
Answer: A
Schistosoma mansoni primarily affects the gastrointestinal system and liver,
not the urinary bladder. Hematuria is more commonly associated with Schistosoma
haematobium.
---
54. In which organ does Echinococcus granulosus most commonly form
lesions?
A) Brain
B) Kidney
C) Spleen
D) Liver
E) Lung
240 Pathology Questions and Answers, Prof.Dr. Murat ALPER
Answer: D
Echinococcus granulosus most commonly forms hydatid cysts in the liver
(about 70%), followed by the lungs and other organs.
---
55. Which parasite causes lymphatic filariasis similar to Wuchereria bancrofti?
A) Brugia malayi
B) Onchocerca volvulus
C) Cysticercosis
D) Trichinella spiralis
E) Diphyllobothrium latum
Answer: A
Brugia malayi, like Wuchereria bancrofti, causes lymphatic filariasis, leading to
lymphatic obstruction and swelling, particularly in the legs and scrotum.
---
56. Which of the following ectoparasites does not have an intermediate host?
A) Clonorchis sinensis
B) Loa loa
C) Trichuris trichiura
D) Paragonimus westermani
E) Dracunculus medinensis
Answer: C
Trichuris trichiura (whipworm) does not require an intermediate host. It
directly infects humans when eggs are ingested.
241 Pathology Questions and Answers, Prof.Dr. Murat ALPER
---
57. Which is the only ciliated pathogen that causes disease in humans
?
A) Giardia lamblia
B) Trichomonas vaginalis
C) Balantidium coli
D) Chilomastix mesnili
E) Acanthamoeba
Answer: C
Balantidium coli is the only ciliated pathogen that causes disease in humans,
leading to dysentery, typically transmitted from pigs.
---
58. Which parasite has a chitin wall and four nuclei, making it resistant to
gastric acid?
A) Entamoeba histolytica
B) Giardia lamblia
C) Echinococcus granulosus
D) Vibrio cholerae
E) Trypanosoma cruzi
Answer: A
Entamoeba histolytica cysts have a chitin wall and four nuclei, making them
resistant to gastric acid.
242 Pathology Questions and Answers, Prof.Dr. Murat ALPER
---
59. Hemozoin pigment is seen in diseases caused by which of the following
organisms?
A) Plasmodium
B) Cryptococcus
C) Blastomyces
D) Histoplasmosis
E) Aspergillus
Answer: A
In malaria, caused by Plasmodium species, hemozoin pigment accumulates in
the spleen and liver as a byproduct of hemoglobin digestion by the parasite.
---
60. Which organism, with a double-contoured capsule, causes granulomatous
infection in the skin and lungs, along with pseudoepitheliomatous hyperplasia in the
skin?
A) Cryptococcus neoformans
B) Coccidioides immitis
C) Mucormycosis
D) Blastomyces
E) Cryptosporidium
Answer: D
Blastomyces dermatitidis has a double-contoured capsule and causes
granulomatous infections in the skin and lungs, with pseudoepitheliomatous
hyperplasia, sometimes mistaken for squamous cell carcinoma.
---
243 Pathology Questions and Answers, Prof.Dr. Murat ALPER
61. Which of the following is most likely associated with T cell deficiency?
A) Haemophilus influenzae meningitis
B) Recurrent staphylococcal furunculosis
C) Absence of germinal centers in the spleen and lymph nodes
D) Pneumocystis jirovecii pneumonia
E) Severe anaphylactic reaction to IgA in transfused blood
Answer: D
Pneumocystis jirovecii pneumonia is a classic example of an opportunistic
infection seen in T cell immunodeficiency, such as in AIDS. T cell deficiencies do not
typically impair bacterial infection resistance as much as B cell deficiencies.
---
REVIEW QUESTIONS:
(Review and write down what you remember about the following topics...)
- Categorization of infectious agents?
- Host resistance?
- Histochemical tests used in diagnosis?
- Types of inflammation?
- Rhinovirus?
- Influenza virus?
- H. influenzae?
- Tuberculosis?
- Primary tuberculosis?
- Secondary tuberculosis?
- Histoplasma?
- Coccidioides?
- Viral enteritis?
244 Pathology Questions and Answers, Prof.Dr. Murat ALPER
- Shigella?
- E. coli?
- C. jejuni?
- Yersinia?
- Salmonella?
- Vibrio cholerae?
- Entamoeba histolytica?
- Giardia lamblia?
- Herpesvirus?
- Chlamydia trachomatis?
- Neisseria gonorrhoeae?
- Treponema pallidum?
- Staphylococcus aureus?
- Streptococcus?
- Clostridium?
- Trichomonas vaginalis?
- Rubeola?
- Rubella?
- EBV?
- Poliovirus?
- Varicella-Zoster virus?
- Diphtheria?
- CMV?
- Pseudomonas?
- Legionella?
- Listeria?
- Candida?
- Cryptococcosis?
- Aspergillus fumigatus?
245 Pathology Questions and Answers, Prof.Dr. Murat ALPER
- Mucormycosis?
- Pneumocystis jirovecii?
- Cryptosporidium parvum?
- Toxoplasma gondii?
- Rickettsiae?
- Leprosy?
- Lyme disease?
- Malaria?
- Leishmania?
- Trypanosoma?
- Chagas disease?
- Trichinella?
- Taenia?
- Hydatid cyst?
- Schistosoma?
- Elephantiasis?
- Onchocerca?
246 Pathology Questions and Answers, Prof.Dr. Murat ALPER
SECTION 11: VASCULAR PATHOLOGY
1. Which of the following factors does not suppress the migration and
proliferation of vascular smooth muscle cells?
A) PDGF
B) Heparan sulfate
C) Nitric oxide
D) Interferon gamma
E) TGF-Beta
Answer: A
The main cellular elements of the vascular wall are endothelial cells and
smooth muscle cells. Endothelial cells contain Wiebel-Palade bodies that store von
Willebrand factor. Smooth muscle cells, in response to normal or pharmacological
stimuli, cause vasoconstriction or vasodilation, synthesize collagen, elastin, and
proteoglycans, and secrete growth factors and cytokines. In case of vascular injury,
they migrate from the media to the intima, proliferate, and produce matrix. Smooth
muscle cell proliferation is stimulated by PDGF, basic FGF, and interleukin-1 derived
from platelets, endothelial cells, and macrophages. In contrast, it is suppressed by
heparan sulfates, nitric oxide, interferon-gamma, and TGF-beta.
2. In Mönckeberg disease, in which part of the vessel is the main pathology
located?
A) Intima
B) Vasa vasorum
C) Adventitia
D) Media
E) Subintima
Answer: D
Arteriosclerosis describes the thickening and loss of elasticity of the arterial
wall. It has four clinical and pathological types: arteriolosclerosis, Mönckeberg medial
sclerosis, fibromuscular intimal hyperplasia, and atherosclerosis. Fibromuscular
intimal hyperplasia is a non-atherosclerotic process seen in muscular arteries, leading
247 Pathology Questions and Answers, Prof.Dr. Murat ALPER
to stenosis due to healed vascular inflammation and intimal hyperplasia, which can be
a problem in solid organ transplants. Arteriolosclerosis is the thickening and rigidity of
the walls of small arteries and arterioles, most commonly seen in hypertension and
diabetes, with hyaline and hyperplastic types. Mönckeberg medial calcific sclerosis
involves calcifications in the media of medium-sized muscular arteries, typically seen
in individuals over 50 years old. In the other types, the intimal layer is involved
(Mönckeberg-Medial-Medium-sized-Muskular all start with M).
---
3. In an autopsy case, atheromatous plaques are found in the abdominal
aorta. Which of the following is least likely to be present in this patient's clinical
history?
A) Hypertension
B) Hyperlipidemia
C) Hypercalcemia
D) Diabetes mellitus
E) Smoking
Answer: C
---
4. Which of the following statements is incorrect regarding partially
controllable risk factors for atherosclerosis development?
A) Acquired hyperlipidemia
B) Hypertension
C) Smoking
D) Diabetes
E) Familial hyperlipidemia
Answer: E
248 Pathology Questions and Answers, Prof.Dr. Murat ALPER
As age increases, the risk of atherosclerosis increases. In premenopausal
women, the risk is lower due to estrogen, but it gradually increases after menopause,
equaling that of men by the 7th or 8th decade. Five basic risk factors that are at least
partially controllable are acquired hyperlipidemia, hypertension, smoking, C-reactive
protein, and diabetes. Metabolic syndrome, Lipoprotein(a) levels, lack of exercise,
increased levels of procoagulants, obesity, and Type A personality are also risk factors.
Familial hypercholesterolemia is a hereditary and uncontrollable risk factor.
Atherosclerosis affects elastic arteries (e.g., aorta, carotid, iliac arteries) and large to
medium-sized muscular arteries (e.g., coronary and popliteal arteries). Some studies
mention CRP levels as an independent indicator for myocardial infarction, stroke,
peripheral arterial disease, and sudden cardiac death.
---
5. Which of the following plays the least role in the formation of
atherosclerotic fibrous plaques?
A) Smooth muscle cells
B) Platelets
C) Macrophages
D) Hyperlipidemia
E) NK cells
Answer: E
NK cells are large granular lymphocytes that are not B or T cells. They play a
role in antibody-dependent cytotoxicity reactions (Type II hypersensitivity), Graft
versus Host reactions, and the elimination of virus-infected and neoplastic cells. They
do not have a role in the formation of fibrous plaques, which is a late sign of
atherosclerosis. Macrophages and smooth muscle cells take up LDL and form foam
cells, an early sign of the lesion. Platelets contribute to growth factors that increase
smooth muscle proliferation. The plaque also contains collagen, elastin, foam cells,
and cholesterol.
---
6. Which of the following diseases can lead to early and severe
atherosclerosis?
249 Pathology Questions and Answers, Prof.Dr. Murat ALPER
A) Homocystinuria
B) Galactosemia
C) Fructosuria
D) Pompe disease
E) McArdle syndrome
Answer: A
Chronic or recurrent endothelial injury plays a key role in the development of
atherosclerosis. The two most important causes of endothelial dysfunction are
hemodynamic disturbances and hypercholesterolemia. Endotoxins, hypertension,
hyperlipidemia, anoxia, carbon monoxide, cigarette smoke products, viruses, and
specific endothelial toxins like homocysteine are also effective. Early and severe
atherosclerosis is associated with homocystinuria. Although folic acid and B12 levels
can increase homocysteine levels, adding these vitamins to the diet has not been
shown to reduce cardiovascular disease.
---
7. Atherosclerotic plaques are most commonly seen in which location?
A) Coronary artery
B) Popliteal artery
C) Abdominal aorta
D) Internal carotid artery
E) Circle of Willis
Answer: C
Atherosclerotic plaques are most commonly seen in the abdominal aorta,
followed by the coronary artery, popliteal artery, internal carotid artery, and the Circle
of Willis arteries, in that order. The key events leading to atheroma formation are
intimal thickening and lipid accumulation. The precursor lesion is a fatty streak,
followed by the development of a fibrous plaque. Changes in the atherosclerotic
plaque, such as rupture, erosion, and intraplaque hemorrhage, can lead to
thrombosis, atheroembolism, and aneurysmal dilatation.
250 Pathology Questions and Answers, Prof.Dr. Murat ALPER
---
8. Atherosclerotic aortic aneurysms are most commonly seen in which
location?
A) Aortic arch
B) Ascending aorta
C) Thoracic aorta
D) Abdominal aorta
E) Just above the diaphragm
Answer: D
Aneurysms are congenital or acquired dilations of blood vessels or the heart.
True aneurysms involve all three layers of the arterial wall: intima, media, and
adventitia. The most common cause of aortic aneurysms is atherosclerosis and
hypertension. Others include syphilis, trauma, vasculitis, infections, and conditions
with defective or abnormal connective tissue synthesis, such as Marfan and Ehler-
Danlos syndromes (especially Type IV EDS). Aneurysms are more common in males.
They are most often found in the abdominal aorta, especially below the renal artery
level, where atherosclerosis causes degradation of the tunica media. A mural
thrombus is frequently present in the aneurysmal sac. These aneurysms can cause
erosion of surrounding structures. While aneurysms smaller than 4 cm rarely rupture,
those larger than 6 cm have an increasing risk of rupture, leading to death. Dissecting
aortic aneurysms, or more accurately termed aortic dissections, are not true
aneurysms. Aortic dissection involves the formation of blood-filled channels within
the vessel wall, most commonly occurring in the ascending aorta within 10 cm of the
aortic valve. Rupture can result in massive hemorrhage or cardiac tamponade.
---
9. In which of the following conditions is atherosclerosis expected to play the
least role?
A) Formation of berry aneurysms
B) Small bowel infarction
C) Claudication in the legs
D) Transient ischemic attacks
251 Pathology Questions and Answers, Prof.Dr. Murat ALPER
E) Non-traumatic amputations
Answer: A
Berry aneurysms are typically congenital and involve a defect in the vascular
wall's internal elastic membrane and muscle at the aneurysm base. The other
options, such as small bowel infarction (superior mesenteric artery), claudication
(aorta and iliac arteries), transient ischemic attacks (carotid artery), and non-
traumatic amputations (popliteal artery), are commonly associated with
atherosclerosis.
---
10. Which of the following pathogens is most frequently seen in mycotic
aneurysms?
A) Candida
B) Histoplasma
C) Cryptococcus
D) Salmonella
E) Staphylococcus
Answer: D
A mycotic aneurysm is a dilation of an artery caused by an infection that
weakens the vessel wall. Thrombosis and rupture are potential complications.
Mycotic aneurysms are often associated with infective endocarditis, where septic
emboli lodge in various points of the artery. They can also arise from the spread of a
neighboring suppurative focus or directly from circulating microorganisms infecting
the artery wall. Salmonella species are most commonly implicated in mycotic
aneurysms, often occurring during bacteremia following a Salmonella gastroenteritis
episode.
---
11. In which type of aneurysm is the involvement of the aortic root commonly
seen?
252 Pathology Questions and Answers, Prof.Dr. Murat ALPER
A) Mycotic
B) Syphilitic
C) Atherosclerotic
D) Berry
E) Pseudoaneurysm
Answer: B
---
12. Which of the following specifically affects the thoracic aorta and causes
obliterative endarteritis in the vasa vasorum?
A) Mycobacterium tuberculosis
B) Brucella abortus
C) Treponema pallidum
D) Yersinia pestis
E) Clostridium perfringens
Answer: C
---
13. Which of the following statements is true regarding syphilitic aortic
aneurysms?
A) They are frequently seen in the abdominal aorta.
B) They are more common in women.
C) They are common in individuals under 35 years of age.
D) The earliest change occurs in the vasa vasorum.
253 Pathology Questions and Answers, Prof.Dr. Murat ALPER
E) They are seen in the second stage of syphilis.
Answer: D
Treponema pallidum, the causative agent of syphilis, leads to obliterative
endarteritis in the third stage of the disease, which can affect any small artery in the
body. However, the most severe clinical presentation involves the aortic root's vasa
vasorum, leading to thoracic aortitis, characterized by aneurysmal dilation of the
aorta and aortic valve ring. This form of cardiovascular syphilis is most commonly
seen in individuals aged 40-50 years, with a male-to-female ratio of 3:1. The earliest
change involves plasma cell-rich infiltrates in the vasa vasorum, progressing to tunica
media damage and scar formation, which narrows the vessels and can lead to the
development of the characteristic tree-bark appearance of the aortic wall. This
condition can result in aortic insufficiency (Cor bovinum).
---
14. Which vascular disease primarily affects elastic tissue-rich large arteries?
A) Leukocytoclastic vasculitis
B) Kawasaki arteritis
C) Temporal arteritis
D) Polyarteritis nodosa
E) Churg-Strauss allergic angiitis
Answer: C
Temporal (giant cell) arteritis affects large arteries rich in elastic tissue,
especially those in the head, including the temporal, vertebral, and ophthalmic
arteries. Polyarteritis nodosa (PAN) and Kawasaki disease affect medium-sized and
small arteries. Hypersensitivity vasculitis, also known as leukocytoclastic vasculitis,
affects venules, capillaries, and arterioles. Churg-Strauss allergic angiitis affects
medium and small arteries as well as veins, often involving the lungs.
---
254 Pathology Questions and Answers, Prof.Dr. Murat ALPER
15. In which vascular disease is antineutrophil cytoplasmic antibody (ANCA)
almost always found?
A) Polyarteritis nodosa (PAN)
B) Granulomatosis with polyangiitis
C) Churg-Strauss syndrome
D) Giant cell arteritis
E) Takayasu arteritis
Answer: B
Two types of antineutrophil cytoplasmic antibodies (ANCAs) are particularly
important: antiproteinase 3 (PR3-ANCA), formerly known as c-ANCA, associated with
granulomatosis with polyangiitis (formerly Wegener's granulomatosis), and
antimyeloperoxidase (MPO-ANCA), formerly known as p-ANCA. PR3-ANCA is found in
almost all cases of granulomatosis with polyangiitis (90%) and is rare in other
vasculitides, including PAN. MPO-ANCA is found in microscopic polyangiitis and
Churg-Strauss syndrome, where there is clinical overlap with PAN and granulomatosis
with polyangiitis.
---
16. In which pair of vascular diseases is the etiology based on similar
mechanisms?
A) Mycotic aneurysm and Berry aneurysm
B) Polyarteritis nodosa and Henoch-Schönlein purpura
C) Kaposi sarcoma and bacillary angiomatosis
D) Kawasaki disease and cavernous hemangioma
E) Pyogenic granuloma and capillary hemangioma
Answer: B
Polyarteritis nodosa (PAN) and Henoch-Schönlein purpura (HSP) are immune-
mediated diseases. In PAN, antibodies target muscular arteries, leading to neutrophil
and monocyte attacks. The resulting free radicals and enzymes damage the vessel
wall, causing local aneurysmal dilatations (nodosa). HSP is a small vessel vasculitis
associated with IgA-C3 immune complex deposition, leading to vessel damage,
255 Pathology Questions and Answers, Prof.Dr. Murat ALPER
rupture, and palpable purpura, typically on the lower extremities and buttocks.
Mycotic aneurysms occur secondary to sepsis, while Berry aneurysms are congenital.
Kaposi sarcoma is a malignancy of endothelial cells associated with KSHV (HHV-8),
while bacillary angiomatosis is an infectious disease caused by Bartonella
(Rochalimaea) henselae. Kawasaki disease is an immune-mediated vasculitis affecting
the coronary arteries, while cavernous hemangiomas are benign vascular neoplasms
(most common benign tumors of the liver, placenta, and spleen). Capillary
hemangiomas are considered either neoplasms or hamartomas, while pyogenic
granulomas are reactive, tumor-like formations that develop after trauma and bleed
easily, sometimes seen as gingival growths during pregnancy (pregnancy tumors).
---
17. Which disease, associated with polymyalgia rheumatica, affects elderly
women, can cause blindness, and involves segmental vascular involvement?
A) Systemic lupus erythematosus
B) Polyarteritis nodosa
C) Temporal arteritis
D) Takayasu arteritis
E) Kawasaki disease
Answer: C
Temporal (giant cell) arteritis is the most common vasculitis, typically affecting
elderly individuals, particularly women. It involves large arteries, especially those of
the head, including the temporal, vertebral, and ophthalmic arteries, and can cause
severe headaches, jaw claudication, and even blindness. It has a segmental
distribution, often featuring granulomatous inflammation with multinucleated giant
cells. Some cases arise from a background of polymyalgia rheumatica. There is a
recognized association between giant cell arteritis and the gene coding for ICAM-1.
---
18. Which of the following diseases generally affects larger arteries?
A) Thromboangiitis obliterans
B) Kawasaki disease
256 Pathology Questions and Answers, Prof.Dr. Murat ALPER
C) Temporal arteritis
D) Takayasu arteritis
E) Granulomatosis with polyangiitis
Answer: D
Takayasu arteritis (pulseless disease) is a granulomatous vasculitis that affects
large and medium-sized arteries, most commonly the aorta and its main branches, as
well as sometimes the pulmonary arteries. Kawasaki disease, PAN, and
granulomatosis with polyangiitis typically affect small to medium-sized arteries, while
thromboangiitis obliterans (Buerger's disease) affects small and medium-sized
arteries and veins. Temporal (giant cell) arteritis affects large elastic arteries,
especially those of the head. There is some overlap in the vessel sizes affected by
these diseases.
19. In an elderly patient with sudden, painful vision loss, which disease should
be considered first?
A) PAN
B) Giant cell arteritis
C) Granulomatosis with polyangiitis
D) Churg-Strauss disease
E) Kawasaki disease
Answer: B
20. Which vascular disease, more common in young women, is associated
with a 30-40 mmHg difference in blood pressure between the right and left arms and
has a frequent association with HLA-DR4?
A) Takayasu arteritis
B) Raynaud's disease
C) Temporal arteritis
D) PAN
E) Kawasaki disease
257 Pathology Questions and Answers, Prof.Dr. Murat ALPER
Answer: A
Temporal (giant cell) arteritis is more common in elderly women, while PAN is
more common in middle-aged men. Raynaud's disease is seen in healthy young
women but does not present with the symptoms described. Takayasu arteritis
(pulseless disease) typically affects individuals under 40, predominantly women, and
is associated with HLA-DR4 positivity. The disease causes granulomatous
inflammation, leading to narrowing of the aorta and its branches, sometimes
involving the pulmonary arteries. This results in vascular insufficiency symptoms in
the upper extremities, including numbness and coldness, and a 30-40 mmHg blood
pressure difference between the arms. Angiography is the primary diagnostic
method.
21. In which of the following conditions is granulomatous vasculitis not seen?
A) Temporal arteritis
B) Takayasu arteritis
C) Granulomatosis with polyangiitis
D) Churg-Strauss syndrome
E) Henoch-Schönlein purpura
Answer: E
22. Which vasculitis, affecting patients over 50 years of age, is associated with
polymyalgia rheumatica?
A) Temporal arteritis
B) Takayasu arteritis
C) Granulomatosis with polyangiitis
D) Churg-Strauss syndrome
E) Henoch-Schönlein purpura
Answer: A
23. Which vasculitis, characterized by IgA deposition, affects the skin,
intestines, and glomeruli?
258 Pathology Questions and Answers, Prof.Dr. Murat ALPER
A) Temporal arteritis
B) Takayasu arteritis
C) Granulomatosis with polyangiitis
D) Churg-Strauss syndrome
E) Henoch-Schönlein purpura
Answer: E
Henoch-Schönlein purpura is a small vessel vasculitis characterized by IgA
deposits, most commonly affecting the skin, gastrointestinal tract, and kidneys. It
frequently presents with palpable purpura, abdominal pain, arthralgia, and
hematuria. Temporal arteritis and Takayasu arteritis are large vessel vasculitides,
while granulomatosis with polyangiitis (formerly Wegener's granulomatosis) and
Churg-Strauss syndrome are associated with granulomatous inflammation of small to
medium-sized vessels.
24. Which disease, affecting small children, involves the coronary arteries,
leading to thrombosis, aneurysm formation, and myocardial infarction?
A) Systemic sclerosis
B) Raynaud's disease
C) Giant cell arteritis
D) Takayasu arteritis
E) Kawasaki disease
Answer: E
25. In a five-year-old child who presents with fever, bilateral non-bacterial
conjunctivitis, erythema and desquamation of the palms and soles,
lymphadenopathy, and who later develops a myocardial infarction, which vascular
disease should be suspected?
A) Takayasu arteritis
B) Kawasaki disease
C) Polyarteritis nodosa
D) Systemic lupus erythematosus
259 Pathology Questions and Answers, Prof.Dr. Murat ALPER
E) Granulomatosis with polyangiitis
Answer: B
Kawasaki disease (mucocutaneous lymph node syndrome) is an acute febrile
illness of early childhood. It is often self-limiting but can lead to serious complications,
including coronary artery involvement. The disease is characterized by vasculitis
affecting the coronary arteries, which can lead to aneurysm formation and myocardial
infarction. Clinically, it presents with fever, conjunctivitis, erythema and
desquamation of the palms and soles, a rash, cervical lymphadenopathy, arrhythmias,
mitral insufficiency, heart dilation, congestive heart failure, and myocardial infarction.
26. A 30-year-old woman presents with purpuric skin lesions on both legs. A
skin biopsy reveals fibrinoid necrosis in small dermal vessels and fragmented
neutrophils around the vessels. What is the most likely diagnosis?
A) Thromboangiitis obliterans
B) Leukocytoclastic vasculitis
C) Kawasaki disease
D) Takayasu arteritis
E) Temporal arteritis
Answer: B
27. Which of the following is not characteristic of Henoch-Schönlein purpura?
A) Proteinuria
B) Abdominal pain
C) Hematuria
D) Arthralgia
E) Involvement of small and medium-sized arteries
Answer: E
Henoch-Schönlein purpura (HSP) is a small vessel vasculitis characterized by
IgA deposition. It typically affects small vessels (arterioles, capillaries, venules) rather
than medium-sized arteries. The disease presents with purpura, abdominal pain,
260 Pathology Questions and Answers, Prof.Dr. Murat ALPER
arthralgia, hematuria, and proteinuria. The involvement of medium-sized arteries is
not characteristic of HSP.
---
28. In which of the following vasculitides is it characteristic to find lesions of
different ages in the same vessel or in different locations at the same time?
A) Kawasaki disease
B) Takayasu arteritis
C) Temporal arteritis
D) Polyarteritis nodosa
E) Granulomatosis with polyangiitis
Answer: D
---
29. Which organ is generally not affected by polyarteritis nodosa (PAN)?
A) Kidney
B) Testis
C) Lung
D) Liver
E) Gastrointestinal tract
Answer: C
Polyarteritis nodosa (PAN) classically affects medium and small-sized arteries,
causing acute transmural necrotizing inflammation. It is a segmental, relapsing
disease that can lead to aneurysmal dilatations (nodosa) and vascular occlusion,
resulting in infarction. PAN frequently involves the kidneys, heart, liver,
gastrointestinal tract, pancreas, and testis, among others, but typically spares the
lungs. Renal involvement is common and often the cause of death, but since small
vessels are not involved, there is no glomerulonephritis. Unlike microscopic
261 Pathology Questions and Answers, Prof.Dr. Murat ALPER
polyangiitis, PAN does not involve the lungs, and all lesions in a given patient are at
the same stage of development.
---
30. Which of the following is not a feature of granulomatosis with polyangiitis?
A) Antiproteinase-3 antibody positivity
B) Involvement of large vessels
C) Necrotizing glomerulonephritis
D) Necrotizing granulomas in the lungs
E) Ulcerative lesions in the sinonasal region
Answer: B
---
31. Which of the following is incorrect regarding granulomatosis with
polyangiitis?
A) Necrotizing granulomas in the respiratory tract
B) Necrotizing vasculitis in the lungs
C) Crescentic glomerulonephritis
D) Hypogammaglobulinemia
E) ANCA positivity
Answer: D
Granulomatosis with polyangiitis (formerly Wegener's granulomatosis) is
characterized by necrotizing granulomas in the upper and/or lower respiratory tract,
necrotizing granulomatous vasculitis, and necrotizing glomerulonephritis, often with
crescent formation. Pneumonia, chronic sinusitis, and mucosal ulceration of the
nasopharynx are common. The disease is also associated with
hypergammaglobulinemia, particularly elevated IgG and IgA levels, and is ANCA-
positive in over 95% of cases, with c-ANCA being the most common target,
262 Pathology Questions and Answers, Prof.Dr. Murat ALPER
particularly against proteinase 3 (PR3). The disease is often fatal within a year if
untreated, but treatment with cyclophosphamide can induce remission.
---
32. Which systemic vasculitis is characterized by fever, severe asthma, and
marked eosinophilia?
A) Granulomatosis with polyangiitis
B) Polyarteritis nodosa
C) Churg-Strauss syndrome
D) Kawasaki disease
E) Hypersensitivity vasculitis
Answer: C
Churg-Strauss syndrome, also known as allergic granulomatosis and angiitis, is
a rare necrotizing vasculitis of small vessels. Classic features include allergic rhinitis,
extravascular necrotizing granulomas, asthma, and marked peripheral eosinophilia.
Unlike granulomatosis with polyangiitis, Churg-Strauss syndrome is typically c-ANCA
negative but p-ANCA positive in about 70% of cases. Severe asthma, cutaneous
involvement (purpura), and gastrointestinal bleeding are common, and coronary
arteritis and myocarditis are major causes of morbidity and mortality.
---
33. Which vasculitis is more common in men?
A) Sjögren syndrome
B) Scleroderma
C) Polymyositis
D) Polyarteritis nodosa
E) Systemic lupus erythematosus
Answer: D
263 Pathology Questions and Answers, Prof.Dr. Murat ALPER
Systemic lupus erythematosus (SLE) occurs in 80% of cases in young women.
Scleroderma is also more common in young women, as is Sjögren syndrome, which
typically affects middle-aged women. Polymyositis occurs in young women and is
frequently associated with malignancy. In contrast, polyarteritis nodosa (PAN) is more
common in men, with a male-to-female ratio of 2:1 to 3:1. PAN is a segmental
necrotizing vasculitis of small and medium-sized arteries, often associated with
Hepatitis B surface antigenemia in about 30% of cases. Symptoms include fever,
weight loss, abdominal pain, hypertension, uremia, allergic asthma, peripheral
neuritis, and leukocytosis.
---
34. Which disease, seen in young men, involves segmental vascular
involvement, Raynaud's phenomenon, recurrent migratory thrombophlebitis, and can
lead to gangrene?
A) Kawasaki disease
B) Thromboangiitis obliterans
C) Scleroderma
D) Takayasu disease
E) Giant cell arteritis
Answer: B
Thromboangiitis obliterans (Buerger's disease) is a segmental inflammatory
disease of small and medium-sized arteries, leading to thrombosis and potentially
involving adjacent veins and nerves. It is strongly associated with smoking and is
more common in young men, although the incidence is rising in women.
Microscopically, acute and chronic inflammation, as well as thrombosis, are seen. The
thrombus may contain microabscesses surrounded by granulomatous inflammation.
Symptoms include Raynaud's phenomenon, migratory thrombophlebitis, claudication,
and eventually gangrene.
---
35. Vasospasm of the arterioles and small arteries in the fingers is most
characteristic of which condition?
264 Pathology Questions and Answers, Prof.Dr. Murat ALPER
A) Buerger's disease
B) Primary Raynaud's phenomenon
C) Granulomatosis with polyangiitis
D) Kawasaki disease
E) Takayasu arteritis
Answer: B
Raynaud's phenomenon involves vasoconstriction of the arteries and
arterioles in the extremities, leading to color changes in the fingers (red, white, and
blue) due to proximal vasodilation, central vas
oconstriction, and distal cyanosis, respectively. Primary Raynaud's
phenomenon, also known as Raynaud's disease, typically affects young women, often
involving the fingers and sometimes the toes. Secondary Raynaud's phenomenon is
associated with systemic conditions like SLE, scleroderma, atherosclerosis, and
Buerger's disease, and requires investigation for the underlying cause.
---
36. Which cancer is most commonly associated with migratory
thrombophlebitis (Trousseau's sign)?
A) Pancreatic adenocarcinoma
B) Bronchoalveolar carcinoma of the lung
C) Small cell carcinoma
D) Gastric adenocarcinoma
E) Lymphoma
Answer: A
Trousseau's sign is most commonly seen in cancers of the internal organs,
particularly abdominal tumors, with pancreatic cancer being the most common.
Thrombophlebitis refers to the inflammation and thrombosis of veins, typically
occurring in the deep veins of the lower extremities, with pulmonary embolism as a
potential complication.
265 Pathology Questions and Answers, Prof.Dr. Murat ALPER
37. A 69-year-old HIV-negative male patient has had three purple nodular
lesions on his left foot for two years. A biopsy of one lesion reveals spindle cell
proliferation in the dermis, with slit-like spaces containing erythrocytes and numerous
eosinophilic hyaline globules. The spindle cells are positive for HHV-8 and CD31 on
immunohistochemistry. What is the most likely diagnosis?
A) Bacillary angiomatosis
B) Angiosarcoma
C) Kaposi's sarcoma
D) Spindle cell hemangioma
E) Epithelioid hemangioendothelioma
Answer: C
Kaposi's sarcoma is a spindle cell tumor of endothelial origin associated with
Human Herpesvirus 8 (HHV-8). It is seen in 20% of AIDS patients but can also occur in
other immunocompromised individuals and older adults. The tumor is characterized
by spindle cell proliferation, slit-like vascular spaces, and eosinophilic hyaline
globules, with HHV-8 and CD31 positivity on immunohistochemistry.
38. A 40-year-old woman who had a kidney transplant three years ago
presents with multiple lesions involving lymph nodes, mucosa, and internal organs.
Histologically, the lesions consist of spindle cells and capillary vessels, and Human
Herpesvirus 8 is detected. What is the most likely diagnosis?
A) Kaposi's sarcoma
B) Cytomegalovirus infection
C) B-cell lymphoma
D) Neurofibromatosis
E) Mycobacteriosis
Answer: A
Kaposi's sarcoma is a spindle cell tumor associated with Human Herpesvirus 8
(HHV-8). It is commonly seen in immunocompromised individuals, including AIDS
patients and organ transplant recipients, and involves the skin, lymph nodes, mucosa,
and internal organs.
266 Pathology Questions and Answers, Prof.Dr. Murat ALPER
39. Which of the following is an intermediate-grade vascular tumor?
A) Hemangioma
B) Glomus tumor
C) Kaposi's sarcoma
D) Angiosarcoma
E) Bacillary angiomatosis
Answer: C
Benign vascular tumors and lesions include hemangiomas, lymphangiomas,
glomus tumors, vascular ectasias, and bacillary angiomatosis. Intermediate-grade
tumors include Kaposi's sarcoma and hemangioendothelioma. Malignant vascular
neoplasms include angiosarcoma. Angiosarcoma typically occurs in elderly individuals
and is most commonly found in the skin, soft tissue, breast, and liver. Hepatic
angiosarcomas are associated with exposure to arsenic, polyvinyl chloride, and other
carcinogens.
---
40. Which disease, which can be familial and present at birth, causes
lymphedema, interstitial fibrosis in the subcutaneous tissues, and predisposes to
cellulitis?
A) Von Hippel-Lindau disease
B) Milroy disease
C) Mikulicz syndrome
D) Marfan syndrome
E) Elephantiasis
Answer: B
Milroy disease, also known as congenital familial lymphedema, is a primary
lymphatic disorder present at birth. It is caused by lymphatic agenesis, hypoplasia,
valve incompetence, and abnormal dilatation, leading to interstitial fibrosis,
ulceration, and a predisposition to cellulitis. Von Hippel-Lindau disease is associated
with cavernous hemangiomas, renal cell carcinoma, and cerebellar
hemangioblastomas. Marfan syndrome is a connective tissue disorder associated with
267 Pathology Questions and Answers, Prof.Dr. Murat ALPER
dissecting aortic aneurysms. Mikulicz syndrome is a salivary gland disorder.
Elephantiasis is caused by lymphatic obstruction due to W. bancrofti and Brugia
malayi, leading to chronic lymphedema and fibrosis.
---
41. Which tumor is commonly seen in Turner syndrome?
A) Glomus tumor
B) Capillary lymphangioma
C) Cystic hygroma
D) Capillary hemangioma
E) Cavernous hemangioma
Answer: C
Cavernous lymphangioma (cystic hygroma) typically occurs in the neck and
axilla and, less commonly, in the retroperitoneum. These tumors are poorly
circumscribed, unencapsulated, and difficult to remove. They may contain
lymphocyte aggregates and can reach large sizes. Cavernous lymphangiomas of the
neck are frequently seen in Turner syndrome.
---
42. A patient presents with venous angiomatous masses localized in the
leptomeninges and port-wine stains in the distribution of the trigeminal nerve. The
patient also has mental retardation, seizures, and hemiplegia. What is the most likely
diagnosis?
A) Osler-Weber-Rendu disease
B) Sturge-Weber syndrome
C) Nevus flammeus
D) Spider telangiectasia
E) Glomus tumor
268 Pathology Questions and Answers, Prof.Dr. Murat ALPER
Answer: B
---
43. Which genetic vascular malformation, inherited in an autosomal dominant
manner, causes nosebleeds, hematuria, and gastrointestinal bleeding?
A) Von Hippel-Lindau disease
B) Osler-Weber-Rendu disease
C) Milroy disease
D) Glomangioma
E) Simple congenital lymphedema
Answer: B
Osler-Weber-Rendu disease, also known as hereditary hemorrhagic
telangiectasia, is an autosomal dominant disorder characterized by multiple
aneurysmal telangiectasias. It affects the skin, oral cavity, respiratory, gastrointestinal,
and urinary tracts, as well as the liver, brain, and spleen. Rupture of these lesions can
lead to nosebleeds, hematuria, and gastrointestinal bleeding. The disease is non-
neoplastic. Von Hippel-Lindau disease is associated with cavernous hemangiomas,
renal cell carcinoma, and cerebellar hemangioblastomas. Milroy disease and simple
congenital lymphedema are primary lymphatic disorders. Glomus tumors
(glomangiomas) are small, painful tumors that arise from the smooth muscle cells of
the glomus body, often located under the fingernails.
---
44. Which of the following describes vascular proliferation due to
opportunistic infection in an immunocompromised patient?
A) Pyogenic granuloma
B) Glomangioma
C) Cavernous lymphangioma
D) Spider telangiectasia
E) Bacillary angiomatosis
269 Pathology Questions and Answers, Prof.Dr. Murat ALPER
Answer: E
---
45. Which vascular tumor, originating from pericytes, can cause
hypoglycemia?
A) Hemangiopericytoma
B) Bacillary angiomatosis
C) Glomus tumor
D) Angiosarcoma
E) Hemangioendothelioma
Answer: A
Hemangiopericytoma is a tumor originating from pericytes. It is positive for
actin and vimentin and negative for desmin and myoglobin. Some patients with this
tumor develop hypoglycemia, which resolves after the tumor is removed. The tumor
is most commonly found in the pelvic retroperitoneum and thigh and is often
surrounded by a pseudocapsule. The tumor cells are individually surrounded by
reticulin fibers, which is important for differential diagnosis. The behavior of the
tumor can be benign or malignant. Hemangioendothelioma is a tumor with behavior
ranging from benign to malignant, with potential for recurrence, metastasis, and
tumor-related death. Glomus tumors are typically found in women and are located in
the distal parts of the fingers, often under the nails. Bacillary angiomatosis is a
vascular proliferation seen in immunocompromised patients, caused by B. henselae
and B. quintana, and is treatable with antibiotics.
---
Review Topics:
- Mönckeberg arteriosclerosis
- Types of hypertension
- Hyaline arteriolosclerosis
270 Pathology Questions and Answers, Prof.Dr. Murat ALPER
- Classification of vasculitis
- Temporal arteritis
- Takayasu arteritis
- PAN (Polyarteritis nodosa)
- Kawasaki disease
- Leukocytoclastic vasculitis
- Wegener's granulomatosis (Granulomatosis with polyangiitis)
- Buerger's disease (Thromboangiitis obliterans)
- Raynaud's disease
- Aneurysm
- Abdominal aortic aneurysm
- Varicose veins
- Thromboembolism
- Phlebothrombosis
- Superior vena cava syndrome
- Lymphangitis
- Lymphedema
271 Pathology Questions and Answers, Prof.Dr. Murat ALPER
SECTION 12: CARDIOVASCULAR PATHOLOGY
SECTION 13: HEMATOPOETIC SYSTEM
SECTION 2: INFLAMMATION
SECTION 2: INFLAMMATION
SECTION 2: INFLAMMATION
SECTION 2: INFLAMMATION
SECTION 2: INFLAMMATION
SECTION 2: INFLAMMATION
SECTION 2: INFLAMMATION
SECTION 2: INFLAMMATION
272 Pathology Questions and Answers, Prof.Dr. Murat ALPER
SECTION 2: INFLAMMATION
SECTION 2: INFLAMMATION
SECTION 2: INFLAMMATION
SECTION 2: INFLAMMATION
SECTION 2: INFLAMMATION
273 Pathology Questions and Answers, Prof.Dr. Murat ALPER
SECTION 2: INFLAMMATION
SECTION 2: INFLAMMATION
274 Pathology Questions and Answers, Prof.Dr. Murat ALPER
SECTION 2: INFLAMMATION
SECTION 2: INFLAMMATION
SECTION 2: INFLAMMATION
275 Pathology Questions and Answers, Prof.Dr. Murat ALPER
SECTION 22: HEAD- NECK PATHOLOGY
276 Pathology Questions and Answers, Prof.Dr. Murat ALPER
SECTION 23: NEUROPATHOLOGY
Certainly! Here's a translation of the entire document you provided:
1. Which of the following regenerates differently from the others?
A) Nerve tissue
B) Kidney
C) Skin
D) Liver
E) Spleen
The central nervous system (CNS) differs from other organs in that if its main cell, the
neuron, is lost, it cannot be replaced, and regeneration of neuron extensions is often
unsuccessful. On the other hand, the CNS has significant plasticity and clinically silent areas,
at least in young people. While even very mild lesions in areas such as the spinal cord,
brainstem, dentate nucleus, posterior limb of the internal capsule, and motor cortex are not
promising, lesions that do not cause significant clinical deficiency can occur in areas such as
the cerebellar cortex, frontal lobe, non-dominant parietal and temporal lobes, and anterior
limb of the internal capsule. (Answer A)
2. Which of the following is the equivalent of Schwann cells in the central nervous
system?
A) Neuron
B) Astrocyte
C) Microglia
D) Oligodendroglia
E) Ependymal cells
Oligodendrocytes are mainly found in white matter and are the CNS equivalent of Schwann
cells. The regenerative capacity of oligodendrocytes is limited compared to astrocytes and
Schwann cells. While Schwann cells can easily show mitotic activity and hyperplasia,
oligodendrocytes rarely do so, typically within or around demyelinating plaques or necrotic
lesions. This is one reason the CNS has poor remyelination and regenerative potential.
277 Pathology Questions and Answers, Prof.Dr. Murat ALPER
Viruses such as Polyoma JC and measles or immune-origin diseases like multiple sclerosis can
also damage oligodendrocytes. (Answer D)
3. In the central nervous system, synaptophysin is used to detect which of the
following?
A) Neurons
B) Oligodendrocytes
C) Astrocytes
D) Microglia
E) Ependymal cells
Due to the morphological similarities between tightly interwoven cell types, special stains are
often needed to evaluate neuropathological materials thoroughly. Immunohistochemical
stains with antibodies against neuronal proteins (neurofilament, synaptophysin), astrocytic
proteins (GFAP, vimentin, S–100), oligodendrocyte membrane glycolipids or proteins
(carbonic anhydrase, Leu–7), and microglia (CD 68) facilitate the recognition of specific cell
types. However, the presence of cross-reactions between some cells (such as microglia and
endothelial cells) and the lack of practical markers (such as for ependymal and
oligodendroglial cells) limit the usefulness of these techniques. (Answer A)
4. Which of the following is the primary cell responsible for repair and scar tissue
formation in the CNS?
A) Schwann cells
B) Microglia
C) Ependymal cells
D) Astrocytes
E) Oligodendroglia
(Answer D)
5. Which cell is responsible for gliosis in the central nervous system?
A) Schwannom
B) Astrocyte
278 Pathology Questions and Answers, Prof.Dr. Murat ALPER
C) Oligodendrocyte
D) Microglia
E) Purkinje cells
(Answer B)
6. Central chromatolysis in neurons develops secondary to which of the following?
A) Neurofibrillary tangles
B) Lipofuscin accumulation in the perikaryon
C) Axonal transection or severe damage
D) Transsynaptic degeneration
E) Herpetic infection
(Answer C)
7. Which of the following is not a change seen in neurons?
A) Chromatolysis
B) Axonal degeneration
C) Neurofibrillary tangles
D) Lewy body
E) Gliosis
(Answer E)
8. Which cells produce myelin in the CNS?
A) Schwann cells
B) Purkinje cells
C) Ependymal cells
D) Astrocytes
E) Oligodendroglia
Schwann cells and oligodendrogliomas wrap around the nervous system and produce myelin.
Oligodendroglia are the equivalent of Schwann cells in the central nervous system.
279 Pathology Questions and Answers, Prof.Dr. Murat ALPER
Astrocytes form glial scar tissue, known as gliosis, in damaged brain tissue, making them the
primary cells responsible for repair. Protoplasmic astrocytes are mainly found in gray matter,
while fibrous astrocytes are found in both gray and white matter. Ependymal cells line the
brain's ventricles. Microglia are elements of the mononuclear phagocytic system. (Answer E)
9. Which of the following is associated with Rosenthal fibers?
A) Neuron
B) Astrocyte
C) Microglia
D) Oligodendroglia
E) Ependymal cells
Rosenthal fibers are astrocytic inclusions. They are intracytoplasmic protein accumulations
with a dark eosinophilic, long, often refractile appearance, composed of alpha–B crystallin.
They stain well with PTAH but are generally not well distinguished with anti-GFAP antibodies.
Rosenthal fibers are indicative of low-grade, especially juvenile pilocytic-type astrocytomas,
but can also be seen in reactive gliosis. (Answer B)
10. Which of the following malformations is most frequently associated with
meningomyelocele?
A) Sacrum agenesis
B) Arnold–Chiari
C) Dandy – Walker
D) Meningioma
E) Dermoid cyst
The failure of the neural tube's anterior end to close results in anencephaly, with the absence
of the brain and calvarium. The most common neural tube defect is spinal cord involvement
due to the failure of the neural tube's caudal part to close or reopen. It can range from an
asymptomatic bony defect (spina bifida occulta) to meningomyelocele. In Dandy–Walker
malformation, the posterior fossa is enlarged, and the cerebellar vermis is absent or
rudimentary, with a large cyst lined by ependymal cells in its place. Meningomyelocele is
located in the lumbosacral region in 90% of cases and is associated with Arnold–Chiari
malformation in 80% of cases. Hydrocephalus is also common. The higher the lesion, the
greater the likelihood of hydrocephalus. (Answer B)
280 Pathology Questions and Answers, Prof.Dr. Murat ALPER
11. Which of the following is incorrect about syringomyelia?
A) Most commonly seen in the cervical region
B) Pain sensation is decreased or absent
C) Temperature sensation is decreased or absent
D) Touch sensation is decreased or absent
E) Position sensation is preserved
Syringomyelia involves the development of a cavity (syrinx) in the spinal cord, most
commonly in the cervical region. Pain and temperature sensations are decreased or absent,
whereas touch, position, and vibration sensations are preserved. (Answer D)
12. What is the most important cause of cerebrovascular diseases?
A) Atherosclerosis
B) Vasculitis
C) Trauma
D) Vasospasm
E) Hematologic diseases
Cerebrovascular diseases are often caused by atherosclerosis, thrombosis, or embolism. Less
common causes include vasculitis, trauma, vasospasm, and hematological diseases.
Atherosclerosis is the most common cause of cerebrovascular disease, with hypertension
and diabetes as major predisposing factors for cerebral artery atherosclerosis, similar to
other parts of the body. Certain areas are at high risk for atheromatous plaque development:
the entry point of the vertebral arteries, the proximal and distal segments of the basilar
artery, the exit point of the internal carotid arteries, and their siphon and supraclinoid
segments. Mural thrombi form as plaques enlarge or ulcerate, leading to stenosis. Cerebral
blood flow is generally compromised when a plaque or thrombus narrows a vessel lumen by
at least 80%. A mural thrombus in a large vessel like the internal carotid artery can be a
source of emboli. If small fragments break off from the clot, they can cause transient
ischemic attacks (TIA) lasting less than 24 hours, with the patient making a full recovery.
These are harbingers of more severe cerebrovascular events. Emboli most commonly occur
in the territory of the middle cerebral artery, particularly at its origin and trifurcation. In
addition to ulcerated atheromatous plaques, another common embolic source is the heart's
major vessels, especially in patients with atrial fibrillation, where thromboemboli can
281 Pathology Questions and Answers, Prof.Dr. Murat ALPER
originate from atrial clots, mural thrombi on endomyocardial infarcts, or vegetations on
valves or prostheses. (Answer A)
13. Which of the following is not characteristic of berry aneurysms?
A) Localization at arterial bifurcation points
B) Common cause of subarachnoid hemorrhage
C) Most often found around the Circle of Willis
D) Most often due to severe atherosclerosis
E) Frequently associated with polycystic kidney disease
Berry aneurysms occur where the continuity of the arterial media is lost, primarily at the
bifurcation points of the arteries of the Circle of Willis. The most common sites include the
intersection of the anterior cerebral and anterior communicating arteries, the bifurcation of
the middle cerebral artery, the intersection of the internal carotid artery with the posterior
communicating artery, and the junction of the
basilar artery with the vertebral arteries. They occur sporadically and are associated with
conditions such as adult polycystic kidney disease, fibromuscular dysplasia, and
neurofibromatosis type 1. Rupture often results in subarachnoid hemorrhage, but they can
also rupture into the brain parenchyma or ventricles. Atherosclerotic (fusiform) aneurysms
are not congenital and are more commonly found in the basilar artery. Infected (mycotic)
aneurysms often develop in the peripheral cerebral arteries as a complication of infectious
endocarditis, and traumatic aneurysms are often related to skull fractures. (Answer D)
14. Which of the following is true about intraparenchymal hemorrhage?
A) Almost always associated with chronic hypertension
B) Almost always associated with subarachnoid hemorrhage
C) Most commonly associated with amyloid angiopathy
D) Almost always associated with acute hypertension
E) Almost always associated with subdural hematoma
Intraparenchymal hemorrhage (IPH) often occurs in hypertensive patients over 50 and can
occur in the absence of pre-existing hypertension. Hypertension is a major cause of vascular
malformations and IPH. Hypertension leads to hyaline arteriosclerosis in small arteries and
282 Pathology Questions and Answers, Prof.Dr. Murat ALPER
arterioles, predisposing them to rupture. Over time, Charcot–Bouchard microaneurysms
develop in small vessels due to degenerative changes, making them susceptible to rupture.
IPH can also occur from vascular malformations, coagulopathy, or cerebral amyloid
angiopathy (CAA), which can rupture and bleed. (Answer A)
15. Which of the following is incorrect about subarachnoid hemorrhage?
A) Most commonly results from rupture of a saccular aneurysm
B) Most commonly occurs in men
C) Can occur due to rupture of vascular malformations
D) Can occur as a result of trauma
E) Can occur idiopathically
Subarachnoid hemorrhage (SAH) can occur from saccular aneurysm rupture, arteriovenous
malformation rupture, trauma, or idiopathically. SAH can be seen in hypertensive patients
with spontaneous cerebellar hemorrhage or a ruptured hemangioblastoma. It can occur in
patients with diffuse brain edema, trauma, coagulopathy, or when there is a break in the
skull base. In SAH, the entire brain surface, including the basal cisterns, can be covered with
fresh blood. In large aneurysms, the basal ganglia, thalamus, and brainstem can also bleed
into the brain parenchyma. (Answer B)
16. Which of the following is incorrect about arteriovenous malformation?
A) It is a congenital vascular malformation.
B) It can lead to subarachnoid hemorrhage.
C) It can lead to cerebral hemorrhage.
D) It is not usually seen in the brain parenchyma.
E) It can cause epileptic seizures.
The most common vascular malformation is arteriovenous malformation (AVM). AVMs often
occur in brain tissue and can cause clinical events such as epileptic seizures, subarachnoid
hemorrhage, and parenchymal bleeding. (Answer D)
17. Which of the following does not belong to lacunar infarcts?
A) Atherosclerosis in deep parenchymal arteries
283 Pathology Questions and Answers, Prof.Dr. Murat ALPER
B) Most commonly seen in the basal ganglia
C) Related to hypertension
D) Seen in areas such as thalamus and pons
E) Appears as white matter demyelination
Vessels affected by hyaline arteriosclerosis in hypertensive patients often undergo occlusion
due to progressive luminal narrowing. Deep, penetrating vessels often undergo small,
cavitary infarcts known as lacunar infarcts. These lesions are most commonly seen in the
basal ganglia, thalamus, internal capsule, and deep white matter. (Answer E)
18. Which of the following is correct about brain herniations?
A) Uncal herniation is more dangerous than tonsillar herniation.
B) Subfalcine herniation often compresses the contralateral anterior cerebral artery.
C) Tonsillar herniation often compresses the cerebellar arteries.
D) Uncal herniation compresses the optic nerve.
E) Tonsillar herniation compresses the brainstem and its vessels.
Brain herniations can occur following a severe head injury, mass lesion, infection, or non-
traumatic cerebral edema. If there is an increase in pressure in the infratentorial
compartment, the cerebellar tonsils herniate into the foramen magnum, compressing the
medulla oblongata. Cerebellar tonsillar herniation can cause death within minutes. In uncal
(transtentorial) herniation, the uncus herniates into the posterior cranial fossa, often
compressing the midbrain. In subfalcine (cingulate) herniation, the cingulate gyrus herniates
under the falx cerebri, which can compress the anterior cerebral artery, resulting in ischemic
infarcts. (Answer E)
19. Which of the following is not a feature of multiple sclerosis?
A) Demyelinating lesions with preserved axons
B) Perivascular lymphocyte infiltration
C) High myelin content in active plaques
D) Proliferation of astrocytes in old lesions
E) Oligodendroglia cell disappearance in demyelinated areas
284 Pathology Questions and Answers, Prof.Dr. Murat ALPER
Multiple sclerosis is an immune-mediated demyelinating disease characterized by relapses
and remissions. Demyelinated axons are initially preserved in the perivascular areas, with
inflammatory infiltrates composed of lymphocytes and macrophages. T-helper cells are a
significant component of the inflammatory infiltrate. Inflammation is not prominent in old
plaques. Despite the axons being preserved, in areas of demyelination, axons become
damaged due to loss of myelin. Gliosis and axonal degeneration occur in demyelinated areas.
(Answer C)
20. Which of the following is true for Guillain–Barré syndrome?
A) Demyelinating disease
B) Transmissible disease
C) Involves the central nervous system
D) Hereditary disease
E) Peripheral nerve ganglion cell degeneration
Guillain–Barré syndrome is a demyelinating disorder affecting peripheral nerves, with
immunological mechanisms playing a crucial role in the disease. The pathogenesis of the
disease involves autoimmune processes triggered by preceding infections, vaccinations, or
other factors, leading to demyelination of peripheral nerves. It is not transmissible or
hereditary. (Answer A)
21. Which of the following is true for Krabbe's disease?
A) Metachromatic leukodystrophy
B) A type of lysosomal storage disease
C) Hereditary demyelinating disease
D) Characterized by the accumulation of phytanic acid
E) Absence of arylsulfatase A
Krabbe's disease is a lysosomal storage disease characterized by an accumulation of
galactocerebroside due to a deficiency in galactocerebrosidase. This results in the
destruction of myelin in the central and peripheral nervous systems, leading to severe
neurological symptoms. It is a hereditary demyelinating disease. (Answer C)
22. Which of the following is not true for metachromatic leukodystrophy?
285 Pathology Questions and Answers, Prof.Dr. Murat ALPER
A) Deficiency of galactocerebrosidase
B) Accumulation of sulfatides
C) Hereditary disease
D) Demyelination of the central nervous system
E) Demyelination of the peripheral nervous system
Metachromatic leukodystrophy is a hereditary demyelinating disease affecting the central
and peripheral nervous systems. It is characterized by the accumulation of sulfatides due to a
deficiency in arylsulfatase A, which leads to myelin destruction. Krabbe's disease, on the
other hand, is characterized by a deficiency in galactocerebrosidase. (Answer A)
23. Which of the following is correct about central pontine myelinolysis?
A) Associated with rapid correction of hyponatremia
B) Results in demyelination of peripheral nerves
C) Associated with multiple sclerosis
D) Characterized by lysosomal storage
E) Involves the anterior horn cells
Central pontine myelinolysis is a demyelinating disorder primarily affecting the central pons.
It is often associated with the rapid correction of hyponatremia, which leads to osmotic shifts
and myelin destruction. It is not related to multiple sclerosis, lysosomal storage, or anterior
horn cell involvement. (Answer A)
24. Which of the following diseases is characterized by neuronal degeneration?
A) Metachromatic leukodystrophy
B) Krabbe's disease
C) Amyotrophic lateral sclerosis
D) Guillain–Barré syndrome
E) Multiple sclerosis
Amyotrophic lateral sclerosis (ALS) is a neurodegenerative disease characterized by the
degeneration of motor neurons in the brain and spinal cord. It leads to progressive muscle
286 Pathology Questions and Answers, Prof.Dr. Murat ALPER
weakness and atrophy. Unlike demyelinating diseases such as metachromatic
leukodystrophy, Krabbe's disease, Guillain–Barré syndrome, and multiple sclerosis, ALS
primarily affects neurons rather than myelin. (Answer C)
25. Which of the following is not true for Alzheimer's disease?
A) Neurofibrillary tangles are characteristic
B) Amyloid plaques are characteristic
C) Associated with the tau
protein
D) Characterized by the presence of Lewy bodies
E) Associated with apolipoprotein E
Alzheimer's disease is characterized by the presence of amyloid plaques and neurofibrillary
tangles in the brain. The tau protein is involved in the formation of neurofibrillary tangles.
Apolipoprotein E is associated with Alzheimer's disease risk, particularly the ApoE4 allele.
Lewy bodies are not characteristic of Alzheimer's disease; they are associated with Lewy
body dementia and Parkinson's disease. (Answer D)
26. Which of the following is incorrect about Parkinson's disease?
A) Associated with the tau protein
B) Associated with Lewy bodies
C) Characterized by the degeneration of dopamine-producing neurons
D) Can cause movement disorders
E) Treated with L-dopa
Parkinson's disease is characterized by the degeneration of dopamine-producing neurons in
the substantia nigra, leading to movement disorders such as tremors, rigidity, and
bradykinesia. Lewy bodies are a pathological hallmark of Parkinson's disease. While tau
protein is associated with other neurodegenerative diseases such as Alzheimer's, it is not
specifically linked to Parkinson's disease. Treatment often includes L-dopa to replenish
dopamine levels. (Answer A)
287 Pathology Questions and Answers, Prof.Dr. Murat ALPER
27. Which of the following is true about Huntington's disease?
A) Characterized by neurofibrillary tangles
B) Characterized by amyloid plaques
C) Characterized by trinucleotide repeat expansion
D) Characterized by alpha-synuclein deposition
E) Characterized by the accumulation of galactocerebroside
Huntington's disease is a genetic neurodegenerative disorder caused by a trinucleotide
repeat expansion (CAG repeats) in the huntingtin gene. This expansion leads to the
production of a mutant huntingtin protein, which causes neuronal degeneration, particularly
in the basal ganglia, leading to movement disorders and cognitive decline. It is not
characterized by neurofibrillary tangles, amyloid plaques, alpha-synuclein deposition, or the
accumulation of galactocerebroside. (Answer C)
28. Which of the following is incorrect about Prion diseases?
A) Caused by infectious proteins
B) Characterized by spongiform changes in the brain
C) Associated with the deposition of amyloid plaques
D) Can be sporadic, inherited, or acquired
E) Associated with rapid neurodegeneration
Prion diseases are a group of neurodegenerative disorders caused by infectious proteins
known as prions. These diseases are characterized by spongiform changes in the brain,
leading to rapid neurodegeneration. They can occur sporadically, be inherited, or acquired
through exposure to prion-contaminated material. While amyloid plaques are associated
with Alzheimer's disease, prion diseases are characterized by the accumulation of misfolded
prion proteins, not amyloid plaques. (Answer C)
29. Which of the following is not a feature of Creutzfeldt–Jakob disease?
A) Rapidly progressive dementia
B) Myoclonus
C) Visual or cerebellar disturbances
D) Slowly progressive course
288 Pathology Questions and Answers, Prof.Dr. Murat ALPER
E) Associated with prion protein
Creutzfeldt–Jakob disease (CJD) is a rapidly progressive prion disease characterized by
dementia, myoclonus, and visual or cerebellar disturbances. It is associated with the
accumulation of misfolded prion protein in the brain, leading to neurodegeneration. CJD
typically has a rapidly progressive course, distinguishing it from other neurodegenerative
diseases with a slower progression. (Answer D)
30. Which of the following is correct about astrocytomas?
A) Most common in children
B) Usually arise in the cerebral hemispheres
C) Associated with rapid progression and poor prognosis
D) Typically calcified
E) Associated with neurofibrillary tangles
Astrocytomas are the most common primary brain tumors in adults, typically arising in the
cerebral hemispheres. They can vary in their progression and prognosis, ranging from slow-
growing pilocytic astrocytomas to aggressive glioblastomas. While they can occur in children,
they are more common in adults. Calcification is not a typical feature of astrocytomas, and
they are not associated with neurofibrillary tangles, which are characteristic of Alzheimer's
disease. (Answer B)
31. Which of the following is not true for medulloblastomas?
A) Most commonly occur in the cerebellum
B) Most common malignant brain tumor in children
C) Arise from neuroectodermal cells
D) Typically calcified
E) Can cause hydrocephalus
Medulloblastomas are malignant brain tumors that most commonly occur in the cerebellum
and are the most common malignant brain tumor in children. They arise from
neuroectodermal cells and can lead to hydrocephalus due to obstruction of cerebrospinal
fluid flow. Calcification is not a typical feature of medulloblastomas. (Answer D)
289 Pathology Questions and Answers, Prof.Dr. Murat ALPER
32. Which of the following is not true for meningiomas?
A) Derived from arachnoid cap cells
B) Usually benign
C) More common in women
D) Typically arise within the brain parenchyma
E) Can cause seizures
Meningiomas are typically benign tumors derived from arachnoid cap cells and are more
common in women. They usually arise from the meninges, not within the brain parenchyma,
and can cause seizures due to compression of adjacent brain tissue. (Answer D)
33. Which of the following is not true for oligodendrogliomas?
A) Most commonly occur in the frontal lobes
B) Characterized by calcifications
C) Associated with a favorable prognosis
D) Derived from glial cells
E) Most commonly seen in children
Oligodendrogliomas are tumors derived from glial cells, most commonly occurring in the
frontal lobes of adults. They are characterized by calcifications and have a relatively favorable
prognosis compared to other gliomas. They are less common in children compared to adults.
(Answer E)
34. Which of the following is not true for ependymomas?
A) Derived from ependymal cells
B) Most commonly occur in the ventricles
C) Most common in adults
D) Can cause hydrocephalus
E) Associated with spinal cord tumors
290 Pathology Questions and Answers, Prof.Dr. Murat ALPER
Ependymomas are tumors derived from ependymal cells that line the ventricles and the
central canal of the spinal cord. They most commonly occur in the ventricles and can cause
hydrocephalus due to obstruction of cerebrospinal fluid flow. Ependymomas are more
common in children and adolescents, particularly in the posterior fossa. They are also
associated with spinal cord tumors in adults. (Answer C)
35. Which of the following is not true for schwannomas?
A) Derived from Schwann cells
B) Most commonly occur on cranial nerve VIII
C) Associated with neurofibromatosis type 2
D) Typically malignant
E) Can cause hearing loss
Schwannomas, also known as acoustic neuromas, are benign tumors derived from Schwann
cells, most commonly occurring on cranial nerve VIII (the vestibulocochlear nerve). They are
associated with neurofibromatosis type 2 and can cause hearing loss due to compression of
the auditory nerve. Schwannomas are typically benign, not malignant. (Answer D)
36. Which of the following is not true for neurofibromatosis type 1?
A) Characterized by café-au-lait spots
B) Associated with Lisch nodules
C) Associated with bilateral vestibular schwannomas
D) Associated with neurofibromas
E) Associated with optic gliomas
Neurofibromatosis type 1 (NF1), also known as von Recklinghausen disease, is characterized
by café-au-lait spots, Lisch nodules, neurofibromas, and optic gliomas. Bilateral vestibular
schwannomas are characteristic of neurofibromatosis type 2 (NF2), not NF1. (Answer C)
37. Which of the following is not true for tuberous sclerosis?
A) Characterized by cortical tubers
B) Associated with renal angiomyolipomas
291 Pathology Questions and Answers, Prof.Dr. Murat ALPER
C) Associated with cardiac rhabdomyomas
D) Associated with bilateral vestibular schwannomas
E) Characterized by subependymal nodules
Tuberous sclerosis is a genetic disorder characterized by the presence of cortical tubers,
subependymal nodules, renal angiomyolipomas, and cardiac rhabdomyomas. Bilateral
vestibular schwannomas are not associated with tuberous sclerosis; they are characteristic of
neurofibromatosis type 2. (Answer D)
38. Which of the following is not true for Von Hippel–Lindau disease?
A) Associated with retinal hemangioblastomas
B) Associated with renal cell carcinoma
C) Associated with pheochromocytomas
D) Associated with pancreatic cysts
E) Characterized by axillary freckling
Von Hippel–Lindau disease is a genetic disorder associated with retinal hemangioblastomas,
renal cell carcinoma, pheochromocytomas, and pancreatic cysts. Axillary freckling is not
associated with Von Hippel–Lindau disease; it is a feature of neurofibromatosis type 1.
(Answer E)
39. Which of the following is not true for Alzheimer's disease?
A
) Associated with neurofibrillary tangles
B) Associated with amyloid plaques
C) Characterized by degeneration of basal ganglia
D) Associated with tau protein abnormalities
E) Associated with apolipoprotein E
Alzheimer's disease is characterized by the presence of amyloid plaques and neurofibrillary
tangles, involving tau protein abnormalities. It is associated with apolipoprotein E,
292 Pathology Questions and Answers, Prof.Dr. Murat ALPER
particularly the ApoE4 allele. The disease primarily affects the cerebral cortex rather than the
basal ganglia, which is more characteristic of Huntington's disease. (Answer C)
40. Which of the following is not true for Parkinson's disease?
A) Characterized by the degeneration of dopamine-producing neurons
B) Associated with Lewy bodies
C) Characterized by resting tremors
D) Associated with neurofibrillary tangles
E) Associated with rigidity and bradykinesia
Parkinson's disease is characterized by the degeneration of dopamine-producing neurons in
the substantia nigra, leading to symptoms such as resting tremors, rigidity, and bradykinesia.
Lewy bodies are a pathological hallmark of Parkinson's disease. Neurofibrillary tangles are
not associated with Parkinson's disease; they are a feature of Alzheimer's disease. (Answer
D)
41. Which of the following is not true for Huntington's disease?
A) Characterized by trinucleotide repeat expansion
B) Associated with choreiform movements
C) Characterized by neurofibrillary tangles
D) Characterized by neuronal degeneration
E) Associated with cognitive decline
Huntington's disease is a genetic disorder characterized by a trinucleotide repeat expansion
in the huntingtin gene, leading to neuronal degeneration, choreiform movements, and
cognitive decline. Neurofibrillary tangles are not a feature of Huntington's disease; they are
associated with Alzheimer's disease. (Answer C)
42. Which of the following is not true for Amyotrophic Lateral Sclerosis (ALS)?
A) Characterized by degeneration of motor neurons
B) Associated with muscle weakness and atrophy
C) Characterized by demyelination of peripheral nerves
293 Pathology Questions and Answers, Prof.Dr. Murat ALPER
D) Associated with spasticity and hyperreflexia
E) Associated with dysphagia and respiratory failure
Amyotrophic lateral sclerosis (ALS) is characterized by the degeneration of motor neurons in
the brain and spinal cord, leading to muscle weakness, atrophy, spasticity, hyperreflexia,
dysphagia, and respiratory failure. ALS primarily affects motor neurons rather than peripheral
nerve myelin. (Answer C)
43. Which of the following is not true for Creutzfeldt–Jakob disease?
A) Associated with prion protein
B) Characterized by spongiform changes in the brain
C) Characterized by slowly progressive dementia
D) Associated with myoclonus
E) Associated with rapidly progressive neurodegeneration
Creutzfeldt–Jakob disease (CJD) is a prion disease characterized by rapidly progressive
neurodegeneration, spongiform changes in the brain, and myoclonus. It typically progresses
rapidly, unlike other forms of dementia, which have a slower progression. (Answer C)
44. Which of the following is not true for astrocytomas?
A) Most common primary brain tumor in adults
B) Typically arise in the cerebral hemispheres
C) Characterized by rapid progression and poor prognosis
D) Associated with neurofibrillary tangles
E) Associated with infiltration into surrounding brain tissue
Astrocytomas are the most common primary brain tumors in adults, typically arising in the
cerebral hemispheres. They vary in their progression and prognosis, ranging from slow-
growing pilocytic astrocytomas to aggressive glioblastomas. They are associated with
infiltration into surrounding brain tissue. Neurofibrillary tangles are not associated with
astrocytomas; they are a feature of Alzheimer's disease. (Answer D)
294 Pathology Questions and Answers, Prof.Dr. Murat ALPER
45. Which of the following is not true for meningiomas?
A) Typically benign
B) More common in women
C) Derived from arachnoid cap cells
D) Typically arise within the brain parenchyma
E) Can cause seizures
Meningiomas are typically benign tumors derived from arachnoid cap cells and are more
common in women. They usually arise from the meninges, not within the brain parenchyma,
and can cause seizures due to compression of adjacent brain tissue. (Answer D)
46. Which of the following is not true for oligodendrogliomas?
A) Most commonly occur in the frontal lobes
B) Characterized by calcifications
C) Associated with a favorable prognosis
D) Derived from glial cells
E) Most commonly seen in children
Oligodendrogliomas are tumors derived from glial cells, most commonly occurring in the
frontal lobes of adults. They are characterized by calcifications and have a relatively favorable
prognosis compared to other gliomas. They are less common in children compared to adults.
(Answer E)
47. Which of the following is not true for ependymomas?
A) Derived from ependymal cells
B) Most commonly occur in the ventricles
C) Most common in adults
D) Can cause hydrocephalus
E) Associated with spinal cord tumors
295 Pathology Questions and Answers, Prof.Dr. Murat ALPER
Ependymomas are tumors derived from ependymal cells that line the ventricles and the
central canal of the spinal cord. They most commonly occur in the ventricles and can cause
hydrocephalus due to obstruction of cerebrospinal fluid flow. Ependymomas are more
common in children and adolescents, particularly in the posterior fossa. They are also
associated with spinal cord tumors in adults. (Answer C)
48. Which of the following is not true for schwannomas?
A) Derived from Schwann cells
B) Most commonly occur on cranial nerve VIII
C) Associated with neurofibromatosis type 2
D) Typically malignant
E) Can cause hearing loss
Schwannomas, also known as acoustic neuromas, are benign tumors derived from Schwann
cells, most commonly occurring on cranial nerve VIII (the vestibulocochlear nerve). They are
associated with neurofibromatosis type 2 and can cause hearing loss due to compression of
the auditory nerve. Schwannomas are typically benign, not malignant. (Answer D)
49. Which of the following is not true for neurofibromatosis type 1?
A) Characterized by café-au-lait spots
B) Associated with Lisch nodules
C) Associated with bilateral vestibular schwannomas
D) Associated with neurofibromas
E) Associated with optic gliomas
Neurofibromatosis type 1 (NF1), also known as von Recklinghausen disease, is characterized
by café-au-lait spots, Lisch nodules, neurofibromas, and optic gliomas. Bilateral vestibular
schwannomas are characteristic of neurofibromatosis type 2 (NF2), not NF1. (Answer C)
50. Which of the following is not true for tuberous sclerosis?
A) Characterized by cortical tubers
B) Associated with renal angiomyolipomas
296 Pathology Questions and Answers, Prof.Dr. Murat ALPER
C) Associated with cardiac rhabdomyomas
D) Associated with bilateral vestibular schwannomas
E) Characterized by subependymal nodules
Tuberous sclerosis is a genetic disorder characterized by the presence of cortical tubers,
subependymal nodules, renal angiomyolipomas, and cardiac rhabdomyomas. Bilateral
vestibular schwannomas are not associated with tuberous sclerosis; they are characteristic of
neurofibromatosis type 2. (Answer D)
51. Which of the following is not true for Von Hippel–Lindau disease?
A) Associated with retinal hemangioblastomas
B) Associated with renal cell carcinoma
C) Associated with pheochromocytomas
D) Associated with pancreatic cysts
E) Characterized by axillary freckling
Von Hippel–Lindau disease is a genetic disorder associated with retinal hemangioblastomas,
renal cell carcinoma, pheochromocytomas, and pancreatic cysts. Axillary freckling is not
associated with Von Hippel–Lindau disease; it is a feature of neurofibromatosis type 1.
(Answer E)
52. Which of the following is true about Alzheimer's disease?
A) Characterized by the presence of Lewy bodies
B) Associated with the accumulation of amyloid plaques
C) Characterized by the degeneration of motor neurons
D) Associated with trinucleotide repeat expansion
E) Characterized by demyelination of peripheral nerves
Alzheimer's disease is characterized by the accumulation of amyloid plaques and
neurofibrillary tangles in the brain. It is not associated with Lewy bodies, which are
characteristic of Lewy body dementia and Parkinson's disease. Alzheimer's disease primarily
affects the cerebral cortex and is not associated with motor neuron degeneration,
trinucleotide repeat expansion, or peripheral nerve demyelination. (Answer B)
297 Pathology Questions and Answers, Prof.Dr. Murat ALPER
53. Which of the following is true about Parkinson's disease?
A) Characterized by the degeneration of dopamine-producing neurons
B) Associated with the accumulation of amyloid plaques
C) Characterized by trinucleotide repeat expansion
D) Associated with demyelination of peripheral nerves
E) Characterized by neurofibrillary tangles
Parkinson's disease is characterized by the degeneration of dopamine-producing neurons in
the substantia nigra, leading to symptoms such as tremors, rigidity, and bradykinesia. It is
associated with the accumulation of Lewy bodies, not amyloid plaques or neurofibrillary
tangles. Parkinson's disease is not associated with trinucleotide repeat expansion or
peripheral nerve demyelination. (Answer A)
54. Which of the following is true about Huntington's disease?
A) Characterized by neurofibrillary tangles
B) Characterized by amyloid plaques
C) Characterized by trinucleotide repeat expansion
D) Characterized by alpha-synuclein deposition
E) Characterized by the accumulation of galactocerebroside
Huntington's disease is a genetic neurodegenerative disorder caused by a trinucleotide
repeat expansion (CAG repeats) in the huntingtin gene. This expansion leads to the
production of a mutant huntingtin protein, which causes neuronal degeneration, particularly
in the basal ganglia, leading to movement disorders and cognitive decline. It is not
characterized by neurofibrillary tangles, amyloid plaques, alpha-synuclein deposition, or the
accumulation of galactocerebroside. (Answer C)
55. Which of the following is true about Creutzfeldt–Jakob disease?
A) Caused by infectious proteins
B) Characterized by slowly progressive dementia
C) Associated with the accumulation of amyloid plaques
298 Pathology Questions and Answers, Prof.Dr. Murat ALPER
D) Characterized by trinucleotide repeat expansion
E) Associated with peripheral nerve degeneration
Creutzfeldt–Jakob disease (CJD) is a prion disease caused by infectious proteins known as
prions. It is characterized by rapidly progressive dementia, spongiform changes in the brain,
and myoclonus. It is not associated with amyloid plaques, trinucleotide repeat expansion, or
peripheral nerve degeneration. (Answer A)
Here are the English translations for questions 56 to 110:
---
56. Which of the following is not a major histopathological finding in Alzheimer's disease?
A) Neurofibrillary tangles
B) Senile plaques
C) Granulovacuolar degeneration
D) Hirano bodies
E) Polymorphonuclear leukocyte infiltration
Alzheimer's disease presents in the 50s with progressive loss of intellectual capacity, mood
changes, and behavioral alterations. It is the most common cause of dementia in the elderly
and is associated with trisomy 21, where amyloid precursor protein is produced. Cortical
atrophy and ventricular enlargement are observed. Histologically, neurofibrillary tangles,
senile plaques, and amyloid angiopathy are seen. Granular vacuolar degeneration and Hirano
bodies are present. Neutrophil infiltration is not. Lewy bodies, seen in Parkinson's, can also
be found in Alzheimer's disease. (Answer: E)
---
57. Which of the following is not a characteristic of multiple sclerosis?
299 Pathology Questions and Answers, Prof.Dr. Murat ALPER
A) Association with various HLA haplotypes
B) Axonal degeneration
C) Frequency varies by geographic region
D) Numerous oligoclonal immunoglobulin bands in cerebrospinal fluid
E) Predilection for optic nerve, brainstem, and periventricular areas
Multiple sclerosis is the most common demyelinating disease, characterized by the
preservation of axons and the destruction of myelin. (Answer: B)
---
58. Intracranial tumors most commonly develop from which of the following?
A) Microglia
B) Neurons
C) Astrocytes
D) Oligodendroglia
E) Ependymal cells
About 1/3 to 1/4 of intracranial tumors are primary. In children, they account for up to 20%
of pediatric cancers, and in adults, the incidence is 1-2 per 100,000. Glioblastoma multiforme
and astrocytoma are the most common primary brain tumors, originating from astrocytes.
Gliomas reflect the diverse morphology of glial cells and originate from astrocytes,
oligodendrocytes, and ependymal cells. (Answer: C)
---
59. Which of the following is the most common primary brain tumor in children under five?
A) Medulloblastoma
B) Ependymoma
300 Pathology Questions and Answers, Prof.Dr. Murat ALPER
C) Astrocytoma
D) Oligodendroglioma
E) Meningioma
Astrocytoma is the most common adult CNS tumor, frequent in middle age, and also the
most common CNS tumor in children. The malignant behavior is seen in glioblastoma
multiforme. Oligodendroglioma accounts for 5-15% of gliomas. Medulloblastomas represent
25% of childhood brain tumors and are primarily seen in the cerebellum. They are the most
frequent tumors in the first few years of life in this age group. (Answer: C)
---
60. Which of the following tumors is less frequently seen in childhood?
A) Astrocytoma
B) Oligodendroglioma
C) Ependymoma
D) Medulloblastoma
E) Craniopharyngioma
More than 50% of childhood tumors are astrocytomas, most of which are cerebellar.
Ependymoma accounts for 8%, medulloblastoma 20-25%, craniopharyngioma 5-10%, and
oligodendroglioma less than 1-2%. (Answer: B)
---
61. Which of the following constitutes 80% of all gliomas?
A) Astrocytoma
B) Ependymoma
C) Oligodendroglioma
301 Pathology Questions and Answers, Prof.Dr. Murat ALPER
D) Medulloblastoma
E) Meningioma
Together with glioblastoma multiforme, astrocytomas constitute 80% of all gliomas. Clinical
findings include headaches, seizures, and focal neurological deficits, depending on the site of
involvement. Astrocytomas have various clinicopathological subgroups with distinct features.
The most common type is diffuse (fibrillary) astrocytoma, comprising more than half of brain
tumors in adults. These astrocytomas are graded as anaplastic (malignant) astrocytomas and
glioblastoma multiforme. Clinical findings rapidly progress, reflecting histological progression.
Histological criteria include cellularity, nuclear and cytoplasmic pleomorphism, mitotic
activity, microvascular proliferation, and necrosis, increasing with grade. In a typical
anaplastic astrocytoma, the presence of necrosis (not single-cell necrosis) indicates
glioblastoma, signaling poor prognosis. A four-stage numeric grading system (St. Anne-Mayo)
is proposed recently, where each histological variable is calculated as one point in the
grading: nuclear atypia, mitosis, microvascular proliferation, and necrosis. If none of these
histological changes are present, the tumor is grade I, one variable is grade II, two variables
are grade III, and three or four variables, especially with microvascular proliferation and
necrosis, grade IV. A higher score indicates poor prognosis. Based on the resemblance of
neoplastic cells to normal protoplasmic, fibrillary, and gemistocytic astrocytes, they are
subdivided into protoplasmic, fibrillary, and gemistocytic astrocytomas. Protoplasmic
astrocytomas typically occur in children and young adults. They usually expand the cortex
from which they originate and have a gelatinous appearance. Microscopically, tumor cells are
somewhat star-like and often loosely arranged in a sparse, fibrillary, and microcystic stroma.
Fibrillary (diffuse) astrocytomas are the most common glial tumors, accounting for
approximately 80% of adult primary brain tumors, affecting the cerebral hemispheres in
adults, and brainstem in children and adolescents, causing expansion and gray discoloration
of the brain tissue. The tumor is fibrous in consistency, and the boundary between normal
and abnormal tissues is indistinct. Approximately 15% of diffuse astrocytomas show spot-like
calcifications and small calcospheres in blood vessels. Distinguishing a low-grade astrocytoma
from chronic astrocytosis (gliosis) can be challenging due to mild to moderate cellular
proliferation and nuclear irregularity. The average survival is 8 years in some series. (Answer:
A)
---
62. Which is the most common brain tumor among the following?
A) Glioblastoma multiforme
B) Oligodendroglioma
302 Pathology Questions and Answers, Prof.Dr. Murat ALPER
C) Ependymoma
E) Hemangioblastoma
In children, the presence of many undifferentiated cells results in more frequent primitive
CNS tumors. In adults, with a lack of such reserve cells and only glial cells, especially
astrocytes, having proliferative ability, glial tumors are more common. Astrocytomas are the
most common tumors, and the astrocytoma, anaplastic astrocytoma, and glioblastoma
multiforme group make up 80–90% of all adult glial tumors. Astrocytomas may evolve into
glioblastoma multiforme over time. CNS tumors in children tend to be infratentorial, while
tumors in adults are mostly supratentorial. Overall, gliomas are twice as common in males
than females, while meningiomas and acoustic schwannomas are more frequent in females.
Glial tumors are not encapsulated. More differentiated gliomas, especially astrocytomas,
have indistinct borders and eventually kill the patient due to their infiltrative growth, the
inability to perform radical surgical excision, or infiltration into vital areas preventing surgery.
Strategically located histologically benign CNS tumors threaten life as much as malignant
ones. Relatively well-demarcated anaplastic tumors do not respond well to surgical
treatment and radiotherapy. Gliomas invade adjacent brain tissue and infiltrate the
meninges, sometimes causing intense desmoplastic reactions that complicate diagnosis.
Gliomas have a tendency to "metastasize" via CSF. This characteristic is not exclusive to
malignant tumors but is sometimes observed in "benign" tumors like choroid plexus
papillomas. Extracranial metastasis of CNS tumors is rare due to the absence of lymphatic
structures in other body parts and anaplastic tumors causing early death by increasing
intracranial pressure before organ metastases develop. When a glial tumor spreads outside
the cranium following a craniotomy or shunt surgery, the most common sites of
dissemination are the lungs, cervical lymph nodes, and coelomic cavity. Oligodendroglioma
constitutes 5–15% of gliomas, most frequently seen in the 4th and 5th decades, primarily
affecting the frontal and temporal lobes of the brain, exhibiting calcification and a fried egg
appearance. The presence of astrocytoma foci in oligodendroglioma determines prognosis
based on their degree of anaplasia. Hemangioblastoma, seen in Von-Hippel Lindau
Syndrome, secretes erythropoietin causing polycythemia. Medulloblastoma, common in
children under ten, affects primarily the cerebellum and can spread via CSF. (Answer: A)
---
63. Which of the following tumors is of a lower grade?
A) Protoplasmic astrocytoma
B) Pilocytic astrocytoma
303 Pathology Questions and Answers, Prof.Dr. Murat ALPER
C) Gemistocytic astrocytoma
D) Anaplastic astrocytoma
E) Glioblastoma multiforme
Fibrillary astrocytoma is the most common variant of astrocytoma and is
Grade II. Choroid plexus papilloma, pilocytic astrocytoma, subependymoma, ganglioglioma,
and gangliocytoma are Grade I tumors. Most meningiomas are grade I. Pleomorphic
xanthoastrocytoma, oligodendroglioma, ependymoma, protoplasmic astrocytoma, and
gemistocytic astrocytoma are Grade II. Anaplastic Astrocytoma, anaplastic meningioma,
Pineocytoma, Pineoblastoma, anaplastic oligodendroglioma, and anaplastic ependymoma
are Grade III, and medulloblastoma, Glioblastoma multiforme, choroid plexus carcinoma, and
neuroblastoma are Grade IV tumors. (Answer: B)
---
64. In the differential diagnosis between anaplastic astrocytoma and glioblastoma
multiforme, which of the following is more valuable?
A) Mitosis
B) Necrosis
C) Cellularity
D) Pleomorphism
E) Increased proliferation
While all other findings are present in both tumors, the presence of necrosis shifts the
diagnosis towards glioblastoma multiforme. Glioblastoma multiforme accounts for 10–20%
of all intracranial tumors and is most common in the fifth and sixth decades. Like anaplastic
astrocytoma, they can arise de novo or from astrocytomas; about 6% of cases are multifocal.
The most common site is the cerebral hemispheres, but they tend to affect the brainstem in
less frequently affected age groups like infants and adolescents. The tumor enlarges the
affected region and often crosses the corpus callosum to the opposite hemisphere, creating
the classic butterfly pattern. Microscopically, tumor cells can be uniform or exhibit diverse
cellular features across a range of anaplasia. Neoplastic cells may be small and spindly or
large, bizarre-shaped, and multinucleated. Necrosis and microvascular proliferation are
304 Pathology Questions and Answers, Prof.Dr. Murat ALPER
diagnostic features of glioblastoma. Tumor necrosis varies from small foci to large, map-like
areas, and viable primitive cells often line up vertically to form a false or real fence pattern.
Vascular changes include hypertrophy and hyperplasia of capillary endothelial cells,
sometimes forming glomerular-like structures. The prognosis for glioblastomas is very poor,
with survival ranging from 6 to 18 months. (Answer: B)
---
65. Which of the following brain tumors is more frequently seen in childhood?
A) Astrocytoma
B) Oligodendroglioma
C) Ependymoma
D) Craniopharyngioma
E) Medulloblastoma
Astrocytoma is the most common brain tumor in children. While adult astrocytomas are
frequently seen in the cerebrum, they are more commonly located in the cerebellum in
children. Craniopharyngioma and medulloblastoma are also common in children.
Ependymoma is the most common tumor in the spinal canal but occurs 5–6% intrcranially.
(Answer: A)
---
66. A 15-year-old patient with a cystic tumor of benign biological behavior, typically located
in the cerebellum, third ventricular floor, and walls, or optic chiasm, and containing
Rosenthal fibers microscopically, is most likely diagnosed with which of the following?
A) Anaplastic astrocytoma
B) Pilocytic astrocytoma
C) Medulloblastoma
D) Ependymoma
E) Protoplasmic astrocytoma
305 Pathology Questions and Answers, Prof.Dr. Murat ALPER
(Answer: B)
---
67. Which type of astrocytoma is most commonly seen in children and young adults,
frequently located in the cerebellum, optic nerve, chiasm, third ventricular walls, and less
often in the brainstem and spinal cord, with an excellent prognosis when occurring in the
cerebellum?
A) Pleomorphic xanthoastrocytoma
B) Pilocytic astrocytoma
C) Fibrillary astrocytoma
D) Gemistocytic astrocytoma
E) Anaplastic astrocytoma
Pilocytic astrocytomas are commonly found in children and young adults and are most
frequently located in the cerebellum; optic nerve and chiasm (optic gliomas), and third
ventricular walls (infundibuloma) are midline structures, and less often in the brainstem and
spinal cord. They are more common in the cerebral hemispheres, especially the temporal
lobe, in young adults. They are well-circumscribed and lobulated. Cyst formation is common,
especially in cerebellar tumors. Juvenile pilocytic astrocytomas consist of tightly packed
pilocytic astrocytes and loosely arranged stellate cells. Pilocytic astrocytes are hair-like
spindle cells with long GFAP-reactive processes. A noteworthy feature is the presence of
granular eosinophilic bodies and dark eosinophilic sausage-like Rosenthal fibers, although
Rosenthal fibers can also be seen in reactive gliosis. They do not show IDH1 and IDH2
mutations. The prognosis of pilocytic astrocytoma is excellent, especially when located in the
cerebellum, with most patients living over 20 years. (Answer: B)
---
68. In a 40-year-old man with a tumor in the cerebral hemispheres, strong GFAP
immunoreactivity due to numerous intermediate filaments is observed. The prognosis is
poor due to 80% of cases progressing to glioblastoma. Which tumor is this?
306 Pathology Questions and Answers, Prof.Dr. Murat ALPER
A) Meningioma
B) Pilocytic astrocytoma
C) Oligodendroglioma
D) Diffuse astrocytoma
E) Astroblastoma
Diffuse astrocytomas are seen in adults, in the 4th-6th decades, and occur in the cerebral
hemispheres. They show strong GFAP immunoreactivity due to numerous intermediate
filaments. About 80% of cases progress to glioblastoma, indicating a poor prognosis, with
most patients dying within 3.5 years of diagnosis. (Answer: D)
---
69. Which tumor is frequently associated with tuberous sclerosis and occurs in the first two
decades of life?
A) Pleomorphic xanthoastrocytoma
B) Pilocytic astrocytoma
C) Fibrillary astrocytoma
D) Gemistocytic astrocytoma
E) Subependymal giant cell astrocytoma
Subependymal giant cell astrocytoma (SEGA) is commonly seen in the first two decades of
life and is associated with tuberous sclerosis. Sometimes it can occur without overt signs of
the disease. This tumor originates from the lateral ventricles and frequently obstructs the
foramen of Monro. Histologically, it is vascular-rich with focal calcifications. It consists of
fusiform or band-like cells with large eosinophilic-hyaline cytoplasm. This tumor is quite
benign, but it presents acutely due to blocking CSF circulation; few recur after resection. In
tuberous sclerosis, the tumor most frequently associated with it is rhabdomyoma in the
heart and angiomyolipoma in the kidney. (Answer: E)
---
307 Pathology Questions and Answers, Prof.Dr. Murat ALPER
70. In young individuals with a long history of seizures and a tumor located mostly in the
temporal lobes, extending to involve both the cortex and the pia-arachnoid membranes with
a yellow, firm consistency and cysts macroscopically, what is the most likely diagnosis?
A) Pleomorphic xanthoastrocytoma
B) Pilocytic astrocytoma
C) Fibrillary astrocytoma
D) Gemistocytic astrocytoma
E) Anaplastic astrocytoma
Pleomorphic xanthoastrocytomas (PXA) typically occur in young individuals with a long
history of seizures and are primarily located in the temporal lobes. They are superficially
located to involve both the cortex and pia-arachnoid membranes and often present with
cysts and typically have a yellow, firm consistency. Microscopically, the tumor is cell-rich and
consists of cells showing pleomorphism, ranging from large globoid to spindle-shaped cells
and variable amounts of foamy cytoplasm. The large cells usually have bizarre,
hyperchromatic nuclei or appear multinucleated. Mitosis and necrosis are rare, but if
present, they indicate a poor prognosis for this generally slow-growing tumor. (Answer: A)
---
71. Which of the following is a neural system tumor often located superficially in the
temporal lobe in children and associated with a history of seizures?
A) Pilocytic astrocytoma
B) Pleomorphic xanthoastrocytoma
C) Brainstem glioma
D) Oligodendroglioma
Pleomorphic xanthoastrocytoma is a tumor located superficially in the temporal lobe in
children and young adults, often associated with a history of seizures. Sometimes lipidized
astrocytes, abundant reticulin deposition, and chronic inflammatory cells are present. In
some cases, anaplastic transformation may occur. (Answer: B)
308 Pathology Questions and Answers, Prof.Dr. Murat ALPER
---
72. Which of the following central nervous system tumors frequently shows calcification?
A) Pilocytic astrocytoma
B) Schwannoma
C) Oligodendroglioma
D) Neuroblastoma
E) Glioblastoma multiforme
(Answer: C)
---
73. In a tumor with microscopically closely packed cells forming a "honeycomb" appearance,
with well-defined cytoplasmic borders, round cells with
perinuclear halo formation creating a characteristic "fried egg" appearance, and no staining
with GFAP, what is the likely diagnosis?
A) Ependymoma
B) Pilocytic astrocytoma
C) Oligodendroglioma
D) Gemistocytic astrocytoma
E) Glioblastoma multiforme
Oligodendrogliomas, tumors of middle age, are most commonly seen in the deep
frontotemporal white matter and constitute 5–15% of all gliomas. They are WHO grade II
tumors. They tend to grow towards the surface and infiltrate the cortex. They are relatively
well-circumscribed, gelatinous, and grayish, with cysts, focal mineralization, necrosis, and
hemorrhage. Microscopically, the tumor consists of closely packed cells giving a
"honeycomb" appearance, with well-defined cytoplasmic borders and round cells. Typically,
309 Pathology Questions and Answers, Prof.Dr. Murat ALPER
the nuclei are round, with fine chromatin and a small nucleolus. The cytoplasm is often clear,
and perinuclear halo formation creates the characteristic "fried egg" appearance if the tissue
is not properly fixed. Perineuronal satellitosis may occur. Typically, tumor cells do not stain
with GFAP. In 90% of cases, calcifications and calcospheres are observed, visible in 40% of
patients' plain skull X-rays. Sometimes calcification is so dense that it partially obliterates the
tumor. In neurological imaging techniques, oligodendrogliomas appear as well-circumscribed
hypodense areas, enhancing with contrast only if there is vascular proliferation. Grade III
tumors indicate a poorer prognosis with features like mitosis, anaplasia, increased cellularity,
and microvascular proliferation. (Answer: C)
---
74. In a 45-year-old patient with several years of neurological disturbances and a history of
seizures, which tumor is more likely to be observed?
A) Oligodendroglioma
B) Ependymoma
C) Pilocytic astrocytoma
D) Pleomorphic xanthoastrocytoma
E) Glioblastoma multiforme
Oligodendrogliomas constitute 5–15% of gliomas and are most common in the 4th–5th
decades. Patients often present with several years of neurological disturbances and frequent
seizures. According to WHO, they are grade II/IV lesions. Lesions are frequently located in the
cerebral hemispheres and white matter. They form a well-circumscribed, gelatinous, gray
mass. Cysts, focal hemorrhage, and calcification are common. Calcification is found in 90% of
cases. Their prognosis is better than astrocytomas. Isocitrate dehydrogenase (IDH) gene
mutations are common in grade II astrocytomas and oligodendrogliomas. Telomerase
mutations, contributing to the immortality of tumor cells, are observed in glioblastoma and
other astrocytic tumors. IDH-mutated tumors rarely show telomerase mutations but often
exhibit ATRX mutations. Those with 1p and 19q loss respond better to treatment. Anaplastic
oligodendrogliomas (grade III/IV) show higher cellularity, nuclear anaplasia, and mitotic
activity, with frequent microvascular proliferation, indicating a poor prognosis. (Answer: A)
---
310 Pathology Questions and Answers, Prof.Dr. Murat ALPER
75. Which tumor is typically located near the fourth ventricle in the first two decades of life?
A) Astrocytoma
B) Medulloblastoma
C) Glioblastoma
D) Ependymoma
Ependymomas develop in the first two decades of life, typically near the fourth ventricle,
accounting for 5–10% of tumors in this age group. In adults, the most common location is the
spinal cord, particularly frequent in Neurofibromatosis-2. They are solid and papillary
masses. Their proximity to the pons and medullary nuclei often complicates surgical removal.
Microscopically, perivascular pseudorosettes are observed. 50% are GFAP (+). The tumor is
well-differentiated. The prognosis is poor due to progressive hydrocephalus from fourth
ventricle obstruction, despite the absence of anaplasia. Myxopapillary ependymoma is seen
in the filum terminale of the spinal cord. Subependymomas are solid and sometimes
calcified, growing slowly, typically found incidentally at autopsy, most commonly located in
the lateral and fourth ventricles. Choroid plexus papilloma is common in the lateral ventricles
in children and the fourth ventricle in adults. (Answer: D)
---
76. Which tumor, located in the fourth ventricle and capable of forming solid or papillary
structures, is typically difficult to remove due to proximity to the medulla and pons nuclei in
an 18-year-old male?
A) Ependymoma
B) Medulloblastoma
C) Anaplastic astrocytoma
D) Meningioma
E) Oligodendroglioma
Ependymoma arises from the epithelium lining the ventricles and the canal descending along
the spinal cord. It is most common in the first two decades, typically in the fourth ventricle,
constituting 5-10% of cases in this age group. In middle age, the spinal cord is the most
311 Pathology Questions and Answers, Prof.Dr. Murat ALPER
frequently affected site, comprising most intramedullary tumors. A tumor in the fourth
ventricle presents as solid or papillary masses, which are critical and challenging to remove,
while spinal cord ependymomas are easier to excise. (Answer: A)
---
77. In a child with a tumor located in the fourth ventricle, which tumor is easier to distinguish
from the surrounding nerve tissue and thus facilitates surgical intervention compared to
astrocytomas?
A) Ependymoma
B) Pilocytic astrocytoma
C) Oligodendroglioma
D) Gemistocytic astrocytoma
E) Glioblastoma multiforme
Ependymomas account for 4 to 6% of all gliomas across all age groups, and about 5-10% of
childhood and adolescent tumors. They can occur anywhere the ventricles are found; 60-70%
are infratentorial. The fourth ventricle is the most common location, presenting as an
exophytic mass. Supratentorial ependymomas tend to develop into adjacent brain
parenchyma rather than within the ventricles. Ependymomas are the most common
intraspinal tumors. Ependymomas are better circumscribed from surrounding nerve tissue
than astrocytomas, facilitating surgical intervention. This diagnostic difference is especially
significant in spinal cord surgery, determining the limits of resection. Histologically, they form
"perivascular rosettes" around blood vessels. The nuclei of the cells are equidistant from the
vessel, with fibrillar material filling the space between. Sometimes this fibrillation can be
very prominent (tanycytic), mimicking pilocytic astrocytomas, posing a challenging problem
for neuropathologists during surgery. In fewer cases, some neoplastic cell clusters exhibit
epithelial characteristics, forming canalicular or slit-like spaces lined with –"ependymal
rosettes"– with prominent blepharoplasts. The prognosis of ependymomas is difficult to
predict, but it is said to be better in "non-invasive tumors" when completely excised and in
adolescents compared to infants. For instance, spinal ependymomas can be more completely
removed, while those located in the fourth ventricle in infants have a poorer prognosis. The
role of radiotherapy in prolonging life expectancy and preventing ependymoma
dissemination through CSF after surgery is debatable. (Answer: A)
---
312 Pathology Questions and Answers, Prof.Dr. Murat ALPER
78. Blepharoplasts are pathognomonic for which of the following?
A) Ependymoma
B) Pilocytic astrocytoma
C) Herpes zoster virus
D) Oligodendroglioma
E) Schwannoma
In ependymoma, tumor cells form ependymal rosettes and perivascular pseudorosettes.
They contain blepharoplasts, which are stained with PTAH and are pathognomonic. Pilocytic
astrocytoma shows Rosenthal fibers and is benign in behavior. Oligodendroglioma consists of
small uniform cells often with perinuclear halos, and mitosis is rare. (Answer: A)
---
79. Which tumor is typically seen in the third to fourth decades and almost exclusively in the
filum terminale, capable of secondarily affecting nerve roots in the conus medullaris and
cauda equina?
A) Subependymomas
B) Myxopapillary ependymoma
C) Ependymoblastoma
D) Malignant ependymoma
E) Oligodendroglioma
Subependymomas are generally located in the fourth and lateral ventricles, often found
incidentally in autopsies of older patients. Depending on their location and size, they may
become symptomatic by obstructing CSF flow and causing hydrocephalus. Myxopapillary
ependymoma is a distinct subtype seen almost exclusively in the filum terminale, capable of
secondarily affecting nerve roots in the conus medullaris and cauda equina. It is typically
seen in the third to fourth decades, forming a large, sausage-like central mass surrounding
and displacing the nerve roots. Classically, neoplastic ependymal cells are often cuboidal and
313 Pathology Questions and Answers, Prof.Dr. Murat ALPER
are arranged circularly around a hyaline or myxoid matrix, frequently containing a central
vessel. The prognosis is
excellent after complete excision of the tumor. Recurrences or distant metastases occur
when the tumor is not entirely removed or if bone erosion has occurred. Ependymoblastoma
should be distinguished from anaplastic ependymomas. They are large embryonal, primitive
tumors almost always seen in childhood, invading surrounding tissues and spreading via CSF,
despite appearing well-circumscribed macroscopically. (Answer: B)
---
80. In a 20-year-old patient with gait and movement disturbances and a cerebellar mass
observed on CT, the fluid analysis reveals erythropoietin. What is the most likely diagnosis?
A) Schwannoma
B) Hemangioblastoma
C) Angiosarcoma
D) Medulloblastoma
E) Ependymoma
Ependymoma is most frequently seen in the fourth ventricle. Medulloblastoma is common in
those under 14 and occurs in the cerebellum. Its frequency decreases in later years.
Schwannoma, especially those affecting the 8th cranial nerve, are known as acoustic
neuromas, leading to unilateral sensorineural hearing loss. Hemangioblastoma is most
commonly found in the cerebellum in young adults, accounting for 7% of primary cerebellar
tumors. Gait and movement disturbances are observed. Cysts may be present, and fluid
analysis may reveal erythropoietin, occasionally causing polycythemia. It may be part of Von
Hippel Lindau syndrome. (Answer: B)
---
81. Which tumor is more frequently seen in children, located in the lateral ventricle in
children and in the fourth ventricle in adults?
314 Pathology Questions and Answers, Prof.Dr. Murat ALPER
A) Colloid cyst
B) Ependymoma
C) Subependymoma
D) Oligodendroglioma
E) Choroid plexus papilloma
Choroid plexus papillomas are most commonly seen in children. In children, they are
frequent in the lateral ventricle, while in adults, they are more frequent in the fourth
ventricle. They form papillary structures. The colloid cyst is a non-neoplastic lesion located in
the third ventricle in young adults. (Answer: E)
---
82. Which tumor, capable of excessive CSF production and with some studies suggesting a
role for papovavirus in its etiology, is described below?
A) Ependymoma
B) Oligodendroglioma
C) Myxopapillary ependymoma
D) Gangliocytoma
E) Choroid plexus papilloma
In choroid plexus papilloma, hydrocephalus develops due to tumor obstruction or sometimes
excessive CSF production. Choroid plexus carcinoma typically develops in children and may
be associated with Li-Fraumeni syndrome, with an adenocarcinoma-like appearance.
(Answer: E)
---
83. Which tumor is more frequently seen in the posterior fossa in childhood?
A) Meningioma
315 Pathology Questions and Answers, Prof.Dr. Murat ALPER
B) Acoustic neuroma
C) Medulloblastoma
D) Cystic astrocytoma
E) Glioblastoma multiforme
Medulloblastoma, frequently seen in childhood, is primarily infratentorial and can spread via
CSF. It is the most common CNS tumor in children under two years old. Some sources state
that astrocytoma is the most common. (Answer: C)
---
84. Which tumor, often seen in children and spread via CSF, is described below?
A) Ependymoma
B) Medulloblastoma
C) Anaplastic astrocytoma
D) Meningioma
E) Pilocytic astrocytoma
Medulloblastoma is a frequently seen nervous system tumor in children under 14, more
common in the cerebellum. In young children, it is typically located in the midline at the
vermis, while in older patients, it is more lateral, affecting the hemispheres. It spreads via
CSF, forming widespread tumor masses in the ependyma and subarachnoid space. It is
clinically and pathologically malignant. However, it is radiosensitive, and with chemotherapy
and radiotherapy, the five-year survival rate can reach 75%. (Answer: B)
---
85. Which tumor, localized in the cerebellum and frequently seen in childhood, consists of
primitive-appearing cells microscopically and forms Homer Wright rosettes?
A) Glioblastoma
316 Pathology Questions and Answers, Prof.Dr. Murat ALPER
B) Ependymoma
C) Pilocytic astrocytoma
D) Astroblastoma
E) Medulloblastoma
(Answer: E)
---
86. Which of the following is a central nervous system tumor of primitive neuroectodermal
origin?
A) Ependymoma
B) Oligodendroglioma
C) Medulloblastoma
D) Chordoma
E) Germinoma
Medulloblastoma is primarily seen in children and predominantly located in the midline of
the cerebellum. It is a highly cellular and malignant tumor consisting of small blue round
anaplastic cells, capable of spreading via CSF. Homer-Wright rosettes seen in neuroblastoma
can also be seen in medulloblastoma. There is a loss of material in the short arm of
chromosome 17. Pineoblastoma is also part of the PNET group, but it is primarily
supratentorial. Ewing's sarcoma is a peripherally located PNET tumor. Ependymoblastoma
and medullary epithelioma are also part of the PNET group. (Answer: C)
---
87. Which primary central nervous system tumor is typically located in the pineal region?
A) Meningioma
B) Medulloblastoma
317 Pathology Questions and Answers, Prof.Dr. Murat ALPER
C) Oligodendroglioma
D) Ependymoma
E) Germ cell tumor
Primary brain germ cell tumors are located in the midline, most frequently in the pineal and
suprasellar regions. The most common type is germinoma, similar to seminoma. It is
common in the first two decades and more frequent in males. (Answer: E)
---
88. In a 60-year-old male patient, what is the most common type of lymphoma seen in the
brain?
A) Burkitt lymphoma
B) Lymphoblastic lymphoma
C) Mycosis fungoides
D) Follicular lymphoma
E) Diffuse large cell lymphoma
Central nervous system lymphomas are primarily diffuse large B-cell lymphomas. They are
common in immunosuppressed individuals and are almost entirely EBV positive. They behave
aggressively in the brain and are CD20 positive. (Answer: E)
---
89. From which of the following do meningiomas originate?
A) Ventricular lining
B) Neurons
C) Vascular endothelium
D) Schwann cells
318 Pathology Questions and Answers, Prof.Dr. Murat ALPER
E) Arachnoid meningothelial cells
Meningiomas are benign tumors more frequently seen in adults, particularly in females,
originating from arachnoid meningothelial cells. They most commonly occur in the
parasagittal convexities. (Answer: E)
---
90. Which tumor, more frequently seen in females, often containing progesterone receptors
and sometimes enlarging during pregnancy, is histologically benign and most commonly
found in the brain convexities, sphenoid bone wing, and olfactory groove?
A) Medulloblastoma
B) Oligodendroglioma
C) Meningioma
D) Astrocytoma
E) Ependymoma
Meningioma is a tumor of middle-aged adults; in less common spinal cases (8–10%), there is
a notable female predominance (10:1). An association with breast cancer may exist, and
hormone receptors (especially progesterone) have been identified, explaining why
meningiomas grow more rapidly during pregnancy. About half of the cases exhibit
monosomy 22, similar to schwannomas. A genetic link between the central form of
neurofibromatosis (NF type 2) and chromosome 22 is established. Meningiomas and
schwannomas are the most common tumors in this disease. Meningiomas most frequently
arise from leptomeningeal arachnoid cells located in parasagittal arachnoid granulations and
dural leaves of the spinal canal. For these dense, firm, dural-based growths, the most
frequent sites include parasagittal, falx, sphenoid ridge (often flattened or en plaque),
tuberculum sella (suprasellar and intrasellar), olfactory groove, optic nerve, tentorium
cerebelli, and especially the convexities of the cerebral hemispheres. The thoracic region is
the most common spinal location. Rarely, intraventricular (tela choroidea), intraosseous, or
even extracranial or extraspinal examples are seen. The most common histological subtypes
are: syncytial (cellular sheets), fibroblastic (spindle or storiform pattern spinal cells), and
transitional (syncytial and fibroblastic with whorls and psammoma bodies) types are often
mixed and have no prognostic significance. The psammomatous form is more predominant in
the spinal canal, and it can be challenging to differentiate the fibroblastic type from
schwannoma, pilocytic astrocytoma, or fibrous histiocytoma. Typically, mitotic activity and
319 Pathology Questions and Answers, Prof.Dr. Murat ALPER
atypia are low, necrosis is absent, and psammoma bodies may be present; vascularity may be
prominent in the angiomatous type, referred to as the "angioblastic" variant. They grow
quietly but
often invade dura, bone, and dural nerves. Most meningiomas are grade I/IV according to
WHO. Atypical meningiomas, particularly certain histological types (clear cell and chordoid),
are grade II/IV. Anaplastic meningiomas are grade III/IV, resembling sarcoma, while papillary
and rhabdoid meningiomas are also grade III/IV. Papillary meningioma is especially common
in children. Dural sinus invasion, although not having metastatic potential, prevents complete
tumor excision and accounts for most recurrences (about 15% in five years). (Answer: C)
---
91. Which of the following is not a phacomatosis group disease?
A) Neurofibromatosis
B) Von Hippel-Lindau disease
C) Sturge-Weber disease
D) Dandy-Walker malformation
E) Tuberous sclerosis
(Answer: D)
---
92. In which of the following is multiple meningiomas most commonly seen?
A) Neurofibromatosis type II
B) Tuberous sclerosis
C) Sturge-Weber
D) Von-Hippel Lindau
320 Pathology Questions and Answers, Prof.Dr. Murat ALPER
In neurofibromatosis type I, cafe-au-lait spots, multiple plexiform neurofibromas, optic nerve
glioma, pigmented nodules in the iris (Lisch nodules), and pheochromocytoma are seen. It is
an autosomal dominant disorder with a frequency of 1/3000, resulting from a mutation in
the neurofibromin gene located on 17q11.2. NF-1 is a tumor suppressor gene, and malignant
transformation from neurofibroma can develop. Neurofibromatosis type II is autosomal
dominant. Bilateral acoustic schwannomas and multiple meningiomas are typical. Gliomas,
especially ependymomas, form in the spinal cord. The NF-2 gene is coded on 22q12, and its
product is merlin. In NF-2, non-neoplastic nodular lesions in the form of Schwannosis,
meningoangiomatosis, and glial hamartomas are common. Type I neurofibromatosis is also
called Von-Recklinghausen's disease, while type II is known as acoustic neurofibromatosis.
Cavernous hemangiomas and pheochromocytomas are seen in Sturge-Weber syndrome.
Osler-Weber-Rendu disease is an autosomal dominant disorder characterized by multiple
aneurysmal telangiectasias, affecting the skin, oral cavity, respiratory, gastrointestinal, and
urinary tracts, as well as the liver, brain, and spleen. Rupture can result in epistaxis,
hematuria, and GI bleeding. In Von Hippel-Lindau disease, cerebellar hemangioblastomas,
and cysts in the pancreas, liver, and other organs are seen. Osler-Weber-Rendu, except for
the latter, forms phacomatoses. (Answer: A)
---
93. Which of the following rarely metastasizes to the brain?
A) Prostate cancer
B) Choriocarcinoma
C) Lung cancer
D) Breast cancer
E) Melanoma
25–30% of brain tumors are metastatic. Lung, breast, melanoma, kidney, and gastrointestinal
system metastases account for 80% of brain metastases. Rare tumors like choriocarcinoma
frequently metastasize, whereas prostate cancer hardly ever metastasizes to the brain, even
though it frequently metastasizes to bones. (Answer: A)
---
321 Pathology Questions and Answers, Prof.Dr. Murat ALPER
94. Which of the following is the most common malformative and non-neoplastic mass lesion
in the central nervous system?
A) Lipoma
B) Epidermoid cyst
C) Dermoid cyst
D) Colloid cyst
E) Craniopharyngioma
Craniopharyngioma is the most common example in this category (about 3-4% of primary
intracranial tumors). It is frequent in the first two decades. It presents as a suprasellar mass.
It accounts for 3% of all brain tumors. It is thought to develop from remnants of Rathke's
duct. Cystic degeneration is common, with the cyst containing machine oil-like fluid and
cholesterol clefts. Microscopically, it resembles a "ameloblastoma" in the tooth. It consists of
anastomosing islands of epithelial cells with typical palisading of the periphery. Squamous
metaplasia foci with widespread keratinization and "ghost cells" are seen. Calcification and
inflammatory, granulomatous reactions are common. Basal palisading in the epithelium is
crucial for differential diagnosis. It can cause symptoms of hydrocephalus, dwarfism, and
hypogonadism, indicating hypopituitarism. Recurrence is possible. It can be confused with
epidermoid cyst, carcinoma, and astrocytoma. Malignant transformation is extremely rare.
(Answer: E)
---
95. Which tumor of suprasellar location originates from Rathke's pouch?
A) Ependymoma
B) Glioblastoma
C) Craniopharyngioma
D) Meningioma
E) Medulloblastoma
(Answer: C)
322 Pathology Questions and Answers, Prof.Dr. Murat ALPER
---
96. Which of the following is a remnant of Erdheim cells?
A) Pilocytic astrocytoma
B) Craniopharyngioma
C) Schwannoma
D) Oligodendroglioma
E) Ependymoma
Craniopharyngioma is a remnant of squamous cells from Rathke's pouch (Erdheim cells). It
can be found at any level from the nasopharynx to the arachnoid covering the mammillary
body. Its peak incidence is in late childhood, early adulthood, and the fifth decade. (Answer:
B)
---
97. In a 10-year-old child, what is the most likely diagnosis when a calcified suprasellar lesion
causing classic visual disturbances and hypothalamic syndromes is observed?
A) Medulloblastoma
B) Ependymoma
C) Pituitary adenoma
D) Craniopharyngioma
E) Chordoma
Craniopharyngiomas, seen between ages 4-16, are classic calcified suprasellar lesions causing
visual disturbances and hypothalamic syndromes. They are multilocular, filled with
cholesterol-containing machine oil-like substance. They resemble ameloblastomas in
appearance. Chordoma is a tumor seen primarily in the third and fourth decades,
323 Pathology Questions and Answers, Prof.Dr. Murat ALPER
predominantly in males, and in 60% of cases, located in the sacrococcygeal region. (Answer:
D)
---
98. Which is the most common intrinsic (intramedullary) spinal cord tumor in children?
A) Ependymoma
B) Astrocytoma
C) Oligodendroglioma
D) Glioblastoma
E) Meningioma
Intrinsic (intramedullary) spinal tumors, comprising about 20% of all spinal tumors, are
mostly gliomas, primarily ependymomas and astrocytomas; glioblastomas and
oligodendrogliomas are rare. When including myxopapillary ependymomas in the filum
terminale, ependymomas are more frequent than astrocytomas. However, astrocytomas are
more common in children. These tumors most often occur in the thoracic segments of the
spinal cord, diffusely infiltrating the parenchyma, causing fusiform enlargement. Sometimes,
especially when low-grade, they cause syringomyelia in the cord. The average survival time is
two years for well-differentiated tumors and less than a year for glioblastomas. (Answer: B)
---
99. Regarding spinal tumors, which of the following is incorrect?
A) They are ten times less common than intracranial tumors
B) 70% are extramedullary
C) 30% are intramedullary
D) Most intramedullary tumors are schwannomas
E) Oligodendroglioma is very rare
324 Pathology Questions and Answers, Prof.Dr. Murat ALPER
The ratio of spinal tumors to intracranial tumors is 1/10. About 70% are extramedullary, and
30% are intramedullary. The majority of intramedullary tumors are ependymomas,
accounting for 60%. This is followed by astrocytomas at 25%. Oligodendroglioma and
glioblastoma multiforme are rare spinal cord tumors. (Answer: D)
---
100. Which of the following statements about chordomas is incorrect?
A) They grow slowly
B) They aggressively invade bone and cranial base structures
C) They are common in children
D) 60% are located in the sacrococcygeal region
E) They originate from notochord remnants
Chordomas originate from notochord remnants and are most commonly seen in the third
and fourth decades, more frequently in males. 60% are located in the sacrococcygeal region
and 30% arise from the clivus. Despite their slow growth, the tumor aggressively invades
bone and basal structures of the cranial base. Physaliphorous cells are observed. (Answer: C)
---
101. Which of the following tumors exhibits Antoni A and Antoni B areas?
A) Oligodendroglioma
B) Craniopharyngioma
C) Medulloblastoma
D) Neurofibroma
E) Neurilemmoma
Neurilemmoma (Schwannoma) is a tumor
325 Pathology Questions and Answers, Prof.Dr. Murat ALPER
arising from Schwann cells of the peripheral nerve sheath. It is most commonly located in
the cerebellopontine angle. Acoustic neuroma (8th nerve schwannoma) causes tinnitus and
hearing loss. Microscopically, Antoni A and B areas, along with Verocay bodies, are observed.
Immunohistochemically, it stains positive with S-100. Malignant transformation is extremely
rare. Malignant schwannoma, more accurately called malignant peripheral nerve sheath
tumor, is more common in the 5th cranial nerve than in the 8th. (Answer: E)
---
102. Which of the following is not a neurocutaneous syndrome?
A) Tuberous sclerosis
B) Ataxia telangiectasia
C) Von Hippel Lindau disease
D) Sturge-Weber disease
E) Adams-Hakim syndrome
Neurocutaneous syndromes (phacomatoses) are hereditary diseases characterized by tumors
and hamartomas, particularly in the nervous system. Tuberous sclerosis, neurofibromatosis,
Sturge-Weber, ataxia telangiectasia, and Von Hippel Lindau disease belong to this group.
Adams-Hakim syndrome is not part of this group. It is characterized by episodes of fainting.
Encephalofacial angiomatosis (Sturge-Weber syndrome) is a rare, non-hereditary condition
characterized by wine-colored angiomas distributed in the areas corresponding to one or
more sensory branches of the trigeminal nerve, often accompanied by ocular and
leptomeningeal angiomas. (Answer: E)
---
103. Which disease is characterized by angiofibromas, seizures, and mental retardation?
A) Von Hippel Lindau disease
B) Tuberous sclerosis
326 Pathology Questions and Answers, Prof.Dr. Murat ALPER
C) Leigh's disease
D) Amyotrophic lateral sclerosis
E) Krabbe disease
The triad of angiofibromas, seizures, and mental retardation is characteristic of tuberous
sclerosis. Cardiac rhabdomyomas and renal angiomyolipomas are common. Leigh's disease is
characterized by arrest of psychomotor development, feeding problems, seizures,
extraocular palsies, and hypotonia, with death occurring between 1-2 years of age. The
primary laboratory finding is lactic acidosis, indicating impaired conversion of pyruvate to
ATP. (Answer: B)
---
104. Which hereditary disease is associated with pigmentation disorders of the skin and
tumor formation in the nervous system, particularly in peripheral nerves?
A) Neuroma
B) Schwannoma
C) Ganglioneuroma
D) Neurofibromatosis
E) Von Hippel Lindau disease
Neurofibromatosis is an autosomal dominant disease with high penetrance. The rate of
spontaneous mutation is high. The most common form, known as Von Recklinghausen's
disease or NF-1, has its gene located on chromosome 17q11-12. It affects approximately 1 in
4000 people and is clinically characterized by multiple hyperpigmented areas (café-au-lait
spots), multiple neurofibromas, Lisch nodules (pigmented iris hamartomas), and rarely, CNS
astrocytomas. The second form, previously known as central neurofibromatosis, is now
referred to as "neurofibromatosis 2" or NF-2, with its gene located on chromosome 22q12.
NF-2 is less common than NF-1, affecting approximately 1 in 35,000 people. The disease is
characterized by bilateral acoustic schwannomas, but other intracranial and intraspinal
tumors can also occur. The presence of numerous neoplastic or hyperplastic lesions of neural
crest origin leads to the classification of neurofibromatosis as a "neurocristopathy." In
addition to schwannomas and neurofibromas, patients with this disease are predisposed to
certain gliomas: optic gliomas, pilocytic astrocytomas of the third ventricle, ependymomas,
and glioblastoma multiforme, as well as other gliomas of the brain and spinal cord. Multiple
327 Pathology Questions and Answers, Prof.Dr. Murat ALPER
meningiomas are also common. Patients with neurofibromatosis are also at increased risk of
developing non-neural malignancies such as leukemia and Wilms tumor. (Answer: D)
---
105. Which of the following is not observed in tuberous sclerosis?
A) Cortical hamartoma
B) Angiomyolipoma
C) Retinal glial hamartoma
D) Autosomal recessive inheritance
E) Subungual fibroma
Tuberous sclerosis (Bourneville's disease) is an autosomal dominant disease affecting the skin
and CNS. It causes epileptic seizures, mental retardation, and facial nodules (sebaceous
adenomas). Tumors of the heart, liver, kidney, or pancreas may accompany. Von Hippel
Lindau is a rare familial disease characterized by multiple hemangioblastomas in the CNS and
retina, malformations, and tumors of organs, especially the kidney and adrenal gland, along
with cysts in the liver, kidneys, and pancreas. Cavernous hemangiomas are common.
(Answer: D)
---
106. Which tumor is most likely to be associated with Von Hippel Lindau disease?
A) Multiple meningiomas
B) Pleomorphic xanthoastrocytoma
C) Hemangioblastoma
D) Schwannoma
E) Medulloblastoma
(Answer: C)
328 Pathology Questions and Answers, Prof.Dr. Murat ALPER
---
107. Which tumor is not found in Von Hippel Lindau disease?
A) CNS hemangioblastoma
B) Kidney malformation and tumor
C) Adrenal tumor
D) Pancreatic cysts
E) Sebaceous adenoma
Von Hippel-Lindau disease is an autosomal dominant disorder associated with the
development of renal cysts and tumors. Its extra-renal manifestations include retinal
angioma, central nervous system hemangioblastoma, pheochromocytoma, cysts, and tumors
of the liver, pancreas, and epididymal cystadenomas. Medullary thyroid carcinoma is
commonly seen in MEN II and III, while schwannomas, meningiomas, and gliomas are
frequent in Von Recklinghausen's syndrome. Cavernous hemangiomas are common in
Sturge-Weber syndrome. (Answer: E)
---
108. Which disease is characterized by B and T cell disorders, ocular and cutaneous lesions,
central nervous system anomalies, elevated alpha-fetoprotein and carcinoembryonic antigen,
with death usually resulting from infection or lymphoma?
A) Nezelof syndrome
B) Ataxia telangiectasia
C) Chronic mucocutaneous candidiasis
D) Wiskott-Aldrich syndrome
E) Severe combined immunodeficiency
(Answer: B)
329 Pathology Questions and Answers, Prof.Dr. Murat ALPER
---
109. Which statement regarding ataxia-telangiectasia is incorrect?
A) Loss of Purkinje and granule cells
B) Autosomal dominant inheritance
C) Dorsal column degeneration
D) Thymic absence
E) Increased risk of lymphoid malignancy
Ataxia-telangiectasia is an autosomal recessive disorder characterized by recurrent
infections, Purkinje cell loss, cerebellar dysfunction, and dorsal column degeneration. It is
also associated with telangiectasias in the conjunctiva and other areas, thymic absence, and
gonadal hypoplasia. (Answer: B)
---
110. Which of the following tumors is not of neuroendocrine origin?
A) Pheochromocytoma
B) Gastrinoma
C) Retinoblastoma
D) Malignant melanoma
E) Carcinoid tumor
Neuroendocrine tumors originate from neural crest cells. These include small cell lung
carcinoma, Ewing's sarcoma, primitive neuroectodermal tumors, malignant melanoma,
pheochromocytoma, carcinoid tumors, insulinomas, gastrinomas, VIPomas, and pituitary
tumors. (Answer: C)
330 Pathology Questions and Answers, Prof.Dr. Murat ALPER
---
REVIEW
(Review and write down what you remember about the following topics...)
Neurons
Astrocytes
Microglia
Oligodendroglia
Ependymal cells
Gliosis
Chromatolysis
Schwann cells
Rosenthal fibers
Meningomyelocele
Arnold-Chiari malformation
Dandy-Walker malformation
Syringomyelia
Cerebrovascular diseases
Berry aneurysms
Subarachnoid hemorrhage
Arteriovenous malformations
Cavernous hemangiomas
Venous angiomas
Capillary telangiectasias
Charcot-Bouchard aneurysm
Subarachnoid hemorrhage
331 Pathology Questions and Answers, Prof.Dr. Murat ALPER
Subdural hemorrhage
Epidural hemorrhage
Intracerebral hemorrhage
Meningioma
Saccular aneurysm
Lucid interval
Meningitis pathogens
E. coli
H. influenzae
N. meningitidis
S. aureus
Pneumococci
Viral encephalitis
Herpes zoster
Herpes simplex
Rabies virus
Poliovirus
PML
Lyssa bodies
Poliomyelitis
Epidural abscesses
Entamoeba histolytica
Naegleria fowleri
Acanthamoeba
Plasmodium falciparum
Toxoplasma gondii
Cerebral cysticercosis
Cryptococcus
Mucormycosis
332 Pathology Questions and Answers, Prof.Dr. Murat ALPER
Candidiasis
Histoplasmosis
Aspergillosis
Subacute sclerosing panencephalitis
Kuru
Progressive multifocal leukoence
SECTION 24: MUSCULOSCELETAL PATHOLOGY
Certainly! Here's the translation of your text into academic English:
---
1. Which of the following causes limb shortening due to cartilage pathology without affecting
the skull and spinal bones?
A) Osteopetrosis
B) Achondroplasia
C) Osteogenesis imperfecta
D) Osteoporosis
E) Laron-type dwarfism
Achondroplasia is the most common skeletal dysplasia and one of the most frequent
causes of dwarfism. In this autosomal dominant disorder, proximal limb shortening, short
arms, and a normally sized head and trunk with a flat nasal bridge are present. A mutation in
the FGF receptor 3 gene leads to inhibited proliferation of chondrocytes in the growth plate.
The cartilaginous growth plates are disorganized and hypoplastic. Intellectual and
reproductive capacities are normal. Thanatophoric dwarfism is the most common lethal
variant of dwarfism, also involving FGFR3 mutations. Affected individuals typically die at birth
due to respiratory distress caused by micromelia, a small thoracic cage, and a bell-shaped
abdomen. (Answer: B)
2. Which of the following is characterized by abnormal matrix formation, resulting in a bone
development defect?
A) Osteopetrosis
333 Pathology Questions and Answers, Prof.Dr. Murat ALPER
B) Achondroplasia
C) Osteogenesis imperfecta
D) Osteitis deformans
E) Osteomalacia
(Answer: C)
3. Which of the following diseases can cause multiple intrauterine fractures?
A) Osteoporosis
B) Osteopetrosis
C) Rickets
D) Achondroplasia
E) Osteogenesis imperfecta
(Answer: E)
4. Which disease is characterized by multiple fractures due to defective collagen synthesis
and gene mutations?
A) Osteopetrosis
B) Fibrous dysplasia
C) Osteogenesis imperfecta
D) Achondroplasia
E) Von Recklinghausen's disease
Osteogenesis imperfecta, the most common hereditary connective tissue disorder, involves
a type I collagen synthesis defect leading to brittle bones. The disease also affects joints,
eyes, ears, skin, and teeth due to dentin deficiency. There is notable cortical thinning and
osteoporosis. Severe forms may be effective even in utero (type II). Osteogenesis imperfecta
is characterized by multiple fractures due to minimal trauma, gene mutations, defective
collagen synthesis affecting teeth, bones, skin, and eyes, and a type I collagen synthesis
disorder. Blue sclera and hearing loss are common. The most frequent form is the autosomal
dominant type. Woven bone is seen histologically due to continuous fractures. (Answer: C)
334 Pathology Questions and Answers, Prof.Dr. Murat ALPER
5. Which of the following diseases is associated with increased bone density due to
insufficient osteoclastic activity?
A) Osteomalacia
B) Osteogenesis imperfecta
C) Osteopetrosis
D) Osteitis deformans
E) Rickets
(Answer: C)
6. Which bone disease is associated with a deficiency of the carbonic anhydrase enzyme?
A) Osteomalacia
B) Osteogenesis imperfecta
C) Osteopetrosis
D) Osteitis deformans
E) Rickets
(Answer: C)
7. In which of the following diseases is the Erlenmeyer flask deformity observed?
A) Osteomalacia
B) Osteogenesis imperfecta
C) Osteopetrosis
D) Osteitis deformans
E) Rickets
In rickets, there is an increase in uncalcified osteoid cartilage and a decrease in calcified
bone. Osteomalacia is the adult form. When secondary to renal disease, it is called renal
osteodystrophy. Paget's disease shows abnormal bone structure due to increased
osteoblastic and osteoclastic activity. Osteogenesis imperfecta (brittle bone disease) is
caused by defective type I collagen synthesis. Osteopetrosis (marble bone disease) shows
increased bone density due to insufficient osteoclastic activity. In some types of
osteopetrosis, there is a deficiency of the carbonic anhydrase enzyme necessary for
osteoclast function. Bones are as hard as stone but easily crumble and break like chalk.
335 Pathology Questions and Answers, Prof.Dr. Murat ALPER
Multiple fractures occur. The ends of long bones are bulbous, and the medullary canal is
absent (Erlenmeyer Flask deformity). Reduced bone marrow space leads to anemia,
recurrent infections, and cranial nerve pathologies like blindness and deafness due to nerve
compression. The autosomal recessive variant is fatal in infants due to leukopenia,
hepatosplenomegaly, and multiple fractures from extramedullary hematopoiesis. The
autosomal dominant variant is milder and also known as Albers–Schönberg disease. (Answer:
C)
8. Which of the following diseases is characterized by soft and osteopenic bones due to
insufficient mineralization of newly formed bone matrix?
A) Rickets
B) Osteoporosis
C) Osteitis fibrosa cystica
D) Osteogenesis imperfecta
E) Osteopetrosis
Osteopenia generally refers to decreased bone mass, while osteoporosis involves a
reduction in bone mass significant enough to increase fracture risk due to predominant
osteoclastic activity. Serum phosphatase, calcium, and phosphorus levels are normal.
Estrogen is protective against osteoporosis. While osteoporosis has secondary types
associated with hyperthyroidism, malnutrition, and steroids, most cases are primary,
frequently senile and postmenopausal. In osteoporosis, histology shows normal but reduced
quantity. Microfractures and eventual vertebral collapse result from trabecular thinning.
Exercise, calcium, vitamin D, and bone resorption-reducing bisphosphonates can aid in
treatment. Osteogenesis imperfecta involves type I collagen defect and brittle bones.
Normally, 90% of osteoid is type I collagen. Symptomatic, untreated primary
hyperparathyroidism causes three bone changes: 1- osteoporosis, especially in phalanges,
femur, and vertebrae, 2- Brown tumors from hemorrhage and reactive tissue, 3- Osteitis
fibrosa cystica. In osteitis fibrosa cystica (von Recklinghausen disease of bone,
hyperparathyroid bone disease), parathyroid hormone levels are elevated. Osteoblastic and
osteoclastic activity increases, forming cysts through generalized bone resorption. In rickets
(osteomalacia in adults), osteoid exists but matrix mineralization is impaired. Normally,
osteoblasts have numerous Golgi apparatuses, produce osteoid, and exhibit alkaline
phosphatase activity. Osteoclasts are mitochondria-rich and high in acid phosphatase. The
metaphysis is a primary site for Ca+2 and PO4 exchange. (Answer: A)
9. Which of the following is most closely associated with the pathogenesis of
postmenopausal osteoporosis?
336 Pathology Questions and Answers, Prof.Dr. Murat ALPER
A) Increased number of osteoblasts
B) Decreased number of osteoblasts
C) Reduced mineralization of bone
D) Osteoclastic activity exceeding osteoblastic activity
E) Reduced exercise and bone remodeling
Normally, estrogen suppresses interleukin-I secretion from osteoblasts that activate
osteoclasts. Consequently, osteoclasts resorb more bone than osteoblasts form. In senile
osteoporosis, decreased osteoblast activity predominates, and the condition is classified as
low turnover osteoporosis. Postmenopausal osteoporosis is classified as high turnover
osteoporosis. Decreased mineralization characterizes osteomalacia. Although reduced
exercise contributes to osteoporosis, it is not the primary mechanism. Changes in osteoblast
and osteoclast numbers are also not the primary mechanism. RANK, a TNF family member,
activates osteoclasts, whereas osteoprotegerin, another TNF family member, inhibits
osteoclastogenesis. Normal bone is composed of 65% inorganic and 35% organic
components, with 90% of the organic component being type I collagen. Osteoblasts originate
from pluripotent mesenchymal osteoprogenitor cells, while osteoclasts derive from
hematopoietic cells. The serum marker for osteoblastic activity is osteocalcin, which is both
sensitive and specific. (Answer: D)
10. Which structural disease is characterized by increased osteoid in bones but impaired or
insufficient bone mineralization?
A) Osteomalacia
B) Osteoporosis
C) Osteonecrosis
D) Osteopetrosis
E) Myositis ossificans
Osteomalacia is characterized by insufficient mineralization of osteoid. The underlying
issue is a deficiency or abnormal metabolism of vitamin D. Unlike osteoporosis, osteomalacia
involves an imbalance between bone formation and resorption, ultimately leading to bone
loss. (Answer: A)
11. Which of the following is not responsible for the development of osteomalacia?
A) Aluminum intake
337 Pathology Questions and Answers, Prof.Dr. Murat ALPER
B) Vitamin D deficiency
C) Cushing's disease
D) X-linked hypophosphatemia
E) Malabsorption syndromes
Osteomalacia can result from primary and secondary vitamin D deficiency, malabsorption
syndromes, pancreatic insufficiency, chronic renal failure, use of phenytoin, phenobarbital,
and rifampin, X-linked hypophosphatemia, antacids, and aluminum intake. It is not typical in
Cushing's disease. (Answer: C)
12. Which condition should be considered when Looser's lines (Milkman fractures) are
observed radiologically in bones?
A) Osteomyelitis
B) Ewing sarcoma
C) Osteomalacia
D) Bone tuberculosis
Pathognomonic pseudofractures, known as Looser zones or Milkman fractures, are seen in
osteomalacia. Ewing sarcoma radiologically presents with an "onion skin" appearance.
Osteosarcoma features a Codman triangle. (Answer: C)
13. Kienböck's disease is the avascular necrosis of which of the following?
A) Femoral head
B) Metatarsal
C) Navicular bone
D) Calcaneus
E) Lunate bone
Legg–Calve–Perthes disease affects the femoral head, Osgood–Schlatter involves the tibial
tuberosity, Sever's disease affects the os calcaneum, Kienböck's disease involves the lunate
338 Pathology Questions and Answers, Prof.Dr. Murat ALPER
bone, Köhler's disease affects the navicular bone, and Preiser's disease involves the carpal
scaphoid bone. (Answer: E)
14. Which region of long bones is most frequently involved in hematogenous osteomyelitis in
children?
A) Metaphysis
B) Diaphysis
C) Epiphysis
D) Articular cartilage
E) Subperiosteal region
(Answer: A)
15. Which of the following is incorrect regarding gout?
A) Hyperuricemia is present
B) Tophus formation
C) Arthritis
D) Calcium deposition in the joint
E) Deficiency of hypoxanthine phosphoribosyl transferase
Gout is a disorder of uric acid metabolism, the end product of purine metabolism.
Hyperuricemia (above 6.8 mg/dl) results from increased uric acid production or decreased
excretion, leading to acute and chronic arthritis. Clinical gout develops in 10% of
hyperuricemic patients, typically after 20-30 years of hyperuricemia. Alcohol, obesity, and
medications like thiazides that reduce urate excretion contribute to gout. The big toe is most
commonly affected. Recurrent attacks lead to tophus formation. Monosodium urate
accumulates in joints, triggering inflammation. Rapid cell destruction during leukemia or
lymphoma treatment, or reduced excretion in renal failure, can cause secondary
hyperuricemia (secondary gout). After joints, the kidneys are most frequently involved,
resulting in urate nephropathy, uric acid nephrolithiasis, and pyelonephritis. The absence of
hypoxanthine guanine phosphoribosyl transferase enzyme is crucial in uric acid synthesis and
contributes to gout. Complete deficiency leads to Lesch–Nyhan syndrome (excessive uric acid
excretion, mental retardation, self-mutilation). Von Gierke disease also causes hyperuricemia,
rarely mimicking gout symptoms. In pseudogout and chondrocalcinosis, calcium
pyrophosphate crystals accumulate. (Answer: D)
339 Pathology Questions and Answers, Prof.Dr. Murat ALPER
16. Which of the following is incorrect regarding the causative agents of osteomyelitis?
A) Hematogenous osteomyelitis–Staphylococcus
B) Open fracture–mixed infection
C) Tooth infection–anaerobes
D) Drug addicts–Pseudomonas
E) Neonates–Klebsiella
Bone infection starts in the metaphysis, where blood flow is slowest. Staphylococci are
responsible for 80-90% of cases. H. influenzae and group B streptococci are common in
neonates. Mixed infections occur in open fractures, while anaerobic pathogens are more
common after dental infections. E. coli, Pseudomonas, and Klebsiella are increasingly found
in genitourinary infections and drug addicts. Salmonella infections are common in sickle cell
anemia, though staphylococci remain most frequent. The spontaneous resolution or chronic
infection site is known as Brodie's abscess. Pathological fractures, endocarditis, sepsis, and
amyloidosis are common complications in chronic osteomyelitis. Chronic cases with sinus
tracts have an increased risk of squamous cell carcinoma and sarcoma in infected bone.
Garre's sclerosing osteomyelitis affects the jaw, with intense new bone formation. (Answer:
E)
17. Which of the following is not a complication of pyogenic osteomyelitis?
A) Periostitis
B) Pott's disease
C) Sequestrum formation
D) Brodie's abscess
E) Involucrum formation
Pyogenic osteomyelitis is a bone infection caused by bacteria, rarely fungi. Staphylococcus
aureus is the most common cause, followed by E. coli and Pseudomonas. H. influenzae and
group B streptococci are significant in neonates. Brodie's abscess involves increased
fibroblastic activity surrounding the inflammatory focus, sequestrum (dead bone fragments),
and periostitis. Pott's disease is the tuberculosis osteomyelitis of the spine. In 40% of
mycobacterial infections, the spine is affected. Unlike pyogenic osteomyelitis, it is a chronic
insidious infection, causing severe spinal deformities through compression fractures.
(Answer: B)
340 Pathology Questions and Answers, Prof.Dr. Murat ALPER
18. Which of the following is not a feature of hematogenous osteomyelitis?
A) Almost always requires surgical drainage
B) More common in males
C) Most frequently affects the metaphyseal end of long bones
D) Most often caused by Staphylococcus aureus
E) Most common in children
In the acute phase, pyogenic osteomyelitis typically resolves with antibiotic treatment. If
the disease progresses to necrosis and sequestrum (dead bone) formation, surgical
intervention is usually required. (Answer: A)
19. A 50-year-old male complains of a continuously increasing hat size. Physical examination
reveals deafness, visual impairment, and compression fractures. Histological examination of
the obtained tissue shows a mosaic pattern. What is the diagnosis?
A) Osteochondroma
B) Osteitis deformans
C) Osteosarcoma
D) Osteoporosis
E) Aneurysmal bone cyst
The described patient has Paget's disease (osteitis deformans). The balance between bone
formation and resorption is disrupted, with both increased. The disease has three phases:
osteolytic, osteoblastic-osteoclastic, and finally osteosclerotic. The cortical and spongy bone
trabeculae thicken. Despite excessive bone, the bone remains soft and porous. Histologically,
the lamellar bone shows a mosaic pattern. Osteoclasts are numerous and large. Chalk stick
fractures can occur in long bones, typically transverse. Thickened skulls may compress cranial
nerves, causing symptoms. When monostotic (affecting one bone), it often involves the tibia,
ilium, and femur. When polyostotic (affecting multiple bones), it commonly involves the
spine (80%), pelvis (65%), followed by the femur, skull, and others. Craniofacial enlargement
may cause a lion-like face. Osteoarthritis is common in the tibia and femoral head. It is more
common in males over 50. Paramyxovirus may be implicated. It can transform into
osteosarcoma in 1% of cases. High-output cardiac failure can occur. Alkaline phosphatase
levels increase. Paget's disease has a relatively benign course. Most patients respond well to
calcitonin and bisphosphonates. (Answer: B)
341 Pathology Questions and Answers, Prof.Dr. Murat ALPER
20. Which of the following tumors may be associated with Paget's disease of bone?
A) Ewing sarcoma
B) Giant cell tumor
C) Metastatic ductal carcinoma of the breast
D) Multiple enchondromas
E) Osteosarcoma
Paget's disease of bone can progress to osteosarcoma. Giant cell tumor, malignant fibrous
histiocytoma, and chondrosarcoma can also develop. This should not be confused with
Paget's disease of the breast, which is almost always associated with underlying ductal
carcinoma of the breast. (Answer: E)
21. Hypertrophic osteoarthropathy is most commonly associated with which disease?
A) Pleural mesothelioma
B) Bronchogenic carcinoma
C) Infective endocarditis
D) Ulcerative colitis
Hypertrophic osteoarthropathy is an idiopathic disorder characterized by periosteal
inflammation, arthritis, and new bone formation in tubular bones. Clubbing of the fingers is a
common manifestation. It is most frequently seen in bronchogenic carcinomas. It can occur
in intrathoracic tumors such as pleural mesothelioma, pulmonary metastases, and Hodgkin's
lymphoma, as well as chronic lung and liver diseases. When only clubbing is present without
new bone formation and arthritis, it may be seen in cyanotic heart diseases, infective
endocarditis, ulcerative colitis, Crohn's disease, and carcinomas of the colon or esophagus.
Recognizing hypertrophic osteoarthropathy is essential, as it may be the first sign of a severe
disease, and the arthropathy may resolve with treatment of the underlying pathology.
(Answer: B)
22. Which disease affects the spine, hip, and knee joints, causing joint destruction and
Heberden's nodes?
A) Osteoarthritis
B) Villonodular synovitis
C) Gouty arthritis
342 Pathology Questions and Answers, Prof.Dr. Murat ALPER
D) Rheumatoid arthritis
E) Lyme disease
While rheumatoid arthritis affects synovial membranes, osteoarthritis damages articular
cartilage. Osteoarthritis can be monoarticular or polyarticular but is typically found in
extremities. Osteophytes in distal interphalangeal joints are known as Heberden's nodes.
Osteoarthritis is the most common form of arthritis, characterized by aching pain, stiffness,
and movement limitation, most pronounced in the morning. Diagnosis is made by
demonstrating joint space narrowing and characteristic osteophytes radiologically. (Answer:
A)
23. Which of the following is a spirochete disease transmitted by arthropods, causing
migratory arthritis and erythema chronicum migrans, and microscopically showing synovial
hyperplasia, fibrin deposition, and dense lymphocytes?
A) Rheumatoid arthritis
B) Reactive arthritis
C) Lyme arthritis
D) Reiter syndrome
E) Suppurative arthritis
Lyme disease is caused by the spirochete Borrelia burgdorferi transmitted to humans by
Ixodes ticks. It causes erythema chronicum migrans at the bite site and oligoarthritis, later
progressing to polyarthritis and joint limitation. It affects the skin, cranial nerves, heart, and
meninges. 60-80% develop migratory arthritis. Infected synovium shows synovial
hyperplasia, fibrin deposition, and dense lymphocyte infiltration, resembling rheumatoid
arthritis morphology in severe cases. (Answer: C)
24. What is the characteristic histological finding observed in degenerative arthritis?
A) Fibrillation and cleft formation in articular cartilage
B) Development of pannus
C) Anisotropic crystals in joint fluid
D) Diffuse proliferative synovitis
343 Pathology Questions and Answers, Prof.Dr. Murat ALPER
E) Fibrous or bony ankylosis
(Answer: A)
25. Which of the following findings is expected in rheumatoid arthritis rather than
osteoarthritis?
A) Cartilage fibrillation
B) Subchondral bone cysts
C) Osteophytes
D) Joint ankylosis
E) Female gender
Rheumatoid arthritis is an immunological disorder characterized by inflammation,
proliferation of synovial tissue over articular cartilage, inflammatory cell and granulation
tissue infiltration (pannus), and subsequent release of enzymes that destroy bone and
cartilage. The synovium in rheumatoid arthritis contains numerous plasma cells and germinal
centers with secondary follicles. Anti-citrullinated protein antibodies (ACPA) contribute to
chronicity. 20% of RA patients have positive ACPA. 50% have HLA class II antigens. Reactive
fibrosis causes ankylosis and immobility in the joint. Osteoarthritis is a non-inflammatory
disease resulting from progressive degeneration of articular cartilage, leading to cartilage
fibrillation, subchondral bone cysts, osteophytes, and decreased mobility due to secondary
synovitis. Both diseases are more common in women. Interestingly, increased estrogen levels
raise the risk of osteoarthritis. Juvenile idiopathic arthritis (JIA) occurs before age 16 and lasts
at least six weeks, with an unknown cause. Unlike rheumatoid arthritis, oligoarthritis is more
common, systemic disease is more frequent, large joints are more often affected, rheumatoid
nodules and factor are usually absent, and ANA positivity is common. (Answer: D)
26. A 45-year-old woman presents with fatigue and low-grade fever, and antibodies against
serum IgG are detected. What is the likely diagnosis?
A) Osteoarthritis
B) Rheumatoid arthritis
C) Gout
D) Villonodular synovitis
E) Suppurative arthritis
344 Pathology Questions and Answers, Prof.Dr. Murat ALPER
Rheumatoid arthritis is a non-suppurative systemic disease affecting synovial membranes.
It is three times more common in women and typically presents between 40-50 years of age.
Symptoms include fever, fatigue, anemia, leukocytosis, joint stiffness, and swelling of soft
tissues around affected joints. Pericarditis and pleuritis may occur. Permanent ankylosis may
develop in later stages. Hypergammaglobulinemia is characteristic. Serum contains
antibodies against IgG, known as rheumatoid factor. (Answer: B)
27. Which disease is characterized by round or ovoid radiolucent lesions resembling ground
glass on X-rays, with connective tissue containing irregular bone and cartilage, and can be
associated with McCune–Albright syndrome?
A) Paget's disease
B) Fibrous dysplasia
C) Bone tuberculosis
D) Osteomalacia
E) Osteitis fibrosa cystica
Fibrous dysplasia most commonly affects the ribs, femur, and tibia. It is monostotic in 70%
of cases and polyostotic in fewer patients. It can be associated with McCune–Albright
syndrome, characterized by café-au-lait spots, endocrinopathy, and precocious puberty, and
less frequently with hyperthyroidism, Cushing's syndrome, and acromegaly. Histologically, it
consists of connective tissue containing irregular bone and cartilage resembling Chinese
characters. Sarcomatous transformation (osteosarcoma) is rare but possible. (Answer: B)
28. What is the likely diagnosis when there is precocious puberty, skin hyperpigmentation,
and fibrous dysplasia in the skeleton?
A) Sturge–Weber
B) Tuberous sclerosis
C) McCune–Albright syndrome
D) Neurofibromatosis
E) Von Recklinghausen syndrome
When fibrous dysplasia with a Chinese figure-like arrangement is seen in women with
precocious puberty and skin hyperpigmentation, it is known as McCune–Albright syndrome.
(Answer: C)
345 Pathology Questions and Answers, Prof.Dr. Murat ALPER
29. Which of the following is a bone tumor, often located in the metaphysis of long bones in
males under 20, characterized by a stalk covered with cartilage and continuous with the
cortex of the host bone?
A) Osteochondroma
B) Giant cell tumor
C) Chondroma
D) Osteoid osteoma
E) Osteosarcoma
Osteochondroma (exostosis) accounts for 30% of benign bone tumors. It occurs in the
metaphysis of long bones, particularly the femur, but originates from the epiphyseal
cartilage. It can be pedunculated or sessile. It is three times more common in males. The
outer layer is a cartilage cap, with bone and bone marrow inside. It stops growing when the
growth plate closes. It is usually asymptomatic and often does not require removal. If
symptomatic, it is excised. Signs of malignancy include a size >8 cm, diameter >3 cm, and
irregular cap. While sporadic cases are rare, they are more common in multiple hereditary
exostosis syndrome, where chondrosarcoma or other sarcoma types may develop. (Answer:
A)
30. Which bone pathology is observed in Ollier's disease and Maffucci syndrome?
A) Osteochondroma
B) Chondroma
C) Giant cell tumor
D) Osteosarcoma
E) Osteoid osteoma
Chondromas are benign cartilage tumors most commonly seen between the 2nd and 5th
decades of life. They are often found in bones of enchondral origin. When arising from the
bone surface, they are called juxtacortical chondromas, and when located in the medullary
cavity, they are called enchondromas. Enchondromas are the most common intraosseous
cartilage tumors. They affect small bones of the hands and feet. They are typically less than 3
cm, grey-blue translucent nodules. They are usually painless but can cause pain and
fractures. Radiologically, they show an intact cortex and an O-ring sign with a sclerotic rim.
Ollier syndrome (enchondromatosis) involves multiple chondromas in the metaphysis and
346 Pathology Questions and Answers, Prof.Dr. Murat ALPER
diaphysis of various bones. Maffucci syndrome is characterized by chondromas,
dyschondroplasia, ovarian cancers, brain gliomas, and numerous hemangiomas in the skin
and internal organs. Chondromas associated with Ollier and especially Maffucci syndromes
have a significantly increased risk of transforming into chondrosarcoma. (Answer: B)
31. Which of the following statements about chondrosarcoma is incorrect?
A) It grows slower than osteosarcoma
B) It can develop from enchondroma
C) It is most frequently seen in the pelvis and femur
D) It is most common between ages 20-35
E) Multiple chondroblasts are found in a single lacuna
Chondrosarcoma is a malignant tumor producing cartilage. It grows more slowly, thus has
a better prognosis than osteosarcoma. Unlike osteosarcoma, it is rare before age 40. It is
most commonly found in flat bones like the pelvis and shoulder girdle, followed by the ribs
and less frequently in the femur and humerus, originating from the medullary cavity. It is
twice as common in males. Although most chondrosarcomas develop directly as
malignancies, they can also arise from pre-existing enchondromas (Ollier, Maffucci) and, less
commonly, from fibrous dysplasia, Paget's disease, and osteochondroma. It frequently
metastasizes to the lungs. Histologically, multiple chondroblasts can be seen in a single
lacuna. It stains positively with S-100. Tumors exceeding 10 cm are considered malignant
regardless of histology. 90% are conventional types
, with less frequent clear cell, dedifferentiated, and mesenchymal types. Clear and
mesenchymal types are seen in children and young adults. (Answer: D)
32. Which tumor is commonly seen in young males, is painful, and radiologically presents as
a nidus, usually smaller than 2 cm?
A) Giant cell tumor
B) Chondrosarcoma
C) Osteoid osteoma
D) Chondroma
E) Osteochondroma
347 Pathology Questions and Answers, Prof.Dr. Murat ALPER
Osteoid osteoma and osteoblastoma are benign bone-producing tumors, identical
histologically but distinct clinically and radiologically. Osteoid osteoma is more common in
young males. It is less than 2 cm and causes pain, often severe at night, relieved by aspirin
due to osteoblast-produced PGE2. Radiographically, it presents as a nidus. It is most
frequently found in the femur, tibia, and humerus, particularly in the metaphyseal region of
long bones. 85% begin in the cortex, and 13% in the medulla. They are round-oval masses
with irregular woven bone lamellae surrounded by prominent osteoblastic cells. High levels
of prostaglandins are detected. Osteoblastoma histology resembles osteoid osteoma but is
more often located in the vertebrae and causes aspirin-unresponsive pain. It is larger than 2
cm. Osteoid osteoma is typically treated with radiofrequency ablation, while osteoblastoma
is curetted or excised. (Answer: C)
33. Which bone tumor is frequently associated with intestinal polyposis and soft tissue
tumors?
A) Osteoid osteoma
B) Chondroma
C) Osteoma
D) Osteochondroma
E) Chondromyxoid fibroma
Osteomas are seen in males aged 40-50. They reflect developmental anomalies or reactive
growth rather than true tumors. They typically affect the skull and face, most commonly in
the paranasal sinuses. Histologically, they resemble normal bone tissue. They are non-
invasive and do not undergo malignant transformation. Gardner syndrome includes intestinal
polyposis, soft tissue tumors, and multiple osteomas. Osteomas are subperiosteal, exhibiting
outwardly convex, oval, round, sessile tumors. (Answer: C)
34. Which tumor presents radiologically as soap-bubble-like lytic lesions, and its histological
findings do not provide a definitive prognosis?
A) Multiple myeloma
B) Chondrosarcoma
C) Ewing sarcoma
D) Giant cell tumor
E) Osteosarcoma
348 Pathology Questions and Answers, Prof.Dr. Murat ALPER
The giant cell tumor of bone (osteoclastoma) is generally benign but can behave
aggressively locally. If not adequately excised, it recurs in 50% of cases and has an increased
risk of sarcomatous transformation. It metastasizes to the lungs in about 4% of cases. It
occurs in individuals aged 20-40 and is more common in females. It is located in the
epiphyseal region, most frequently in the distal femur, proximal tibia, distal radius, and
proximal humerus. Those around the knee may present with arthritis-like symptoms. This
tumor does not occur in skeletally immature individuals. It accounts for 20% of benign bone
tumors. Microscopically, it features osteoclast-type multinucleated giant cells, without
cartilage or bone. Radiologically, it presents as soap-bubble lytic lesions. Its microscopic
appearance does not predict behavior. Necrosis, hemorrhage, and reactive bone formation
are common. The RANKL inhibitor denosumab may be used. (Answer: D)
35. A 20-year-old patient presents with tibial metaphyseal pain and swelling. Radiology
reveals a well-defined lytic lesion, and CT shows fluid-fluid levels in a lesion with multilocular
blood-filled cystic septa. What is the most likely diagnosis?
A) Aneurysmal bone cyst
B) Osteochondroma
C) Osteosarcoma
D) Giant cell tumor of bone
E) Chondrosarcoma
(Answer: A)
36. Which bone most commonly hosts a hemangioma?
A) Skull
B) Hands and feet
C) Vertebrae
D) Pelvis
E) Rib
Bone hemangiomas are benign bone tumors located in the medulla, most commonly
found in the skull, vertebrae, and jawbones in individuals aged 20-50, affecting males and
females equally. (Answer: A)
349 Pathology Questions and Answers, Prof.Dr. Murat ALPER
37. A 1 cm firm, mobile mass is found on the wrist. Microscopically, it shows a cystic lesion
without epithelium and with myxoid changes. Radiologically, there is no connection with the
joint cavity. What is the most likely diagnosis?
A) Ganglion cyst
B) Baker's cyst
C) Tenosynovial giant cell tumor
D) Gout
E) Pseudogout
The described findings indicate a ganglion cyst located near the joint capsule and tendon
sheath. Despite its name suggesting a neurological origin, it has no relation to the nervous
system. A Baker's cyst is a popliteal synovial cyst that can occur in rheumatoid arthritis or
osteoarthritis, with hyperplastic and inflamed synovium. (Answer: A)
38. Which bone tumor is epiphyseal in location?
A) Ewing sarcoma
B) Osteosarcoma
C) Fibrous dysplasia
D) Giant cell tumor of bone
E) Chondrosarcoma
(Answer: D)
39. Which tumor is typically located in the epiphysis?
A) Chondroblastoma
B) Osteosarcoma
C) Osteochondroma
D) Osteoid osteoma
E) Ewing's sarcoma
Epiphysis: Chondroblastoma and giant cell tumors almost always originate from the
epiphysis of long bones.
350 Pathology Questions and Answers, Prof.Dr. Murat ALPER
Metaphysis: Giant cell tumor, enchondroma, chondrosarcoma, osteosarcoma,
osteochondroma, and osteoblastoma.
Metaphyseal-diaphyseal junction: Fibrosarcoma, fibrous cortical defect, chondromyxoid
fibroma, fibrous dysplasia, and osteoid osteoma.
Diaphysis: Ewing sarcoma, osteofibrous dysplasia, and adamantinoma.
Chondroblastoma is more common in males aged 10-20, is characteristically painful, and is
often found in the femur, humerus, and tibia. It exhibits a "chicken wire" appearance and is
S-100 protein positive. (Answer: A)
40. Which bone tumor is characterized by tumor cells producing osteoid?
A) Osteoid osteoma
B) Chondrosarcoma
C) Osteoclastoma
D) Osteosarcoma
E) Multiple myeloma
(Answer: D)
41. Which tumor frequently co-occurs with retinoblastoma?
A) Chondrosarcoma
B) Chondroma
C) Osteoma
D) Osteosarcoma
E) Ewing sarcoma
(Answer: D)
42. Which is the most common primary malignant bone tumor aside from multiple
myeloma?
A) Osteosarcoma
B) Chondrosarcoma
C) Leukemia
351 Pathology Questions and Answers, Prof.Dr. Murat ALPER
D) Lymphoma
E) Ewing sarcoma
(Answer: A)
43. A 15-year-old male presents with leg pain and swelling. A radiograph shows a Codman's
triangle in the extremity. What is the most likely diagnosis?
A) Osteochondroma
B) Chondrosarcoma
C) Multiple myeloma
D) Osteosarcoma
E) Osteomyelitis
(Answer: D)
44. Which of the following has the best prognosis?
A) Parosteal osteosarcoma
B) Telangiectatic osteosarcoma
C) Small cell osteosarcoma
D) Osteosarcoma developing in Paget's disease
E) Intramedullary classic osteosarcoma
Osteosarcoma is a highly malignant bone tumor characterized by neoplastic cells
producing osteoid matrix or mineralized bone. It is the most common primary malignant
bone tumor, excluding myeloma and lymphoma. It is 1.6 times more common in young males
aged 10-20. RB gene mutations are common in sporadic tumors. It arises from the
metaphyseal ends of long bones. 60% occur around the knee, 15% in the hip joint, 10% in the
shoulder, and 8% in the jaw. In the elderly, it arises from Paget's disease, bone infarcts, or
post-radiation, as secondary osteosarcoma causes. Radiologically, a Codman's triangle is
seen. While a Codman's triangle indicates aggressiveness, it is not specific to osteosarcoma.
The 5-year survival rate is 5-20%. Metastasis most commonly occurs in the lungs.
Osteosarcoma is frequently associated with retinoblastoma (RB gene) and Li–Fraumeni
syndrome (p53 gene). Individuals with the RB gene are 1,000 times more likely to develop it.
Primary osteosarcomas have a better prognosis than
352 Pathology Questions and Answers, Prof.Dr. Murat ALPER
secondary ones.
Good prognosis: Jaw or distal extremities, parosteal, periosteal, and well-differentiated
intramedullary types.
Poor prognosis: Paget's disease-related, radiation-induced, multifocal, focal
dedifferentiated, and telangiectatic types. Age and gender do not affect prognosis. (Answer:
A)
45. Which bone tumor is characterized by the t(11;22)(q24;q12) translocation?
A) Multiple myeloma
B) Osteosarcoma
C) Ewing sarcoma
D) Giant cell tumor of bone
E) Chordoma
(Answer: C)
46. Which bone tumor can clinically mimic osteomyelitis?
A) Osteosarcoma
B) Plasmacytoma
C) Giant cell tumor of bone
D) Ewing sarcoma
E) Chondrosarcoma
(Answer: D)
47. Which tumor most commonly resembles malignant lymphoma histopathologically?
A) Ewing sarcoma
B) Giant cell tumor
C) Osteosarcoma
D) Osteoblastoma
E) Osteoid osteoma
(Answer: A)
353 Pathology Questions and Answers, Prof.Dr. Murat ALPER
48. A 10-year-old child presents with local leg pain, tenderness, and fever. Radiology shows
an "onion skin" appearance with soft tissue spread. What is the most likely diagnosis?
A) Osteosarcoma
B) Multiple myeloma
C) Ewing sarcoma
D) Lymphoma
E) Neuroblastoma
Ewing sarcoma, seen between ages 10-15, primarily affects long bones (especially the
femur and pelvic flat bones) and can also occur in unexpected locations like the orbit, soft
tissue, and meninges. It is a primitive small round blue cell tumor with the characteristic
(t11:22) (q24;q12) molecular features and varying degrees of neuroectodermal
differentiation. It is the second most common malignant tumor in children after
osteosarcoma. It originates in the medullary cavity, typically in the diaphysis of long bones,
and invades bone. Radiologically, it presents with periosteal reaction and new bone
formation around the bone shaft, creating an "onion skin" appearance. It is soft and may
show signs of hemorrhage and necrosis. Microscopically, it contains cells with PAS-positive
cytoplasmic granules. They are MIC 2 (CD 99) positive. These cells can be mistaken for other
small round blue cell tumors like neuroblastoma, Wilms tumor, lymphoma, and
rhabdomyosarcoma. Homer Wright rosettes may indicate neuroectodermal differentiation. A
reciprocal translocation between the 11th and 22nd chromosomes should be demonstrated.
Clinically, it mimics osteomyelitis with local pain, tenderness, and fever. The tumor grows
rapidly and metastasizes to the lungs and brain. The 5-year survival rate with chemotherapy
is 75%. (Answer: C)
49. Which tumor causes osteoblastic bone metastasis?
A) Lung
B) Prostate
C) Colon
D) Kidney
E) Thyroid
Although most bone metastases are osteolytic, prostate cancer and some breast
carcinomas can cause osteoblastic bone metastases. (Answer: B)
354 Pathology Questions and Answers, Prof.Dr. Murat ALPER
50. Which tumor most frequently metastasizes to bone?
A) Lung
B) Breast
C) Kidney
D) Prostate
E) Colon
Tumors most frequently metastasizing to bone are, in order, prostate, breast, kidney, and
lung. In children, neuroblastoma, Wilms tumor, osteosarcoma, Ewing sarcoma, and
rhabdomyosarcoma frequently metastasize to bone. Metastases are most commonly found
in the vertebral column, followed by ribs, skull, and pelvis. (Answer: D)
51. Which condition is most likely indicated by hypercalcemia, anemia,
hypergammaglobulinemia, proteinuria, and normal serum alkaline phosphatase with
widespread bone demineralization?
A) Ewing sarcoma
B) Hyperparathyroidism
C) Multiple myeloma
D) Osteomalacia
E) Paget's disease of bone
Although multiple myeloma typically presents with punched-out bone lesions, it often
accompanies widespread bone demineralization. Features distinguishing multiple myeloma
from other demineralizing conditions include anemia, hypergammaglobulinemia,
proteinuria, and normal (rather than elevated) serum alkaline phosphatase. (Answer: C)
52. Which X-linked recessive disease predominantly affects boys, with symptoms appearing
when the child begins to walk, involving the pelvic girdle muscles and elevated creatine
kinase?
A) Myasthenia gravis
B) Becker dystrophy
C) Duchenne dystrophy
355 Pathology Questions and Answers, Prof.Dr. Murat ALPER
D) Trichinosis
E) Poliomyelitis
Duchenne muscular dystrophy (DMD) and Becker muscular dystrophy (BMD) are
dystrophinopathies. DMD is the most severe and common muscular dystrophy, passed down
in an X-linked recessive manner. It predominantly affects males. Calf pseudohypertrophy may
occur. Symptoms such as difficulty rising from a seated position, standing, and walking
appear when the child begins to walk. Respiratory muscle involvement commonly leads to
death around age 20. Creatine kinase levels are elevated. Becker muscular dystrophy has a
milder clinical presentation than Duchenne muscular dystrophy. In dystrophies, muscle tissue
is replaced by fat and fibrous tissue. There is notable variation in fiber size, with hypertrophic
and atrophic fibers observed. (Answer: C)
53. A 30-year-old female presents with progressive generalized weakness and diplopia.
Muscle weakness first appears in the extrinsic ocular muscles, face, and tongue after
exercise. Which disease involves antibodies against acetylcholine receptors?
A) Myasthenia gravis
B) Lambert-Eaton syndrome
C) Duchenne muscular dystrophy
D) Becker muscular dystrophy
E) Polymyositis
Myasthenia gravis is a disease affecting primarily young women, characterized by
generalized muscle weakness and extrinsic ocular muscle involvement. It is more common in
females aged 30-40. Antibodies target the postsynaptic acetylcholine receptors at the
neuromuscular junction. Muscle weakness improves with rest and worsens with exercise.
Diplopia and ptosis are common signs of extraocular muscle weakness. 65% of patients have
thymic hyperplasia, and 15% have thymoma. In thymoma, removal of the thymus can
improve myasthenia gravis. The worst prognosis in myasthenia gravis is seen when
associated with thymoma. Sensory and autonomic functions are unaffected. Cholinesterase
inhibitors and plasmapheresis are effective treatments. Lambert-Eaton syndrome involves
autoantibodies inhibiting presynaptic calcium channels, often seen as a paraneoplastic
syndrome in small cell lung carcinoma. Cholinesterase inhibitors are ineffective in this
syndrome. (Answer: A)
356 Pathology Questions and Answers, Prof.Dr. Murat ALPER
54. A patient exhibits ascending paralysis and loss of deep tendon reflexes following an
influenza-like illness, with increased protein in CSF. Peripheral nerve biopsy reveals
segmental demyelination and axonal damage. What is the most likely diagnosis?
A) Type 2 muscle atrophy
B) Guillain–Barré syndrome
C) Myasthenia gravis
D) Leprosy
E) Denervation atrophy
Guillain–Barré syndrome (acute inflammatory demyelinating polyradiculopathy) is
characterized by ascending paralysis, starting distally and progressing proximally.
Histologically, inflammation and demyelination are observed in spinal nerve roots and
peripheral nerves. Two-thirds of patients have an antecedent influenza-like illness. It can be
triggered by infections with Campylobacter jejuni, CMV, EBV, Zika virus, and Mycoplasma
pneumoniae. Plasmapheresis can be effective in treatment. Type 2 muscle fibers are
responsible for fast movements. They appear light in ATPase stain at pH 4.2 and dark at pH
9.4. They have low lipid content, abundant glycogen, and contract quickly, appearing white.
Type 2 atrophy is often seen in patients confined to bed for extended periods. (Answer: B)
55. Which is the most common soft tissue tumor?
A) Lipoma
B) Fibroma
C) Rhabdomyoma
D) Fibrous histiocytoma
E) Chondroma
Lipoma is the most common soft tissue tumor in adults, typically found in the back,
shoulders, and neck. It consists of mature adipose tissue encased in a thin capsule. It is
mobile and painless (angiolipoma is painful). Atypical lipomas in subcutaneous tissue may be
confused with liposarcoma. (Answer: A)
56. Where is liposarcoma most commonly found?
A) Retroperitoneum
B) Hip
357 Pathology Questions and Answers, Prof.Dr. Murat ALPER
C) Inguinal region
D) Thigh and knee
E) Shoulder
Liposarcoma is the second most common soft tissue tumor after undifferentiated
pleomorphic sarcoma. It is most frequently found in the thigh and popliteal fossa, followed
by the retro
peritoneum, then the inguinal region, hip, and other locations. It is most common around
age 50. It typically locates deep, often near the quadriceps muscle. Retroperitoneal tumors
can grow large but do so slowly. Transformation from lipoma is extremely rare. Grossly, they
tend to be well-defined and encapsulated. There are five types, with the myxoid type being
the most common. The well-differentiated type has the best prognosis, with the least
metastasis and recurrence. The myxoid and well-differentiated types metastasize less than
the round cell, pleomorphic, and dedifferentiated types. The pleomorphic variant is generally
aggressive and frequently metastasizes. (Answer: D)
57. What is the most common malignant soft tissue tumor?
A) Liposarcoma
B) Fibrosarcoma
C) Undifferentiated pleomorphic sarcoma
D) Leiomyosarcoma
E) Rhabdomyosarcoma
Undifferentiated pleomorphic sarcoma (UPS, formerly known as malignant fibrous
histiocytoma) is the most common soft tissue sarcoma. It is generally well-defined but lacks a
capsule. It can occur in the extremities, retroperitoneum, subcutaneous tissue, and other
areas. Leiomyosarcoma is most commonly found in extremities and arises from the vessel
wall. It is more frequent in adults and females. Immunohistochemically, it is positive for
desmin and caldesmon. Cutaneous and subcutaneous leiomyosarcomas have a good
prognosis, while deep-seated tumors tend to metastasize. Retroperitoneal and/or
mesenteric tumors are always malignant. Tumors larger than 5 cm have a poor prognosis.
Necrosis indicates poor prognosis. The term UPS replaces the MFH classification, which
included various tumors now recognized as pleomorphic rhabdomyosarcoma, liposarcoma,
and leiomyosarcoma. Many tumors previously diagnosed as fibrosarcoma are now identified
358 Pathology Questions and Answers, Prof.Dr. Murat ALPER
as desmoid tumors, MFH, malignant peripheral nerve sheath tumors, or synovial sarcoma.
(Answer: C)
58. What is the most common soft tissue tumor resulting from radiotherapy or radiation
exposure?
A) Synovial sarcoma
B) Leiomyosarcoma
C) Fibrosarcoma
D) Malignant fibrous histiocytoma
E) Rhabdomyosarcoma
The most common soft tissue sarcoma following radiotherapy is UPS (formerly generally
termed malignant fibrous histiocytoma). MFH is also the most common soft tissue tumor
overall. Among post-radiation soft tissue sarcomas, MFH is the most frequent, accounting for
70%, followed by osteosarcoma, fibrosarcoma, chondrosarcoma, and angiosarcoma, most of
which are high-grade. Synovial sarcoma typically exhibits a biphasic pattern and is more
common between 15–35 years. (Answer: D)
59. What is the most likely diagnosis for a 28-year-old woman who develops a mass in the
rectus abdominis muscle following pregnancy?
A) Keloid
B) Dupuytren's contracture
C) Fibromatosis
D) Leiomyosarcoma
E) Fibrosarcoma
Fibromatoses include superficial and deep forms. Deep-seated ones are often called
desmoid tumors. Desmoid tumors (fibromatoses) are considered grade 0.5 fibrosarcoma.
They occur between the 2nd and 4th decades in three locations: 1. In the abdominal wall
muscles during and after pregnancy, 2. In the shoulder, chest wall, and hip muscles, 3. In the
abdomen in patients with Gardner syndrome. They grow infiltratively and contain dense
collagen. The superficial forms of fibromatosis include Dupuytren's (palm of the hand) and
Peyronie's (dorsolateral penis). Proliferating cells are myofibroblastic. (Answer: C)
359 Pathology Questions and Answers, Prof.Dr. Murat ALPER
60. What is the term for flexion contracture of the IV and V fingers due to contracture and
hypertrophy of the palmar aponeurosis?
A) Carpal tunnel syndrome
B) Volkmann's contracture
C) Dupuytren's contracture
D) Peyronie's disease
E) De Quervain's disease
Dupuytren's disease is a slow-progressing fibromatosis of the palmar fascia, which is
bilateral in 50% of cases, leading to flexion contracture, particularly in the 4th and 5th
fingers. De Quervain's disease is a narrowing of the thumb extensor sheath. Peyronie's
disease is penile fibromatosis. (Answer: C)
61. Which tumor is commonly seen in young individuals, rapidly growing over a few weeks
after trauma, and rarely exceeds 3 cm in size?
A) Osteosarcoma
B) Nodular fasciitis
C) Fibromatosis
D) Synovial sarcoma
E) Malignant fibrous histiocytoma
Nodular fasciitis is a self-limited fibroblastic and myofibroblastic proliferation. It rapidly
develops over a few weeks and rarely exceeds 3 cm. 25-50% of patients have a history of
trauma. It is more common in young adults, usually occurring in the forearm and trunk,
primarily in the upper body, often in subcutaneous tissue, rarely deeper. It consists of spindle
fibroblasts and myofibroblasts in a loose myxoid stroma. Mitoses are present, causing
confusion with sarcoma. Nodular fasciitis can spontaneously regress and, if removed, rarely
recurs. There is no malignant transformation. (Answer: B)
62. A 20-year-old male reports a mass and pain in his arm, with a history of a basketball
injury 3 weeks prior. Radiology shows a well-defined soft tissue lesion with peripheral
calcifications. What is the most likely diagnosis?
A) Myositis ossificans
B) Fibromatosis
360 Pathology Questions and Answers, Prof.Dr. Murat ALPER
C) Ewing sarcoma
D) Osteosarcoma
E) Fibrosarcoma
(Answer: A)
63. Which of the following is incorrect regarding von Recklinghausen's disease?
A) It is autosomal dominant
B) Multiple nodules are present
C) Sciatic nerve involvement
D) Malignant peripheral nerve sheath tumors can occur
E) Neuroblastoma can occur
Neuroblastoma is the most common extracranial solid organ tumor in infancy and
childhood, frequently showing calcification. Pathologically, rosettes and neurofibrillary
formation are characteristic. Neuroblastoma is unrelated to von Recklinghausen's disease.
Von Recklinghausen's disease is autosomal dominant, can present with multiple nodules
(cutaneous neurofibromas), and may involve peripheral nerves, including the sciatic nerve.
Café-au-lait spots aid in diagnosis. 4-5% of patients develop malignant peripheral nerve
sheath tumors (MPNST), with half of MPNSTs associated with this disease. (Answer: E)
64. Which is the most common location for rhabdomyosarcoma?
A) Head and neck
B) Paratesticular region
C) Retroperitoneum
D) Lower extremities
E) Upper extremities
Most tumors of striated muscle are malignant. Benign rhabdomyomas are more common
in tuberous sclerosis patients. Rhabdomyosarcoma is the most common soft tissue sarcoma
in individuals under 15 and remains common in those under 45. The head and neck region
accounts for 44% of cases, followed by the paratesticular region, retroperitoneum, lower, and
upper extremities. Rhabdomyoblasts are present in the tumor. Ultrastructurally,
rhabdomyoblasts contain sarcomeres and are immunohistochemically positive for MyoD1
361 Pathology Questions and Answers, Prof.Dr. Murat ALPER
and myogenin. Embryonal rhabdomyosarcoma is the most common type, representing 60%
of cases, typically found in the head and neck in individuals under 15. Botryoid
rhabdomyosarcoma is a subtype of embryonal rhabdomyosarcoma, most commonly
occurring in the vagina and bladder. It contains a Cambium layer with a submucosal
concentration of tumor cells. Alveolar rhabdomyosarcoma is more common in extremities in
individuals aged 15-25. The pleomorphic type is more common in those over 40. Good
prognostic factors include embryonal and especially botryoid types, young age, extravesical
genitourinary and orbital sites, while poor prognostic factors include adulthood, head and
neck, abdomen, extremities, and alveolar and pleomorphic types. (Answer: A)
65. Which is the most common malignant soft tissue tumor in children?
A) Rhabdomyosarcoma
B) Liposarcoma
C) Leiomyosarcoma
D) Synovial sarcoma
E) Malignant fibrous histiocytoma
Rhabdomyosarcoma is commonly found in the head-neck region, genitourinary tract,
retroperitoneum, and extremities. It is the most frequent soft tissue sarcoma in children and
adolescents, particularly in the head-neck region during childhood. Rhabdomyosarcoma is
most common between ages 0–35, particularly in those aged 0–15. Synovial sarcoma is most
common between ages 5–45, particularly between 15–30. Malignant fibrous histiocytoma is
most common between ages 15–70, especially over 45. Fibrosarcoma is most common
between ages 15–40, and liposarcoma is most common between ages 25–55. (Answer: A)
66. Which statement about synovial sarcoma is incorrect?
A) It is most common between ages 15-35
B) It is common in the lower extremity
C) It has a clinical history of a few weeks
D) It shows a biphasic pattern
E) It has a poor prognosis
Synovial sarcoma can occur at any age but is most common between 20–40. There is a
reciprocal translocation between the X chromosome and chromosome 18: t(X;18)
(p11.2;q11.2). It is most common in the lower extremities, especially around the knee, foot,
362 Pathology Questions and Answers, Prof.Dr. Murat ALPER
and ankle, where tendons, joints, or bursae are found. It accounts for 10% of soft tissue
sarcomas. 10% occur in non-joint areas like the parapharyngeal region and abdominal wall. It
is often well-defined and may be painful. It typically has a long clinical history before
diagnosis, ranging from 6 months to 2 years, sometimes up to 10 years. Radiologically,
calcifications may be seen. The prognosis is poor, with frequent metastasis to the lungs.
Histologically, it exhibits a biphasic or monophase pattern of epithelial and spindle cells.
Unlike other sarcomas, tumor cells stain positively for epithelial markers like keratin and
EMA. (Answer: C)
### Review Topics:
Achondroplasia: A genetic disorder affecting bone growth due to a mutation in the FGFR3
gene, leading to disproportionate short stature, with normal torso and shortened limbs. It is
the most common form of dwarfism.
Osteogenesis Imperfecta: A group of genetic disorders affecting collagen production,
characterized by fragile bones that break easily, blue sclerae, hearing loss, and dental issues.
It involves defects in type I collagen.
Osteopetrosis: A rare inherited disorder characterized by increased bone density due to
defective osteoclasts, leading to brittle bones, anemia, recurrent infections, and nerve
compression syndromes.
Osteoporosis: A condition characterized by decreased bone mass and density, leading to
fragile bones and an increased risk of fractures. It is common in postmenopausal women and
the elderly.
Paget's Disease: A chronic bone disorder characterized by abnormal bone remodeling,
leading to enlarged and misshapen bones. It can cause bone pain, deformities, and an
increased risk of fractures and osteosarcoma.
Rickets/Osteomalacia: Rickets refers to defective mineralization of the growing skeleton in
children, often due to vitamin D deficiency, while osteomalacia is the adult equivalent,
leading to soft and weakened bones.
363 Pathology Questions and Answers, Prof.Dr. Murat ALPER
Hyperparathyroidism: A condition characterized by excessive parathyroid hormone
production, leading to increased bone resorption, hypercalcemia, and a variety of skeletal
changes, including osteitis fibrosa cystica.
Fractures: Disruptions in the continuity of bone, typically caused by trauma, but also due to
pathological conditions that weaken bones.
Osteonecrosis: The death of bone tissue due to a lack of blood supply, often affecting the
femoral head, leading to pain and joint dysfunction.
Osteomyelitis: A bacterial or fungal infection of the bone, commonly caused by
Staphylococcus aureus, leading to bone destruction and systemic symptoms.
Osteoma: A benign bone tumor commonly found in the skull and facial bones, characterized
by a growth of compact or cancellous bone.
Osteoid Osteoma: A small, benign, and painful bone tumor, often found in the long bones,
typically relieved by NSAIDs.
Osteoblastoma: A rare benign bone tumor similar to osteoid osteoma but larger, typically
affecting the spine.
Osteosarcoma: A malignant bone tumor commonly affecting adolescents and young adults,
typically arising in the metaphysis of long bones around the knee.
Osteochondroma: A benign bone tumor characterized by a cartilage-capped bony projection,
usually occurring in the metaphysis of long bones.
Chondroma/Enchondroma: A benign cartilage tumor typically found in small bones of the
hands and feet, often asymptomatic unless causing deformity or fracture.
Chondroblastoma: A rare, benign, but locally aggressive bone tumor affecting the epiphysis
of long bones, typically in adolescents.
364 Pathology Questions and Answers, Prof.Dr. Murat ALPER
Chondrosarcoma: A malignant cartilage-producing tumor most commonly found in the pelvis
and proximal femur, typically affecting older adults.
Fibrous Dysplasia: A bone disorder where normal bone and marrow are replaced with fibrous
tissue, leading to bone deformities and fractures.
Undifferentiated Pleomorphic Sarcoma (UPS): A high-grade malignant soft tissue tumor
previously classified as malignant fibrous histiocytoma (MFH), characterized by pleomorphic
spindle cells.
Ewing Sarcoma/PNET (Primitive Neuroectodermal Tumor): A malignant bone tumor primarily
affecting children and adolescents, characterized by small round blue cells and often
associated with the t(11;22) translocation.
Giant Cell Tumor of Bone: A typically benign but locally aggressive tumor characterized by
multinucleated giant cells, usually occurring in the epiphysis of long bones in young adults.
Osteoarthritis: A degenerative joint disease characterized by the breakdown of articular
cartilage, leading to joint pain and stiffness.
Rheumatoid Arthritis: An autoimmune disease characterized by chronic inflammation of the
synovium, leading to joint destruction, pain, and deformity.
Seronegative Spondyloarthropathies: A group of inflammatory rheumatic diseases affecting
the spine and peripheral joints, often associated with the HLA-B27 antigen.
Arthritis: Inflammation of one or more joints, causing pain and stiffness, encompassing a
wide range of conditions, including osteoarthritis, rheumatoid arthritis, and gout.
Gout: A metabolic disorder characterized by recurrent episodes of acute arthritis due to
deposition of monosodium urate crystals in joints, caused by hyperuricemia.
365 Pathology Questions and Answers, Prof.Dr. Murat ALPER
Villonodular Synovitis: A benign proliferative disorder of the synovium, leading to joint
swelling, pain, and reduced mobility, typically affecting large joints.
Lipoma: A benign tumor composed of mature adipose tissue, commonly found in the
subcutaneous tissue, presenting as a soft, mobile mass.
Liposarcoma: A malignant tumor of adipose tissue, often occurring in deep soft tissues of the
limbs and retroperitoneum, exhibiting varying degrees of differentiation.
Nodular Fasciitis: A self-limiting, rapidly growing benign soft tissue lesion composed of
fibroblasts and myofibroblasts, often misdiagnosed as a sarcoma.
Myositis Ossificans: A benign condition characterized by heterotopic ossification within
muscle tissue, often following trauma.
Dupuytren's Contracture: A condition characterized by thickening and shortening of the
palmar fascia, leading to flexion contractures of the fingers.
Desmoid Fibromatosis: A locally aggressive but non-metastasizing fibroblastic proliferation,
often associated with Gardner syndrome, occurring in the abdominal wall or extremities.
Fibromas/Fibrosarcomas: Fibromas are benign tumors composed of fibrous tissue, while
fibrosarcomas are malignant, characterized by spindle-shaped fibroblasts.
Rhabdomyosarcoma: A malignant tumor of skeletal muscle origin, most common in children,
with several subtypes including embryonal, alveolar, and pleomorphic.
Leiomyoma/Leiomyosarcoma: Leiomyomas are benign smooth muscle tumors, often found
in the uterus, while leiomyosarcomas are malignant and can occur in various locations,
including the retroperitoneum and extremities.
Synovial Sarcoma: A malignant soft tissue tumor typically occurring near joints, characterized
by biphasic or monophasic histological patterns, affecting young adults.
366 Pathology Questions and Answers, Prof.Dr. Murat ALPER
SECTION 25: SKIN PATHOLOGY
1. What is the term for the hyperplasia of the stratum corneum?
A) Parakeratosis
B) Hyperkeratosis
C) Acanthosis
D) Papillomatosis
E) Spongiosis
(Answer: B)
2. What is the term for intercellular edema in the epidermis?
A) Hyperkeratosis
B) Acanthosis
C) Parakeratosis
D) Spongiosis
E) Exocytosis
(Answer: D)
3. Which term describes epidermal hyperplasia?
A) Parakeratosis
B) Hyperkeratosis
C) Acanthosis
D) Spongiosis
E) Exocytosis
(Answer: C)
4. Where is parakeratosis considered normal?
A) Facial skin
367 Pathology Questions and Answers, Prof.Dr. Murat ALPER
B) Back of the hand
C) Ankle
D) Inner elbow
E) Mucous membranes
Hyperkeratosis is the hyperplasia and thickening of the stratum corneum.
Parakeratosis is the presence of nucleated cells in the stratum corneum. Acanthosis is
the hyperplasia of the epidermis. Spongiosis is intercellular edema in the epidermis.
Exocytosis is the invasion of the epidermis by inflammatory cells and erythrocytes.
(Answer: E)
5. In a patient with microscopic findings of spongiosis in the epidermis and
perivascular lymphocytes and eosinophils in the upper dermis, what is the most likely
diagnosis?
A) Acute eczematous dermatitis
B) Psoriasis
C) Lichen planus
D) Vitiligo
E) Mycosis fungoides
Eczema is a term for a group of pathologies with different etiologies,
characterized by erythematous papules and itching. Subtypes include: Allergic contact
dermatitis: a delayed-type hypersensitivity reaction to topical allergen exposure.
Atopic dermatitis: occurs due to exposure to potential antigens or allergens. Drug-
induced eczematous dermatitis: a drug reaction. Photocontact dermatitis: an
abnormal response to UV or light. Primary irritant dermatitis: results from exposure
to chemical, physical, or mechanical damage. Atopic dermatitis often includes a
genetic predisposition and may persist for years, while others have an acute and
contact-related history. Most atopic dermatitis cases begin in childhood and are
associated with the atopic triad, including asthma and allergic rhinitis. The most
common form of eczema is allergic contact dermatitis, characterized by spongiosis,
i.e., intercellular edema, and sometimes spongiotic vesicles may develop. The dermis
contains perivascular lymphocytes, a few mast cells, and occasionally eosinophils,
particularly in drug reactions. The epidermis may be thickened (acanthotic) and the
stratum corneum may be thickened (hyperkeratotic). Lichen simplex chronicus may
develop due to intense scratching. (Answer: A)
368 Pathology Questions and Answers, Prof.Dr. Murat ALPER
6. In which of the following is there an increase in melanocytes?
A) Albinism
B) Vitiligo
C) Lentigo
D) Melasma
E) Freckle
(Answer: C)
7. Which disease involves melanocytes but lacks melanin production due to a
defect in the tyrosinase enzyme?
A) Albinism
B) Vitiligo
C) Lentigo
D) Freckle
E) Melasma
(Answer: A)
8. What is the term for linear melanocytic hyperplasia in the basal cell layer?
A) Albinism
B) Vitiligo
C) Lentigo
D) Freckle
E) Melasma
(Answer: C)
9. What is the term for facial hyperpigmentation seen during pregnancy?
A) Albinism
B) Vitiligo
369 Pathology Questions and Answers, Prof.Dr. Murat ALPER
C) Lentigo
D) Mastocytosis
E) Melasma
(Answer: E)
10. In which disease is there partial or complete absence of melanocytes in
the epidermis?
A) Albinism
B) Vitiligo
C) Lentigo
D) Freckle
E) Melasma
Vitiligo is characterized by the complete or partial absence of melanocytes in
the epidermis. In most forms of albinism, melanocytes are present but melanin is not
produced due to a defect or deficiency in the tyrosinase enzyme. Freckle (ephelis) is
the most common pigmented lesion in children, appearing in sun-exposed areas. It
can increase in summer and decrease in winter, distinguishing it from lentigo.
Melasma is facial hyperpigmentation seen during pregnancy, also known as the
pregnancy mask. It can also occur in those using oral contraceptives. Lentigo is linear
melanocytic hyperplasia in the basal cell layer. It may be a simple increase or part of a
malignant lesion. Vitiligo is frequently associated with autoimmune diseases such as
pernicious anemia, Addison's disease, and autoimmune thyroiditis. (Answer: B)
11. Which of the following is a chronic disease characterized by symmetric
pigment loss around the mouth and in trauma-exposed areas?
A) Acanthosis nigricans
B) Vitiligo
C) Pemphigus
D) Epidermolysis bullosa
E) Mastocytosis
370 Pathology Questions and Answers, Prof.Dr. Murat ALPER
Vitiligo is a chronic disease characterized by symmetric pigment loss around
the mouth and in trauma-exposed areas. The disease affects any pigment-containing
areas, including the choroid layer of the eye, meninges, and possibly through an
autoimmune mechanism. It has been suggested to be autosomal dominant, but there
is evidence of polygenic inheritance. Biopsy of the depigmented area shows total loss
of melanocytes and no melanin in keratinocytes. In the margins of lesions and areas
with partial pigment loss, damaged, edematous, and deformed melanocytes are
observed. Clinically, in erythematous bordered lesions, there is slight vascular dilation
and sparse lymphocytic infiltration around superficial vessels. Sometimes,
repigmentation occurs, progressing inward from the periphery due to melanocyte
proliferation from hair follicles. Most vitiligo patients have antibodies against
melanocytes. The differential diagnosis includes Ito's hypomelanosis. (Answer: B)
12. Which skin lesion is characterized by metachromatic staining cells?
A) Mastocytosis
B) Pemphigus vulgaris
C) Acne rosacea
D) Pityriasis rosea
E) Keratoacanthoma
Mastocytosis shows Darier's sign and dermatographism. It forms lesions
containing mast cells in the tissue. Mast cells are metachromatic with Toluidine Blue
and Giemsa stains. (Answer: A)
13. In a patient with joint pain, erythematous plaque-like skin lesions are
found on both knees and elbows. Skin biopsy reveals regular acanthosis,
parakeratosis, and suprapapillary thinning in the epidermis. What is the most likely
diagnosis?
A) Lichen planus
B) Erythema multiforme
C) Pemphigus vulgaris
D) Psoriasis vulgaris
E) Dermatitis herpetiformis
(Answer: D)
371 Pathology Questions and Answers, Prof.Dr. Murat ALPER
14. In which of the following diseases are Munro microabscesses seen?
A) Lichen planus
B) Lupus erythematosus
C) Lichen simplex chronicus
D) Psoriasis
E) Pemphigus
Psoriasis is a common chronic inflammatory dermatosis. It most often occurs
on the wrist, knees, scalp, gluteal region, glans penis, and vulva. Nail involvement is
seen in 30% of patients. Arthritis, myopathy, enteropathy, spondylitic heart disease,
and frequent occurrence in AIDS are noted. A rare variant, pustular psoriasis, can be
life-threatening with fever, leukocytosis, arthralgia, diffuse cutaneous and mucosal
pustules, secondary infection, and electrolyte imbalance. In psoriasis, the stratum
granulosum is absent or very thin, and there is intense parakeratosis. Munro
microabscesses with neutrophil accumulation in the stratum corneum and
spongiform pustules (Kogoj) are seen. Since fungi can produce similar findings, PAS
staining is used for differentiation. Clinically, due to suprapapillary epidermal thinning,
pinpoint bleeding upon removal of the keratin layer (Auspitz sign) and lesion
formation due to local trauma (Koebner phenomenon) in predisposed individuals are
present. In this disease affecting 1% of the population, the pathology is increased cell
turnover. In psoriasis, cell turnover is excessively accelerated. TNF is the most
important mediator in pathogenesis. Lichen simplex chronicus is an important type of
chronic spongiotic dermatitis in the differential diagnosis of psoriasis. It presents with
hyperkeratosis, acanthosis, and hypergranulosis. Rete ridges are elongated. The
nodular form is prurigo nodularis. (Answer: D)
15. If a patient with symmetrically distributed skin lesions on the extremities
and oral lesions shows a sawtooth appearance in the reticular layer and Civatte
bodies with band-like infiltration in the upper dermis on skin biopsy, what is the most
likely diagnosis?
A) Psoriasis
B) Scleroderma
C) Lupus erythematosus
D) Acne vulgaris
E) Lichen planus
372 Pathology Questions and Answers, Prof.Dr. Murat ALPER
The findings described are typical for lichen planus. Known as the disease of
"P's": prur
itic, purple, polygonal, planar papules and plaques. It is a disease of the skin
and squamous mucosa. Wickham striae are seen. Microscopically, it is an interface
dermatitis, characterized by significant basal cell damage. There is a sawtooth-like
elongation of the rete ridges, hyperkeratosis, hypergranulosis, eosinophilic apoptotic
bodies called Civatte bodies in the epidermis, melanophages, and dense band-like
lymphocytic infiltration in the upper dermis. There is also an increased risk of
squamous cell carcinoma, particularly in mucosal lesions. In scleroderma, there is
excessive connective tissue, and skin appendages are absent or minimal. (Answer: E)
16. In pemphigus vulgaris, where do lesions appear first?
A) Scalp
B) Face
C) Oral mucosa
D) Trunk
E) Pressure areas
Pemphigus is an autoimmune bullous disease that can sometimes be fatal. It is
a type II antibody-mediated hypersensitivity reaction. The most common type is
pemphigus vulgaris. It occurs on the scalp, face, axillae, trunk, and pressure areas.
Oral lesions may appear months before skin lesions. Histologically, pemphigus vulgaris
shows acantholysis, suprabasal acantholytic blisters, and intercellular IgG
accumulation in studies. Blisters in pemphigus vulgaris, unlike bullous pemphigoid,
rupture easily. The Nikolsky sign is positive. Paraneoplastic pemphigus clinically
resembles pemphigus vulgaris or erythema multiforme and is associated with
lymphoid diseases such as non-Hodgkin lymphoma, chronic lymphocytic leukemia,
and Castleman disease, as well as thymomas, spindle cell sarcomas, and
Waldenström macroglobulinemia. (Answer: C)
17. In a seventy-year-old patient, bullous lesions are detected on the skin, and
later lesions develop on the oral mucosa. If blisters do not rupture on examination
and histologically subepidermal non-acantholytic lesions are observed, what is the
most likely diagnosis?
373 Pathology Questions and Answers, Prof.Dr. Murat ALPER
A) Dermatitis herpetiformis
B) Epidermolysis bullosa
C) Psoriasis
D) Bullous pemphigoid
E) Pemphigus vulgaris
In pemphigus vulgaris, oral lesions precede skin lesions, and suprabasal
separation is observed. Separation in pemphigus foliaceus is subcorneal. Bullous
pemphigoid is an autoimmune disease seen in older individuals. Blisters appear first
on the skin, then in the mouth. Blisters, unlike those in pemphigus, do not rupture
easily and heal without scarring if they become infected. Separation is subepidermal
and non-acantholytic. Eosinophils are abundant. Bullous pemphigoid shows linear
deposition of immunoglobulin and complement at the basal membrane. Dermatitis
herpetiformis is characterized by urticaria and vesicles and is often associated with
celiac disease. IgA deposits are observed in the dermal papillae. (Answer: D)
18. In which of the following diseases is ochronosis seen?
A) Phenylketonuria
B) Alkaptonuria
C) Fabry disease
D) Neurofibromatosis
E) Lesch-Nyhan disease
Ochronosis is dark pigmentation in cartilage and fibrous tissue. Alkaptonuria is
due to homogentisic acid oxidase deficiency. Homogentisic acid is excreted in the
urine, resulting in dark-colored urine. Ochronosis is observed. Lesch-Nyhan syndrome
occurs due to hypoxanthine-guanine phosphoribosyltransferase deficiency and is
characterized by gout, mental retardation, choreoathetosis, spasticity, and self-
mutilation. In Fabry disease, there is a deficiency of alpha-galactosidase A and
accumulation of ceramide trihexoside, leading to skin lesions such as angiokeratoma
and severe burning extremity pain. (Answer: B)
19. In a patient with intensely itchy vesicles and IgA deposits in the dermal
papillae, what is the most likely diagnosis?
A) Bullous pemphigoid
374 Pathology Questions and Answers, Prof.Dr. Murat ALPER
B) Ichthyosis
C) Dermatitis herpetiformis
D) Lichen planus
E) Epidermolysis bullosa
Dermatitis herpetiformis (Duhring disease) is a chronic skin disease
characterized by intensely itchy vesicles, papules, and urticarial-like lesions.
Subepidermal separation and IgA deposits in the dermal papillae are observed. In
80% of patients, there is gluten-sensitive enteropathy (celiac disease), some of which
is asymptomatic. Ichthyosis involves excessive scaling of the skin and is a feature of
Refsum syndrome, a hereditary ataxia characterized by phytanic acid deficiency and
polyneuritic changes, and Sjögren-Larsson syndrome, which includes hereditary
mental deficiency and spastic paralysis. Both syndromes are autosomal recessive.
Dermatitis herpetiformis is a rare blistering disease associated with dietary gluten
sensitivity and IgA mediation. Lesions are very itchy, with small vesicles symmetrically
distributed on the extensor surfaces of the legs, lower trunk, and buttocks. They form
clusters reminiscent of herpes simplex or zoster infection, hence the name
"herpetiformis," although there is no relation to the herpes virus. Blisters begin at the
tips of the dermal papillae and coalesce to form larger vesicles and bullae.
Neutrophils are prominent in the inflammation, with fewer eosinophils than in
bullous pemphigoid. Immunofluorescence microscopy reveals granular IgA deposits
at the tips of the dermal papillae, particularly in areas adjacent to clinically blistered
sites. Immune complex deposits are located beneath the lamina densa of the
epidermal basement membrane. The basal membrane is damaged in areas where
neutrophils cluster and vesiculation occurs. (Answer: C)
20. Which of the following can cause mutations and cancer by forming
pyrimidine dimers in the DNA chain?
A) Ultraviolet radiation
B) Ionizing radiation
C) Alkylating agents
D) Asbestos
E) Iodine-131
(Answer: A)
21. Which disease is characterized by a defect in DNA repair?
375 Pathology Questions and Answers, Prof.Dr. Murat ALPER
A) Discoid lupus erythematosus
B) Lichen planus
C) Seborrheic keratosis
D) Xeroderma pigmentosum
E) Kaposi's sarcoma
Xeroderma pigmentosum (XP) refers to several diseases characterized by
defects in repairing DNA damage caused by ultraviolet (UV) radiation. In the primary
type of XP, there is an absence of the endonuclease enzyme required to eliminate UV-
induced pyrimidine dimers. Rarer forms of the disease, caused by defects in other
stages of DNA repair, have also been described. In affected children, numerous
lentigines, solar keratoses, squamous and basal cell carcinomas, and malignant
melanomas develop on sun-exposed skin. Even minimal sunlight exposure can cause
such lesions, which are associated with very little solar elastosis. There is also a slight
increase in the frequency of deep dermal tumors and soft tissue tumors. UV damage
affects many genes and chromosomes, including the N-ras oncogene. In squamous
cell carcinomas, it has been shown that UV radiation can cause mutations in the p53
tumor suppressor gene. (Answer: D)
22. Which of the following characterizes a pilonidal sinus?
A) Supradermal blister formation
B) Cysts on the face, axillae, and buttocks
C) Congenital collagen defect
D) Increased adipose tissue
E) Foreign body giant cells around hair
(Answer: E)
23. A superficial bacterial infection of the epidermis is expressed as a clinical
term. It begins as small papules 1-2 mm in diameter and develops into small, thin-
roofed vesicles. Upon rupture, the vesicles form a yellow or honey-colored crust.
Which of the following diseases is described?
A) Impetigo
B) Vitiligo
376 Pathology Questions and Answers, Prof.Dr. Murat ALPER
C) Tinea (dermatophytosis)
D) Epidermolysis bullosa
E) Mastocytosis
Impetigo is a clinical term expressing a superficial bacterial infection of the
epidermis. Impetigo contagiosa (non-bullous type) starts as small papules 1-2 mm in
diameter and develops into small, thin-roofed vesicles. The vesicles rupture and form
a yellow or honey-colored crust. There is an erythema resulting from inflammation in
the dermis. These papules coalesce to form plaques of varying sizes and sometimes
heal from the center. Most lesions are caused by group A beta-hemolytic streptococci,
either alone or in conjunction with Staphylococcus aureus. Conversely, bullous
impetigo contagiosa is caused by S. aureus. These cocci produce an exotoxin, called
exfoliatin, which separates cells in the upper spinous layer and causes epidermal
peeling. A focus of internal infection by exfoliatin-producing staphylococci can lead to
staphylococcal scalded skin syndrome, a widespread epidermal separation disease.
This syndrome especially occurs in children with immature kidneys or in adults with
severe renal failure who cannot eliminate the exfoliatin toxin. (Answer: A)
24. In which skin lesion are koilocytotic cells observed?
A)
Epidermolysis bullosa
B) Molluscum contagiosum
C) Condyloma acuminatum
D) Erythema chronicum migrans
E) Porphyria
Lesions caused by HPV most commonly include verruca vulgaris and
condyloma acuminatum. Verruca vulgaris is common on the dorsal hands and
periungual areas. Verruca palmaris occurs on the palm and verruca plantaris on the
sole of the foot. Koilocytotic cells, characterized by cytoplasmic vacuoles, are seen in
condyloma acuminatum (venereal wart), caused by human papillomavirus. There are
keratohyalin granules, papillomatosis, and parakeratosis. It forms soft, cauliflower-like
lesions on the penis, vulva, urethra, perianal area, and rectum. HPV types 6 and 11
cause warts, while types 16 and 18 are implicated in the etiology of cervical
carcinoma. Erythema chronicum migrans is seen in Lyme disease. In molluscum
377 Pathology Questions and Answers, Prof.Dr. Murat ALPER
contagiosum, diagnostic molluscum bodies are observed with Giemsa staining.
(Answer: C)
25. Which histochemical method is used to demonstrate fungi in the skin?
A) Trichrome
B) Congo red
C) Oil red O
D) PAS
E) Reticulin
Superficial fungal infections may have an annular appearance; however, they
can mimic psoriasiform and eczematous dermatoses as pruritic lesions. The effect of
fungal growth in the stratum corneum varies with changes in epidermal thickness or
the amount of inflammation in the dermis. In tinea versicolor, the combination of
yeast or hyphal forms 1-3 microns thick gives the lesion a distinctive appearance. The
fungi of tinea nigra have slightly thicker hyphae and contain brown pigment. The use
of PAS stains is recommended to identify fungi, especially when they are not readily
apparent. (Answer: D)
26. Which inflammatory disease of the sebaceous follicles frequently destroys
follicles and leads to scarring in the skin?
A) Acne vulgaris
B) DLE
C) Acne rosacea
D) Alopecia
E) Impetigo
Acne vulgaris is an inflammatory disease of the sebaceous follicles that
frequently destroys follicles and leads to scarring in the skin. It typically occurs in
adolescence but can persist into adulthood. The most common sites are the face,
anterior chest, and upper back. Deep nodules and pustules, cysts, and sinuses
opening to the surface (cystic acne) occur in some individuals. Its etiology is complex,
involving hormonal, bacteriological, and genetic factors. The tendency for acne is
inherited in a polygenic manner with variable penetrance approaching autosomal
dominance. Genetic factors and androgenic hormones increase sebum production in
378 Pathology Questions and Answers, Prof.Dr. Murat ALPER
sebaceous follicles. Bacteria, particularly Propionibacterium acnes (previously
Corynebacterium acnes, known as type I) in the normal flora, break down
triglycerides in sebum into fatty acids, leading to follicular hyperkeratosis, comedone
formation, and eventual follicular rupture. Extrusion and rupture from the follicle
elicit an intense inflammatory cell response. (Answer: A)
27. In a seventy-year-old male patient with widespread brown protrusions on
the skin and biopsy showing epidermal thickening and keratin cysts, what is the most
likely diagnosis?
A) Nevocellular nevus
B) Basal cell carcinoma
C) Malignant melanoma
D) Lentigo senilis
E) Seborrheic keratosis
(Answer: E)
28. In the Leser-Trelat sign, which of the following lesions suddenly and
excessively increases?
A) Solar lentigo
B) Actinic keratosis
C) Dysplastic nevus
D) Seborrheic keratosis
E) Squamous papilloma
(Answer: D)
29. In a sixty-eight-year-old male patient with scaling lesions on the face and
frontal region, biopsy from the raised, dark brown, well-defined lesion reveals
epidermal thickening, epidermal keratinocytes with increased melanin pigment
developing towards the surface and dermis, and small keratin cysts. What is the most
likely diagnosis?
A) Lentigo maligna
B) Basal cell carcinoma
379 Pathology Questions and Answers, Prof.Dr. Murat ALPER
C) Keratoacanthoma
D) Verruca vulgaris
E) Seborrheic keratosis
(Answer: E)
30. Which skin lesion consists of cells resembling basal cells and contains small
keratin cysts?
A) Seborrheic keratosis
B) Polyp
C) Acanthosis nigricans
D) Melasma
E) Psoriasis
Seborrheic keratosis occurs in adults and the elderly, especially on the trunk,
extremities, head, and neck. It consists of monotonous cells resembling basal layer
cells and contains keratin cysts. It may contain varying degrees of pigment, leading to
clinical confusion with malignant melanoma. In individuals with internal malignancies,
numerous seborrheic keratoses with unusual appearances may suddenly develop on
the upper trunk, sometimes referred to as the "Leser-Trelat sign," and may be
associated with various stomach, lung, and colon carcinomas and lymphomas.
Acanthosis nigricans involves thickening and hyperpigmentation of skin areas,
typically occurring on flexural surfaces. It is often associated with pituitary, pineal
gland tumors, and diabetes, and it may show autosomal dominant inheritance. It can
sometimes be associated with an internal organ adenocarcinoma. (Answer: A)
31. Which skin lesion can histologically mimic squamous cell carcinoma and
heal spontaneously without treatment?
A) Basal cell carcinoma
B) Epidermal cyst
C) Pilar cyst
D) Keratoacanthoma
E) Dermoid cyst
380 Pathology Questions and Answers, Prof.Dr. Murat ALPER
Keratoacanthoma is a rapidly progressing skin condition that can heal
spontaneously without treatment. It histologically resembles well-differentiated
squamous cell carcinoma. It is more common in men over 50 and sun-exposed areas.
Histologically, it shows craters filled with keratin and squamous cells. The fact that
lesions do not last longer than 5-6 months helps differentiate keratoacanthoma from
carcinoma. In skin diseases, especially non-tumoral conditions, several diseases may
produce similar appearances, so clinical presumptive diagnoses are important in
differential diagnosis. (Answer: D)
32. In which disease does localized mast cell degranulation play a role, leading
to increased permeability in dermal vessels?
A) Eczema
B) Urticaria
C) Psoriasis
D) Lichen planus
E) Dermatitis herpetiformis
The mechanism of urticaria is described in the question. It is also characterized
by itchy, edematous, plaque-like lesions. In most cases, it is an example of type I
hypersensitivity. Mast cells and IgE play a role. Responsible antigens include viruses,
pollens, foods, drugs, and insect-related factors. Histopathology shows edema and
mononuclear inflammatory cells. Eosinophils may sometimes be present.
Angioedema is a related concept to urticaria, where edema occurs deeper in the
dermis and subcutaneous fat tissue. Eczema involves intercellular fluid accumulation
in the epithelium, leading to spongiotic dermatitis. (Answer: B)
33. In which lesion are foamy cytoplasm histiocytes densely present in the
dermis?
A) Dermatofibroma
B) Dermatofibrosarcoma protuberans
C) Xanthoma
D) Cutaneous T-cell lymphoma
E) Mastocytosis
381 Pathology Questions and Answers, Prof.Dr. Murat ALPER
Xanthoma involves the dense accumulation of foamy cytoplasm histiocytes in
the dermis. It can be associated with familial or acquired hyperlipidemia and
lymphoid malignancies. The cells may contain cholesterol, triglycerides, and
phospholipids. Dermatofibroma is the most common form of benign fibrous
histiocytoma. Dermatofibrosarcoma protuberans is a well-differentiated dermal
fibrosarcoma, most commonly seen on the trunk. Despite its slow growth, it exhibits
locally aggressive behavior. Cutaneous T-cell lymphoma, another name for mycosis
fungoides, involves T helper cells in the skin. Sézary-Lützner cells and Pautrier
microabscesses are observed. (Answer: C)
34. Which of the following tumors exhibits apocrine differentiation and is
often located on the forehead and scalp?
A) Eccrine poroma
B) Cylindroma
C) Syringoma
D) Trichoepithelioma
E) Sebaceous carcinoma
Cylindroma is a skin appendage tumor with apocrine differentiation, typically
occurring on the forehead and scalp. Over time, nodules may merge to form a
"turban tumor." Syringoma shows eccrine differentiation, particularly around the
lower eyelid. Trichoepithelioma is a skin appendage tumor with follicular
differentiation. Sebaceous carcinoma is a malignant tumor that develops from the
Meibomian gland of the eyelid. Eccrine poroma is located especially on the palms and
soles. (Answer: B)
35. Which neoplastic skin disease forms plaques and tumors in the skin,
characterized by T-cell infiltration in the dermis and epidermis?
A) Mycosis fungoides
B) Psoriasis vulgaris
C) Lichen planus
D) Pemphigus vulgaris
E) Mastocytosis
(Answer: A)
382 Pathology Questions and Answers, Prof.Dr. Murat ALPER
36. In which tumor are Séz
ary-Lützner cells observed?
A) Squamous cell carcinoma
B) Basal cell carcinoma
C) Eccrine poroma
D) Syringoma
E) Mycosis fungoides
Mycosis fungoides is a cutaneous T-cell lymphoma. CD4+ lymphocytes
proliferate. Sézary-Lützner cells are present, and the invasion of these cells in the
epidermis leads to Pautrier microabscesses. HTLV-1 may play a role in the etiology.
(Answer: E)
37. Which skin cancer is actinic keratosis most likely to progress into?
A) Basal cell carcinoma
B) Malignant melanoma
C) Squamous cell carcinoma
D) Sebaceous carcinoma
E) Merkel cell carcinoma
(Answer: C)
38. Which of the following diseases can lead to cancer development,
particularly in fair-skinned individuals?
A) Solar elastosis
B) Porphyria
C) Alkaptonuria
D) Actinic keratosis
E) Lichen planus
383 Pathology Questions and Answers, Prof.Dr. Murat ALPER
Actinic keratosis, also known as "senile keratosis" or "solar keratosis," is a
scaly, erythematous patch commonly found on sun-exposed skin, being one of the
most common neoplasms in fair-skinned individuals. Typically less than 1 cm, it shows
pronounced cytological atypia, particularly in the lower part of the epidermis, along
with dysplasia and basal cell hyperplasia. When cytological atypia extends throughout
the epidermis, it is referred to as "squamous cell carcinoma in situ." Lesions may be
hyperpigmented, extensive, and confluent. It is directly related to cumulative doses of
UVB radiation (especially in the 280-320 nm wavelength range). Other forms of
ionizing radiation can produce similar keratoses. Patients exposed to inorganic arsenic
compounds may also develop similar keratoses, appearing on sun-protected skin,
including the palms and soles. When seen on the lips, it is called actinic cheilitis.
Actinic keratosis is part of a continuum that can progress to squamous cell carcinoma
in situ. About 0.1-2.6% per year progress to squamous cell carcinoma, while most
regress or remain stable. Lesions begin with hyperchromasia and nuclear
enlargement of basal keratinocytes at the dermoepidermal junction. Over time,
acanthosis develops as clearly atypical cells ascend to the upper layers of the
epidermis. When atypical cells fill the entire epidermal thickness, and the lesion is
extensive, covering hair follicles and sweat gland ducts, the diagnosis of squamous
cell carcinoma is more appropriate. UV radiation can cause mutations in the tumor
suppressor gene p53, which may be one of the reasons for squamous cell carcinoma
development in sun-exposed skin. (Answer: D)
39. In which tumor are globe corne structures observed?
A) Basal cell carcinoma
B) Eccrine poroma
C) Merkel cell carcinoma
D) Squamous cell carcinoma
E) Malignant melanoma
SCC occurs in older individuals on sun-exposed areas. UV radiation causes DNA
damage, and patients with xeroderma pigmentosum have a high risk early in life.
High-risk HPV types 16, 18, etc., arsenic, chronic wound ulcers, burn scars, and
ionizing radiation are risk factors. Globe corne (keratin pearls) and single-cell
keratinization are observed in squamous cell carcinoma. These two features and
intercellular bridges are indications of good differentiation in squamous cell
carcinoma. This applies to squamous cell carcinoma in all organs. (Answer: D)
384 Pathology Questions and Answers, Prof.Dr. Murat ALPER
40. Which of the following more frequently shows localized ulceration in the
skin?
A) Basal cell carcinoma
B) Squamous cell carcinoma
C) Small cell carcinoma
D) Apocrine carcinoma
E) Eccrine poroma
(Answer: A)
41. Which of the following statements about basal cell carcinoma is false?
A) It grows slowly.
B) It rarely metastasizes.
C) It forms a rodent ulcer.
D) It is associated with papillomavirus.
E) It is related to sunlight.
Basal cell carcinoma is the most common cancer in humans. Basal cell
carcinoma grows slowly and very rarely metastasizes. Like SCC, it is common in
immunocompromised individuals and those with DNA repair disorders. There is a
defect in the PTCH1 gene in the hedgehog signaling pathway. Gorlin syndrome
involves familial occurrences of numerous BCCs. Histologically, basal cells exhibit
palisading arrangements at the periphery. Tumor islands show separation from
surrounding stroma (important for differentiation from trichoepithelioma). Clinically,
pearly papules with telangiectatic appearance are present. Pigmented forms may be
confused with malignant melanoma. Rodent ulcers are typical. It is more common in
sun-exposed areas. Unlike squamous cell carcinoma, it is not associated with HPV. In
basal cell nevus syndrome, hereditary basal cell carcinomas appear at an early age
and in large numbers. Bone anomalies, and abnormalities in the nervous system,
eyes, and reproductive system are present. (Answer: D)
42. Which of the following is a non-painful tumor?
A) Basal cell carcinoma
B) Glomus tumor
C) Traumatic neuroma
385 Pathology Questions and Answers, Prof.Dr. Murat ALPER
D) Eccrine spiradenoma
E) Vascular leiomyoma
Painful tumors of the skin and soft tissue include angiolipoma, traumatic
neuroma, glomus tumor, eccrine spiradenoma, and vascular leiomyoma. Basal cell
carcinoma forms rodent ulcers. Pain is not typical. (Answer: A)
43. Which of the following has the highest risk of progressing to malignant
melanoma?
A) Spitz nevus
B) Compound nevus
C) Junctional nevus
D) Dysplastic nevus
E) Blue nevus
Spitz nevus is pink-red and common in children, clinically resembling a
hemangioma. Blue nevus is a black-blue nodular lesion often confused with malignant
melanoma. Dysplastic nevus resembles a compound nevus but has cells with
cytological atypia. Its borders are irregular, and it is often larger than 5 mm. It has a
high risk of progressing to malignant melanoma, especially when sporadically
occurring in more than ten lesions or in individuals with familial dysplastic nevus
syndrome. In halo nevus, there is dense lymphocytic infiltration around nevus cells. In
junctional nevus, nevus cells are located at the dermoepidermal junction and carry a
risk of malignant melanoma transformation. Congenital nevi may also progress to
malignant melanoma if large. However, most malignant melanomas develop de novo.
(Answer: D)
44. In which of the following locations is malignant melanoma least likely to
develop?
A) Oral mucosa
B) Meninges
C) Anogenital mucosa
D) Bladder
E) Eye
386 Pathology Questions and Answers, Prof.Dr. Murat ALPER
(Answer: D)
45. Which type of malignant melanoma has the worst prognosis?
A) Lentigo maligna melanoma
B) Juvenile melanoma
C) Lentigo maligna
D) Nodular melanoma
E) Superficial spreading melanoma
Nodular melanoma has the worst prognosis among malignant melanomas
because it grows vertically. Lentigo maligna melanoma and superficial spreading
melanoma initially grow radially and have a better prognosis. Juvenile melanoma is a
benign tumor seen in children; previously, this term was used for Spitz nevi. Lentigo
maligna or Hutchinson's freckle contains atypical melanocytes and is a precursor to
lentigo maligna melanoma, a form of malignant melanoma. In malignant melanoma
development, UV-induced DNA damage and mutations in the CDKN2A locus, BRAF,
PTEN, and RAS play a role. 5-10% of cases are hereditary, among which familial
dysplastic nevus syndrome is included. While most tumors occur in the skin, they can
also appear in anogenital mucosa, esophagus, meninges, and the eye. (Answer: D)
46. Which pigmented lesion is most likely to metastasize earlier than others?
A) Dysplastic nevus
B) Nodular melanoma
C) Pigmented nevus
D) Spitz nevus
E) Superficial spreading melanoma
In melanoma, the most important issue is whether the tumor is in the radial
growth phase or the vertical growth phase. Melanoma cells in the radial growth
phase are within the epidermis and lack the capacity for metastasis and invasion.
Over time, as they enter the vertical phase, the situation becomes more aggressive,
with a higher risk of metastasis. Among malignant melanomas, nodular melanoma
has the worst prognosis. Factors influencing prognosis include invasion depth, mitosis
number, presence of regression, presence of tumor-infiltrating lymphocytes, gender,
and location. Tumors smaller than 1.7 mm, with few mitoses, no regression, intra-
387 Pathology Questions and Answers, Prof.Dr. Murat ALPER
tumor lymphocytes, in females and on extremities have a better prognosis. The HMB-
45 marker is used immunohistochemically. Targeted therapies are showing promising
results in metastatic tumors with BRAF and KIT mutations. (Answer: B)
47. According to Clark's classification, in which stage is a malignant melanoma
that has invaded the reticular dermis?
A) Stage I
B) Stage II
C) Stage III
D) Stage IV
E) Stage V
According to Clark's classification, stage I melanoma is in the epidermis. Stage
II has spread to the loose, papillary portion of the dermis. Stage III fills the papillary
dermis and
is between the reticular dermis and papillary dermis. Stage IV is in the
reticular dermis. Stage V has invaded the subcutaneous tissue. In Breslow's
classification, the thickness is measured from the granular layer to the deepest part of
the tumor. (Answer: D)
48. Which of the following malignant tumors is less commonly seen as a
primary tumor in the skin?
A) Malignant melanoma
B) Basal cell carcinoma
C) Kaposi's sarcoma
D) Adenocarcinoma
E) Squamous cell carcinoma
(Answer: D)
REVIEW
(Review and write down what you remember about the following topics...)
388 Pathology Questions and Answers, Prof.Dr. Murat ALPER
Macroscopic definitions?
Microscopic definitions?
Hyperkeratosis?
Parakeratosis?
Acanthosis?
Dyskeratosis?
Acantholysis?
Papillomatosis?
Spongiosis?
Vitiligo?
Ephelis?
Lentigo?
Nevus?
Dysplastic nevus?
Malignant melanoma?
Seborrheic keratosis?
Acanthosis nigricans?
Fibroepithelial polyp?
Keratoacanthoma?
Actinic keratosis?
Squamous cell carcinoma?
Basal cell carcinoma?
Benign fibrous histiocytoma?
Dermatofibrosarcoma protuberans?
Xanthoma?
Histiocytosis X?
Mycosis fungoides?
Mastocytosis?
Urticaria?
389 Pathology Questions and Answers, Prof.Dr. Murat ALPER
Ichthyosis?
Eczematous dermatitis?
Erythema multiforme?
Psoriasis?
Lichen planus?
Lupus erythematosus?
Bullous diseases?
Pemphigus?
Bullous pemphigoid?
Dermatitis herpetiformis?
Epidermolysis bullosa?
Acne vulgaris?
Panniculitis?
Verruca?
Molluscum contagiosum?
Impetigo?
Superficial fungal infections?
Basal Cell Carcinoma
Keratoacanthoma:
Seborrheic keratosis:
Lichen planus:
Spongiotic dermatitis:
Psoriasis:
390 Pathology Questions and Answers, Prof.Dr. Murat ALPER
Malignant melanoma:
---
391 Pathology Questions and Answers, Prof.Dr. Murat ALPER
392 Pathology Questions and Answers, Prof.Dr. Murat ALPER